You are on page 1of 281

PROMETRIC PREPERATION FOR GP

1- initial regulation of BP in vascular system occur at


: a- Arterioles
b-Aorta and its
branches c-Heart
d-Capillaries
e-Vein and venules

2- hematological disease occurs in children treated with heparin


and fresh frozen plasma what is the disease :
A)hemophilia A
B)hemophilia B
C )VON WILL brand disease
d)DIC thrombosis

3- Patient came with low iron and high AST and high MCV
no megaloblasts in the blood wt is the Dx?
Alcohol
Vitamin B12defeciency
Folic acid deficiency
Due to drugs
Patient with macrocytic anemia without megaloblast. What’s the most likely
diagnosis:
a. Folic acid
b. Vitamin B12 deficiency
c. Alcoholism

4- pregnant lady with hepatits, how to confirm dx :


a- ALP
B-SGOT
C-WBC
D-ESR
E-………

5- which of the following antidepressant drugs causes


agitation, anxiety :
triAD

1
tetraAD
SSRI

6- PT WITH ASBESTOSISLEAD TO
Plural calcification
Plural effusion
Or bilateral fibrosis
7- which of the folowing is the most radiosenstive testicular tumor
a- semenoma
b- yolc sac tumor
c- germ cell tuor
d- choriocarcinoma
e- immature teratoma
8- ttt of pyoderma gangrenosum
a- sys steroids
b- methotrexate

9- ttt of foliculitis after shaving the


bread a- oral steroid
b- topical steroid
c -oral antibiotics
may be there was topical antifungal but no topical antibiotic choice
10- Baby in NICU has a heart rate of 300, good blood pressure
level. What should u do:
• DC shock
• IV amiodrane
• Digoxin
• Carotid massage??

Newborn with 300 bpm , with normal BP , normal RR , what do you will
do for newborn : (atrial flutter) imp.
🗷 Cardiac Cardiversion
🗷 Verpamil
🗷 Digoxin
🗷 Diltzam iv

if you suspect atrial flutter : Consider digoxin if not already in use


because it frequently increases the conduction ratio and decreases
the ventricular rate. , Avoid adrenergic and atropinic agents during
sedation or anesthesia for cardioversion. Ketamine is relatively
contraindicated

_
2
the cause of premature ventricular contraction?
decrease O2 supply to the muscle
decrease blood supply to the muscle
the heart trying to beat fast

11- yr old sustained trauma to the chest present with severe short
of breath with cyanosis, his rt lung is silent with hyperresonance.
The FIRST step to treat this pt:
a. O2 mask
b. Tube thoracostomy
c. CXR

12- Patient with hypertension , DM, somking, which the following


are most important to be deal with :
obesity and HTN ???
smoking and obesity
smoking and HTN

13- baby with streptococcal pharyngitis:


- Ttt after 9 days carries no risk of GN
- Ttt effective in prevention of GN
- Clindamycin effective against gram –ve
organisms all choices are wrong

14- Hx of trauma in DIP(finger hyperextention)with palm


pain: (incomplateQ)
- Extraarticular fracture in DIP
- Intraarticular fracture in PID
- Superficial tendon tears
- Tendon profundus tear??
15- pt with pict of bilateral pneumonia , high grade fever , normal WBC
. organism causing that:
- Chlamydia pneumonae
- Leigonella sp. (cause neutropenia)
- Staph pneumonae

16- ttt of cold induced urticaria:


- Cemitidine
-diphenhydramine.

17- methyl-progesteron used for PPH what is contrindication


: Pregnant with asthma
Pregnant with
hypertension Pregnant
3
with DM

4
18- what is the most common cause of death in patients with
Ludwig's angina?
sepsis
Sudden asphyxiation
rupture of the wall

19- 4 days post c-section pt with profound


hypotension a-normal saline 5ooml IV with to big lines
b-dopamine

20- Adult male during excercise he suddenly felt pain in the middle
of his rt. Thigh posteriorly. On exam. He has discoloration in the
same site and mass in the hamstring ms. No bone tenderness or
palpable defect. Mx:
Surgery.
Splint.
Bandage.
Ice, elevation and
bandage Cast.
21- patient has complete ptosis in hih rt eye. pupil is out and
down, fixed dilated. restricted ocular movements. dx
a. 3rd n palsy.
b. 4th n palsy.
c. 3rd and 4th.
d. 6th n palsy

22- target lesion are found in erythema:


a-annular
b-marginatom
c-multiform
d-nodosum

23- patient came with cervical carcinoma next investigation :


-cone biopsy
- Direct biopsy
-pap smear

24- the best to give as DVT prophylaxis post surgery which is


cost effective, safe with high efficacy:
a) LMWH
b) Unfractioned Heparin
5
c) asprin
d) Warfarin
25case of right hypocnodrial abdominal pain for 2 hours after eating
for months
no fever no jaundice, no radiation and nothing suggistive of
cholycytitis
what is the investigation
a- US
b- oral cholystogram
c- iv cholystogram
26-72- pt intubated ,the most reliable method to make sure for tube
proper position:
- 5 point auscultation bilaterally breathing heard
- CXR
27-23 years old female with regular menses. On US, she has a 7cm
ovarian cyst. otherwise everything is normal. dx:
a. corpus luteum cyst
b.follicular cyst
c.teratoma
d.another cancer

30- regarding paracetamol toxicity:


- Not toxic if dose exceed 150-180 mg
- Cause vomiting and neuropathy
- Therapeutic effect after 4 hours
- Use Deferoxamine
-the liver enzyme reach the max. Level 4-6 hours after ingestion

31- 22 yr, low HGB low PLT and high WBC , peripheral smear shows
blast cell with large nucleus and scant cytoplasm and some nucleoli
-- positive meyloperoxidase test and negative esterase , DDx:
- Acute lymphocytic
- Acute myelocytic
- Acute monocytic

32 child with hyperemia and pulging of tym mem – had previous


history of treated impetigo so ttt is:
- Cefuroxime
- Amoxicillin (not sure)
- Erythromycin
- Ceftriaxone
- Cephalexine

6
33-child , urine odor like burned sugar:
- Phenylketonuria
- Maple syrup urine disease

34-40 yo presented by hx of syncope when he do excurses and there


was hix of same complain when he on rest . and there is chest pain
on ex. There was ejection systolic murmur 2-4 degree most side is
lower lf sternum not radiating to other site increase when he lying
down and there is non specific s and t changing and there is lf atrium
enlargement
Aortic stenosis
p.s
hypertrophic cardiomyopathy
constrictive cardiomyopathy
35-Pt. had chest pain and fainting , ECG shows st- elevation
and significant Q wave in -v4 and st-depression in inferior
leads : Ant. MI
Inf. MI
Pericarditis
Post. MI

36-A patient with normal kidney function post MI. The troponin level
will last for :
A. 48 h
B. 73 h
C. 24 h
D. 12 h
E. 8 h

37- In lung diseases like pneumonia, it usually affects certain site


based on the anatomy, this is more obvious in which of the
following:
• Right upper lobe
• Right middle lobe
• Right lower lobe
• Left upper lobe
• Left lower lobe
38- in lung diseases like pneumonia, it usually affects certain site
based on the anatomy, this is less obvious in which of the
following:
• Right upper lobe
• Right middle lobe
• Right lower lobe
7
• Left upper lobe
• Left lower lobe

39- Pt. above 40 , pregnant and ask you for down syndrome screening
: Triple test
US
Amniocentisis
Chorionicvillus
40- You received a call from a father how has a son diagnosed
recently with DM-I for six months, he said that he found his son lying
down unconscious in his bedroom, What you will tell him if he is
seeking for advise:
a. Bring him as soon as possible to ER
b. Call the ambulance
c. Give him his usual dose of insulin
d. Give him IM Glucagone
e. Give him Sugar in Fluid per oral

41- Pt. with headache and vertebral lesion (Moth-eaten),Investigation?


- Bone scan
- …………………………
- ……………………………

42uveitis is treated by
a-chloramphinicol drops
b-tetracine drops
and other choises i dont remember
uveitis is treated with cycloplegic and steroid

43- Which drug can use in acute back pain


Diazepam
Alprozam
Metoxelen
44- patint C/o menomentogia 6 month and pelvic pain .all investigatin
and examinatin normal what is ttt
Mafnemic
Compined pill

45- pt was PDD –ve , know become + ve , there is no symptoms


, normal x ray, the management :
-Reassure
-Rifambicin and INH for 6 month

8
-Streptomycine for 7 month
_ rifambicin for 6 months

46_ pt was PDD –ve , know become + ve , there is no symptoms ,


normal x ray, the management :
Isonized and rifampcin for 6 month
Isonized for 6 month
Isonized and rifampcin and streptomycin for 12 month

the treatment of latent TB:


1-INH for 6-9 months
2- alternative is rifampicin 4-6 month

47_During blood transfusion , the pt develop fever and pain at


infusion site – your action:
-slow infusion+antibiotic
-slow infusion + acetaminophen
-stop infusion + crystalloid fluid
- stop infusion+ mannitol+acetaminophen

48To deffrentiate between sinua arythmia and atrial firbllation


a- carotid massage
b- Temporal artery massage
c- Amidrone
d- digoxin

49A 35 yr old pt she is on phenytoin since she was 29 due to partial


epilipsy she didn’t have any attack since. She want to stop taking the
drug due to facial hair growth:
a. It is reasonable to stop it now
b. Stop it after 6 months
c. Stop after 10 years
d. Don’t stop it
50-Patient with seasonal watery nasal discharge, sneezing and nasal
block. What should you give him as a treatment:
• Topical steroid
• Decongestants
• Antihistamines
• Systemic Steriods
51child presented with decreased hearing for 1 year, on exam. there
is fluid behind the ear drum and adenoid hypertrophy. In addition to
adenoidectomy what will you do:
9
myringotomy.
gromet tube insertion.
antibiotics.

52-Pt. with perforated tympanic membrane ttt:


Topical ABX
Topical steroid
Systemic ABX
Systemic steroid
53_Pregnant lady presented with bleeding from gums. On exam,
spleen in palpable 4 cm below the costal margin. Ix; platelets 50,000.
Dx:
HELLP.
ITP
Gestational thrombocytopenia
Thromboembolic disorder.

5440 ys female, multigravida, no sexual intercourse for 1 year bcz her


husband going abroad, C/O was intermenistrual bleeding with
menorrhgia, provotional Dx:
Endometriosis
Endometrial CA ??
ch endometrites
this is a common age of DUB not sure of the answer but according to
the choices it is B

55- patient with bed sore involve skin and extend to fascia and
muscle what a grade
Grade1
Grade 2
Grade 3
Grade 4

56- patient with rhumatic heart disease and had mitral valve stenosis
Mitral vave diameter less than 1 mm
In order to maintain COP what will happen :

Left atrial hypertrophy and decrease pulmonary


prusser Left atrial hypertrophy and champer dilatation
RV hypertrophy and decrease pulmonary
prusser RV hypertrophy and champer dilatation

10
choices in other words
a- Dilatation in the atrium with chamber hypertrophy
b- Dilatation in the ventricle with chamber hypertrophy
c- atrium dilatation with decrease pressure of contraction
d- ventricle dilatation with decrease pressure of contraction .

57-55 years male with bleeding on examination have external


hemorrhoid what to do
advise him to remove it
do rigid sigmoidscopy
go home and visit after 6 months
do barrium enema
58-infant with high grade fever .. Irritable .. Look sick .. Complain of
anuria 4 hour with multiple petechiea and purpura on body .. He was
tachycardic and hypotensive DX
Renal fauiler
Septic shock
59Verrryy long scenario of old age pt with DM, HTN, hx of multiple
cardiac attack, CVA, came for routine check up in PHC, u found
bilateral opacification in both lenses, with decreasing of visual acuity,
u will:
Refer to lazer therapist
refer to cataract surgeon
refer to ophthalmologist
follow up
60- patient on glaucoma medication for weeks came with SOB,
cough the cause
a- timolol
b- betoxolol
c- pilocarpin

61- Considerd positive mantux test in


Erythema more than 5 mm in HIV
patient
Induration more than 10 mm in diabetic patient
Induratin more than 5 mm in iv drug abuse
Induration more than 10 mm in philpine man

62- old with bilateral hydronephrosis:


- Stricture of uretheral meatus
- Prostate enlargement
- Bladder tumor

11
_

An 80 year old male presented with dull aching loin pain & interrupted
voiding of urine. BUN and creatinine were increased. US revealed a
bilateral hydronephrosis. What is the most probable Dx?
a) Stricture of the urethra
b) Urinary bladder tumor
c) BPH
d) Pelvic CA
e) Renal stone

63- old man healthy felt in collapse before he collapsed there


was epigastric discomfort , came with pain n the back, pulse
114, bp 140L…dx:
Perforated peptic ulcer
Leakage aortic aneurysm

64- in pt with RA to preserve joint function and movement:


-disease modifying medication is sufficient
- fish oil gives subjective improvement
- cold (ice) compression and decrease joint movement

65- patient with breast cancer and metastasis came complain of


tachycardia hypotension , engorged neck vein and SOB what is
most next action
D-dimer
Ventlation prefusion scan
Give fursamide and refer to do echo

66- 19 year old athlete, his weight increase 45 pound in last 4 months
. in examination , he is muscular , BP 138/89 . what is the cause
1.alcohol
2. cocaine abuse
3. anabolic steroid use

67Which antiviral drug causes fever and muscle pain:


Acyclovir
Oseltamivir
Interferon
Gancyclovir
12
68-5 yr-old baby presented with his parents with pallor his HB is 9, he
has microcytic hypochromic anemia, no other complain .. what u'll do
for him ??

iron therapy and close observation


daily multivitamins with iron

69- patent with vaginal discharege ,suprabubic pain for 3 days ,fever
and bilateral fornieces tenderness what is the Dx
1- apendicytis
2- acute salpyngitis
3-chronic salpingytis
70- patent is presented with SOB. On Xray he has cardiomegaly and rt.
pleural effusion. pleural aspiration reveals: protein is <30, LDH <200
IU. dx:
CHF.
pneumonia.
TB
hyperproteinemia.
71- pt have mutiple risk factor , obese , HTN not on medication BP
130/90 , unhealthy diet, hyperlipediemia, lack exercise – which
factors control improve survival:
- Cholesterol, HTN, obesity ???
- Cholesterol, sedentary lifestyle , diet
- Triglyceride, obesity, HTN
- Low HDL, ??, ??

72- 44- male old patient has S&S of facial palsy ( LMNL) ; which of
the following correct about it;
A- almost most of the cases start to improve in 2ed
weeks b- it need ttt by antibiotic and anti inflammatory
c- contraindicated to give corticosteroid
d- usually about 25 % of the cases has permanent affection

73- pateints with appendicits what is most helpful to make


DX Age
Fever
High
WBC
High sedmentation rate

13
74- Child with high fever and after 2 day develop sorethorate on
examination there is congested thorat and pharynx and white to
yellowish papule on erthymatus base in mouth and lip what is most
likly DX
Coxsacki virus
Herps simplix
virus

75 Which drug can not be use in acute cholysystits


Naproxen
Morphine
Mepriden
Acetamenophin
Perdoxyphen

76pt na 123, k 3 what to do


a-normal saline with 20 meq kcl 80
cc/h b-normal saline with 5 meq kcl
20cc/h
c-half normal saline with 20 meq kcl 80
cc/h d-half normal saline with 5 meq kcl
20cc/h
77female after placement of IUD , she develop abdominal pain and
watery brown discharge ,
1, uterine rupture
2, bacterial vaginosis
3, pelvic inflammatory disease

78- Female with recent hx of IUCD insertion, coming withwatery


brownish vaginal discharge & abdominsl pain what is th most likely
dx: a. Uterine rupture
B. Ovarian torsion
C. Bacterial vaginosis
D. Ectopic pregnancy

NO PID here !!!!!! in alqaseem questions diagnosed as


ectopic pregnancy not sure

79 Rx. Of scabies in pregnant women:


permethrin

14
80- Profeational player came with history of truma on the
lateral side of left knee , on examination there is swelling in
the medial aspect of left knee , the diagnosis is :
a- Medial collateral ligament spasm .
b- Lateral collateral ligament spasm .
c- Medial meniscus tear .
d- Lateral meniscus tear
81- Patient he had multiple problem in his chest and
he lives in crowded area what your action:
a- Immunoglobulin
b- H.influnza
c-meningococal

82- Pt came with cough , wheezing , his chest ascultation revealed


monophonic sound , on xray ther is patchy shadows in the upper
lobe+ low volum wirh fibrosis ,, he lives in a crowded place .. What
is the injection shuold be given to the pateint's contacts :
hemophe.influanza type b
Immunoglobuline
Menngioc. Conjugated C
Basil calament.......!!?
83- young male complains of generalized skin lesions and redness,
before that there is a hx of mouth and lips swelling for couple of
days the its denied any hx of traveling or unusual exposure
the is the Dx ?

?? urticarai
coxsackievirus infection
cold urticaria
hot urticaria

84- paient suspected to have connective tissue disease what is


most favurable to SLE
Cystoid body in fundoscopy??
Cavitaion in lung
+ve anti RNP
Sever Ryundoe phenomena

85- pt with pycosis on medication developed rigidity and uprolling


eyes , afebrile :
- Tradive dyskinesia

15
- Malignant neuroleptic
- Hypotonic
86- What you will find in patient with idiopathic autonomic
insuffeciency?
Orthostatic hypotension
Horner syndrome
Anhydrosis
Palpitation
Diaphoresis

87- What is true about hormonal


contraception? Decrease breast Ca
Decrease ovarian Ca
Contraindecated in diabetic women
Increase risk of ectopic pregnancy

88- Patient with colon cancer stage 3 and chemotherapy was


prefered so when do you start it?
As soon as possible
When the lab results normalized
After psychological preperation
No need to start
89- young female complains of 6 weeks amenorrhea and history of
VP bleeding for many days and by laparoscopy the is free fluid in
douglas of pouch(I don't remember the exact NO.) what is the most
probable cause??

rupture ectopic pregnancy


90- pregnant woman with UTI which is the best antibiotics to be
given if she has no allergy?
nitrofurantoin
ampicillin
sulfatrimethoprim
tetracyclin
aminoglycoside

91Pt has a scaly hypopigmented macules on the chest and arms


They seem even lighter under the sunlight,,, what is the
ttt? (diagnosis s Pityriasisalbaorpityriasiversicolr)

Topical
steroid Na

16
selinum

17
Topical antibiotics
Oral antibiotics

25 years old male complaining from scaly lesion in his chest , then
become hypopigmented , last 2 months in winter he spend his time neat
to sea, by examination showed hypopigmented lesion over chest &
arms Dx :
Vitiligo
taenia versicolor

92 attributable risk factor is :::???

measurement of exposed and not have the disease mius those


exposed and have the disease

93-29 years old male diagnosis as case of gastric ulcer , culture -ve
h.pylori pathology –ve of cancer ttt:
proton pumb inhibitor
antihistamine
after 6-8 wk do endoscope after therapy
referral to surgery

94-32 years old with cystic mobile breast mass, no LN


enlargement. What would you do:
• Aspiration with cytological evaluation
• Reassurance
• Fluroscopic biopsy
• Mammography then discuss the options according to the
new information

95-Pt with hix of URTI for 3d then stat to develop rt. Ear pain
rinnen test was negative and wiber test (loud sound in affected
side) Mastoditis
O.M
O.E
96-3 months old baby brought by his parents complaining of abd.
distention bilious vomiting, constipation, the parents informed that
the constipation has been an issue since his birth
what is the single diagnostic investigation to do ??

18
barium enema
pain xray
??metery
rectal examination
97- A patient presents with long time history of knee pain suggestive
of osteoarthritis. Now he complains of unilateral lower limb swelling
and on examination there is +ve pedal & tibial pitting edema. What is
the next appropriate investigation?
a. CXR
b. ECG
c. Echocardiography
d. Duplex ultrasound of lower limb
98- patient came with MI 2 day after addmission develop. Sever
abdominal pain and bloody diarrhea ‫وفياشياتانيهنسيتها‬DX

Ischemic colitis
- diffuse abdominal pain , bleeding per rectum and fever 38.3 c ,
preceded by urinary infection 3 weeks back treated with AB ,
diagnosis :
Ischemic colitis
Amoebic colitis
Pseudomembranous colitis

99- looong scenario about old male came with typical history of
MI all of the following can be used in the mX of this its except:
atenolol
heparin

sorry I couldn't remember the rest of the choices :(

100- Pt. with long hix of hyperthyroidism to screen about the


complication of hyperthyroidism do:
liver us
Ct brain
Bone scan (for osteoporosis)
101- to increase absorption of oral iron , give with:
- Vit C
- Vit E
- Zinc
- Ca
- Antacid

19
102- old, black macule on his back with irregular border and
color variation :
- Sq cell carcinoma
- Basal cell carcinoma
- Melanoma
- Acanthic keratosis
103--cord prolapse at level L4-L5 the patient will presented
by a-painful calf muscle
b-absent ankle jerk
c-parethesis of knee
joint d-weak dorsiflextion
104Old male with acute pancreatitis, (high glucose, low Ca)the
appropriate nutrition:
TPN
Regular diet with low sugar
High protein ,high ca , low
sugar Naso-jujenal tube
105- stroke with loss of smell, which lobe is affected
: a- frontal
b-parital
c-occipetal
d-temporal

106- Adult P.t recived a vaccine ( i don't remeber the name


) After that he complain of itching , tachycardia and SOB
What is the ttt?
IV hydrocortizone 500 mg
SC epinephrine
107- polycythemia vera ttt:
-Myelosupression
-Plephotomy
-X ray therapy

108- CHILD with eczema on 1% hydrocortisone what other


medication u can add
Dexamethazon
Cyclosporine
Tacrolimus
109- Pt with heartburn use antiacid for long time but not improve what
is appropriate drug he can use
H2 reseptor antagonist
PPI
…..ANTIACID

20
110- OSTIOMYLITIS start in
Metaphysic
Epiphysis
Dyaphysis
111- Most benign cause of postmenopausal bleeding
Cervical polyp
Atrophic vaginitis
112- Pt came by hx os sudden eye pain burning vision photophobia
and by ex. Small pupil and keretic cell on cornea and cell in
humorus Ttt
Cyclospoine + corticosteroid
113- which of the following is not a feature of normal ECG:
-P wave is the repolarization of the atria

114- n normal puerperium..


-lekoria lasts for up to 4 weeks
-the uterus can't be felt after the 1st week in abdomen
epidural analgesia can cause urinary retention
115- best test to detect age of gestation is
-LMP
-U.S.

116ibuprofen is contraindicated in
-htn
-dm
-peptic ulcer
117-a pt with AF came with black stool (and i think hypotenstion)..dx
is:
-ischemic mesntry
118- Female with greenish vaginal discharge, red cervix(srawberry
appearance). under the microscope it was a protozoa..Dx:
a. Trchimoniosis

119 Old man with left lower abdominal pain with fever and
constipatin, imaging showed decreased the fatty shadows around
distal colon, your next step:
Double contrast
IV antibiotic
Control diet
120- Child with SCD, about pneumococcal vaccine
- give 23 valent in high risk only
- give heptavalent after 2 yr

21
- child with high risk give the vaccine along with antibiotics when
exposed to infected ppl )

121- Which of the following increases the quality of the


randomized controlled study & make it stronger:

a. Systemic Assignment predictability by participants


b. Open Allocation
c. Including only the participants who received the full intervention
d. Following at least 50 % of the participants
e. Giving similar intervention to similar groups

122- self breast examination:


a. 3-5 day after period
b. 7-10 day ''''''''''''''''''
c. 2weeks ''''''''''''''''

123right lung anatomy


1. one fissure
2. 7 pulmonary segment
3. no relation with azygus
vein 4. 2 pulmonary veins
5. no sibson's fascia

124- antidepressant action starts within


-1 day
-1 wk
-2wk
-3-4 wk
125- child came with generalized body swelling, fever , dark urine
with decrease urine output ,,, what is the most useful investigation for
diagnosis:
CBC
Renal function test
Abd. US
Urine sedmintation test
126-a man with 2nd and 1st degree burn over his face and neck
a) wash, cover all burns with Silver sulfadiazine, cover with sterile
gauze, give IVfluid, antibiotic and tetanus toxoid and discharge
home with daily dressing
b) cover burn with Silver sulfadiazine, sterile gauze, oral fluid,
and discharge home
22
c) doesn’t make sense
d) doesn’t make sense
e) Silver sulfadiazine, sterile gauze, IV fluid and admit to hospital
127-pateint with decrease lipido and weak erection ( or ejaculation)
In investigation prolactin high , LH and FSH normal what is next
step Brain MRI
Abdomen and pelvic CT
‫اخترتالمارايبرينالناتكثرسببللبرولتكتنيمياهوالبيتيوتريادينوما‬

Patient with Premature ejaculation + libido + Erectile dysfunction he is


thin and looks sad, he is married for 26years obese and annoying
wife, he came for treatment:
A- Testosterone Injection every one week
B- Sublingual Nitroglycerin 6h before intercourse
C- SSRI
D- ????

1284 or 5 ( not sure ) brought by his parents with weight > 95th
percentile , height < 5th percentile & bowing of both legs what is the
appropriate management :
a- Liver & thyroid function tests
b- Lower limb X-ray
c- Pelvis X-ray
d- Thyroid or ( liver not sure ) function test
129-18month old boy came with bite by her brother what you will
do ?
A) augmentin
B) titanus toxoid
C) suture

130- old pt with 2 years bone pain , lethargy , fatigue, wedding gait ,
came with table show high calcium and high phosphorus ;
A_ osteoporosis
B_ osteomalacia
C_ paget disease of bone
D_ metastases prostate cancer
E_ paraneoplastic syndrome

131- child with inferior and pain but with normal


movment of knee , no effusion on knee what the important thing to
do ;
23
A_blood culture
b-ESR
c_ASO titer
d-aspirate from knee joint
d-plain film on thigh

132-miliary TB caractarized
by a- spare lung apical
(b- septal line
c- multiple lung nudules

133-a man who has had MI you will follow the next enzyme
a) CPK
b) ALP
c) AST
d) Amylase

134a child of parents who have TB, PPD test done for him and
revealed 10 cm induration, this is
strong +ve.
intermediate +ve.
weak +ve.
-ve
135- in aspirin overdose:
a) liver enzyme will peak within 3-4 hr
b) first signs include peripheral neuropathy and loss of reflexes
c) 150 mg/kg of aspirin will not result in aspirin toxicity

136- female pregnant has HIV +ve , what is the most


accurate information to tell her about risk of transmition to
baby ;
A-likely transmtion through placenta
b-through blood cord
c-hand contamion of
mother d-by breast feeding

137- Female with dysurea, urgency and small amount of urine


passed .. she received several courses of AB over the last months
but no improvement .. all investigations done urine analysis and
culture with cbc are normal .. you should consider:
a) interstitial cystitis
b) DM
c) Cervical erosion
d) Candida albicans
24
138- patient is presented with acute chlangitis, what you will do to
alleviate the symptoms:
IV antibiotics + gastric lavage.
IV antibiotics + drainage of bile.
hydration + chlolecystectomy.

139- Which of the following drugs prolongs QT interval as side effect?


a. Respridone
b. Clozaopine
c. Amisulpride
d. Aripiprazole
e. Ziprasidone

140- Which of the following is a 3-hydroxy-3-methyl-glutaryl-


CoA reductase (HMG CoA)?
a. Statin
b. Fenofibrate
c. Niacin
141- picture of bulls in food ... In biosy there is epidermal lysis and
on immunoflurescen: deposition of IgG DX
Bulls pemphigoid
Pemphigoid valgarius

142- perthes disease all except


Can be presented with painless
limp It always unilateral

-how to calculate -143relative risk

144-- paient complain of infirtility 6 year ago and sever pain with
cycle ( dysmeanorhea) DX
Endometriosis
Pelvic congestion
Endometritis

145-you want to give varcilla vaccine in one no have vaccin before


‫كيفتعطيه‬
Two dose and 6 weak between

146-- pregnant not vaccinated against measls and mumps and


rublla .. She exposed to rublla 3 day ago what you do
25
No treatment
Immunoglobin
Tell her no affected on her pregnancy if she take the vaccine
147- Pateint complian of diplopia , weakness , and frequant aspiration pnumonia in last
2 month ... In examination there is spascity and fasciculation DX
Mythenia gravis
Mythenia syndrome
Motor neuron disease

148- child with low grade fever , sore thorat in examination there is lymph node
enlarment but not tender and no exudate on phrynx DX

It is most likly streptococcal than viral


It is viral more than bactrial
Most likly EBV
149- what is the. symptom Most likly occure with hiatus hernia
Skin pigmentation
The symptom increase with pregnancy

150- patient with bilateral eye redness . Discharge and tearing on examination cornea
, lens all normal Nd tere is conactival follicle DX
Acute conjunctivitis

151- child C/O fever , sore thorat all examination was normal What is the ttt
: Cefruxime
Ceftriaxone
Give paracetamol and take pharynx swab

152-- pateint C/o ictrus in skin and eye on investigation WBC 2500
plt 70,000 HG 7 lekocytosis 17% total bilirubin 51 and direct bilrubin 12 what is the test
most likly positve
+ve coomb's test
In US obestructive billiary duct
antiparietal antibodies

‫ فاكسين يحميه طول‬... -153 ‫ناسيه كيف كان السؤال بس الخلهصه ان في مريض سالك ليش ياخذ فاكسين النفلونزا كل فتره ليه مافي‬
‫العمر‬
Because the organsim develop resistant
Develop new antigenic drift
154- patient c/o low self steam and fatigue .. Lack of intersted and concentration loss of
sleaping , depressed mood for last 2 years what DX
Dysthymic

26
155--lacteting mother complain of fever and breast tenderness and redness diagnosed as
bactrial mastitis what is ttt :
Continoue breastfeeding and hot compresser and antibiotic
Discontinue breast feeding and give antibiotic to mother and baby
156- the most common cause of nipple discharge in non lactating
women is ;
a-prolactenoma
b-hypothyroidism
c- breast CA
d-fibrocystic disease with ductal ectesia .
e ductal papiloma

157- with patient has fear ,SOB ,sweating when he is in


automobile DX
a-specific phobia
b-panic disorder
c-generalize anxiety disorder

158which of the following causes the highest maternal mortality in


pregnancy
a. toxoplasma
b. hyperbilirubenia
c. pheochromocytoma
d.rubella

159-Old pt presented with abdominal pain, back pain, pulsatile


abdomen what's the step to confirm dx:
a. Abdominal US
b. Abdominal CT
c. Abdominal MRI

160The most common cause for chronic irregular rectal bleeding


is: Diverticulitis
Hemorrohids
Colon cancer
UC
161the most common cause of excessive day time sleepiness is
1- circadian rhythm
27
2nacrolepsy
3-sleep apnea

162-the most common malignant tumor of parotid in childern


1-acinic cell ca
2- mucoepidermoid ca
3- adenocarcinoma
163- The useful exercise for osteoarthritis in old age to
maintain muscle strength and bone density
 Low resistance, high repetition muscle training
 Conditioning, low repetition muscle training ??
 Walking and endurance muscle training
 Low resistance and conditioning muscle training
164- pregnant with uterine fibroid , has no symptoms only abd. Pain , US showed live
fetus ,,,,, What is the appropriate action to do:
Myomectomy
Hysteroectomy
Pain management
Pregnancy termination
165Pt came with eye pain, watery discharge and light sinsitivity
Eye examination showed corneal ulceration. Her symptoms are frequently repeated .
Which of the folowing is triggring for recurrence of her symptoms:
Dusts
Hypertension and hyperglycemia
Dark and driving at night
Ultraviolet light and stres
166p.t taking a medication , came to the ER suspecting she has overdose of her
medication, her symptoms ( convulsion, dilated pupil, hyperreflexia and strabismus) the
medication is:
TCA
SSRI
Hypervitaminosis

167Pt complain of hearing voices from the microwave and refrigerator


Visual hallucination
Auditory hallucination
168- Old retired man having ansomnia only . Has no symptoms related to anxity
or depression .. U will give him :
Diazepam
If zolpidem is in choices it is more accurete

169-pt take cephalexin after tooth extraction for days


After that he develop profusre , green foul smilling diarrhea with low grad fever . He
28
has tachycardia and mild abdominal dist. Sigmoioscopy showed white mucosal
patches , what is the most ttt for this condition?
Clarythromycine
Vancomycine
Cephalosporine
Lineozides
170-pt with COPD came with couph , wheezing and greenish sputum
The causative organism:
H.influanza
Strep.pneumonia
Chlaymedia
Mycoplasma pneum.
171-what is the most effective measure to limting the complications in
COPD: Pnumococcal vaccination
Smoking cescation

172-25 years old female came complaining of difficult hearing , she mentioned that
their a family history of early oncet hearing loss ( her grandmother)
Oto. Exam was normal .. Weber and rinne tests result in ( bone conduction is greater
than air conduction ) ... Next action is :
Refer her for aid hearing
Tell her there is no avalible ttt
Refer her to otolaryngologist

173Old man came complaing of progressive hearing loss , it is mostly profounded


when he listining to the radio, he does not has any symptoms like that before
Weber and rinne tests result in bilateral sensorineural hearig loss.. Diagnosis:
meniere's disease
Otoscelerosis
Noise induced deffnese
Hereditary hearing loss
174-25 y wear glass 10 y and diagnosis DM type 2 when u do eye
screen 4 her:
6m
12m
2y
5y
175-Pregnant women has fibroid with of the following is True:
Presented with severe anemia
Likely to regress after delivery
Surgery immediately
d. Presented with Antepartum Hemorrhage
176-70y male with osteoporosis the T score of bone density would be:

29
-3.5
-2
1
2
3.5
177Pt G3 P3 all her deliveries were normal except after the second
one she did D&C for retained placental parts, presented with
amenorrhea after a period of irregular cycle, labs all normal except :
high FSH, high LH, low estrogen DX:
a- Asherman syndrome
b- Ovarian failure
c- Sheehan syndrome
d- Turner $
178-The most dangerous red eye that need urgent referral to
ophthalmologist:
1. associated with itching
2. presence of mucopurulant
discharge 3.bilateral
4.associated with photophobia
179Neonate with mucopurulant eye discharge lid swelling and culture
positive for gm –ve diplococcic , treatment (neonatal gonococcal
conjunctivitis)
1. intravenous cephalosporin
2.topical sulfonide
3. oral floroquinolol
4. IM aminoglycoside
180a baby with blood in the stool and bought of crying and x ray
shows obstructive pattern.. looks like intussusception you will do:
a) surgery
b) Barium enema
c) observation
d) giv e IV fluids and let obstruction solve itself

181-pt with nasal congestion, watery nasal discharge and


conjunctivitis, ttt:
a- oral antihistamine
b- Na cromoglycate
c- Topical steroid
d- ??
182initial treatment of OA in adult who has knee pain bilaterally:
a- Opoid
b- Intraarticular steroid
c- Quadriceps strengthening exercise
30
d- ??
183IV drug user has macular rash on palms ,splinter he, and
ophthalmoscope shows macules with clear center in retina DDx:
Syphilis
Infective endocarditis

184patient with red eyes for one day with watery discharge
No itching or pain or trauma (nothing indicate allergy or bacterial
infection)there is conjuctival injection
visual acuity 20/20
what is next management

antihistamines
topical AB
No further management is needed
refer to ophthalmologist
topical steroids

if allergic rhinitis :topical steroid


second line:antihistamine
185- newborn apgar score 3 (cyanotic, limp, decrease breathing,
HR less than 60) your action:
- Volume expansion
- Chest expansion
- Ventilation
- Bicarbonate
186- pt presented with sweating, myosis, and garlic breath odor:
- Organophosphorus toxicity
- Cyanid toxicity
- Alchol
- DKA
- Cocaine toxicity
187-in rheumatic fever:
- Bacteria in blood
- Bacteria lodge in myometrium
- Skin invasion
- ???
188-86- female G3P0 , c/o infertility , have regular non heavy cycle,
trichomonus infection treated at age of 17 , previous 3 elective D/C in
first month gestation ,DDx:
- Asherman $
31
- Sheehan $
- Endometritis
- ???
189-40 yr heavy and intercyclical bleeding , not pregnant , does
not on OCP:
- Anovulatory cycle
- ????
190- smoker , CXR shows lung mass, hyponatremia and diluted urine:
- Heart failure
- SIADH
- Renal failure
- Conn's disease
191- common cause of AOM in all age groups:
- H influenza
- St. pneumonae
- ???
192old, which fracture caused by trauma on outstreatched hand:-
colle's Fx
193 female, malodor vaginal discharge, dysuria, normal
urinalysis, leukocyte and gram -ve diplococci :
- N gonerrhea
194 old, black macule on his back with irregular border and color
variation :
- Sq cell carcinoma
- Basal cell carcinoma
- Melanoma
- Acanthic keratosis
195
osteoporosis risk

65 75 80
According to above graph:
- 18 % develop osteoporosis after age of 80
- 80 % of elderly have osteoporosis
- Age directly related to risk of osteoporosis
- Pt after 80 at high risk of osteoporosis
196 upper limb HTN , decrease lower extremities pulsation:
- Coarcutation of aorta
197 pain and swelling at first metatarsophalyngeal joint:
- Na urate crystals
32
- Ca phosphate crystals
- ???
198 pain and swelling at first metatarsophalyngeal joint:
- Na urate crystals
- Ca phosphate crystals
- ???
199 old, smoker , rectal bleeding , wt loss: >>>>Colorectal cancer
200- 45 years old female came to ER with acutely swollen knee + ballotment patella ..
The most important to do is:
MRI of the knee
Aspiration
Complete blood count
Rhumatoid factor
201- Pt came with a history of about 12 dayes duration severly red , swollen painful first
metatarsophalangeal joint.. He is hypertensive with inverted T wave on ECG ... The
most appropriate meaure for diagnosis:
CBC
Uric acid level
Troponin level
C-reactive protein
202- Pregnant on iron supplementation throughout her pregnancy for her anemia , now
she come complaining of weakness and easy fatigability
Her Hemoglubin 7 , MCV 60......What is the diagnosis?
Iron def. Anemia
Hypothyrodism
Vit B12 def.
Beta thalassemia
203- baby who can name 4 colors......His Age :
48 months ( 4 years )
‫ـــــــــــــــــــــــــــــــــــــــــــ ـــــ‬
204- Pt came after fight ( gunshot ) there is a pice of the omentum coming out from the
wound . Vital signs ( HR 98 , BP 130/80, RR 18 ) .. What is the best action to do ?
CT
DPL
Fast us
Wound exploration
Scheduled laprotomy

205- about which breast mass present with bloody discharge


? intraductal papilloma

206- Most Dangerouse sign during pregnancy?

33
Vaginal bleeding
207- the most common cause of epistaxis in children is:
Nasal polyps
Self induced
208- one of the folowing manifest. As croup:
Forigne body
Pneumonia
Common cold
Asthma
209- clear scenario of varicocele ( bag of worms scrotum )
210- scenario of glucoma in old pt ,, what is the best ttt?
Acetazolamide + pilocarpine ( sure 100% )
211- clear scenario of keratitis .. on examination there is dendritic ulcer:
Herpes simplex keratitis

212- Mass in the upper back .. with punctum and releasing white
frothy material…
a- It's likely to be infected and Antibiotic must be given before
anything
b- Steroid will decrease its size
c- It can be treated with cryotherapy
d- It must be removed as a whole to keep the dermis
intact 213-Drug use in CHF with systolic dysfunction?
Nifidepine*
deltiazm*
and two drugs from ACEI I forget their names *
the 5th choice is one of B blocker*
214-A patient with severe headache, behind the eye, 4 times in
one week ? with other symptoms
(i don't remember the whole scenario, it seems a cluster headache
case, not sure)

which drug is not useful in prophylaxis:

CCB (may be verapamil)


methysergide
valium
lithium
prednisolone

read about the prophylaxis of Migraine and Cluster


and if any one remember the scenario please write it here

34
215- Pt diabetic he has wound in his leg with poor healing , Exudate
,no sign of inflammation the hyperglycemia cause poor wound
healing by :
a- inhibit phagocytosis
B-stimulate bacterial growth
c-decrease immunity ‫الخيرنسيته‬
216- wt is non hormonal drug use to decrease hot flush
in postmenuposal women:
paroxitine

217- old pt complain of dull hip pain increase after walking and
activity and it make pt wake up from sleep many time and(several
hours) morning stifness whate is the mos DX
a-osteoarthritis
b-osteomlitis
c-osteoprosis
e-depression

218 csf examination show high igG and anbnormal band cell on
agarose gel electrophoresis whate is the DX
a-muscular dystrophy
b- multiple sclerosis

219-senario of pt e GER since 10 years endoscopy done reveal low-


grade dysplasia of lower oesophagus whate is the next step
1-esophageal resection
2-fundoplication
3-rescreening in (i forget the time)
the other choises I don’t remember please read about barrot
esophegous
220- perinatal mortality
A-include all stillbirth after the 20th wk of pregnancy
B- include all neonatal deaths in the firist 8wk of life
c-in clude all stillbirth and firist wk of neonatal deaths
d-is usually death per 10,000 live birth

221- pregnant 41 wk on complete biophysical profile


oligohydraminous is found whate is next step
a-induction of labor

35
222- female came wuth vulval irritation the doctor told her to stop
using bubble bath she stopped it but the irritation continues 0n
examination the vulva skin was waxy like and specked
appearance the dx is
1- psoriasis
2- atopic dermitites
3- contact dermitits
4- lichen planus

223- the mechanism of action of propylthiouracil is


inhibits the enzyme thyroperoxidase

224- pt e hx of erythema and vesicle in the forehead but not affect


the vision whate is the best managment
1- oral acyclovire and F/U
2- oral acyclovire and opthalmologist refere

225 (Picture of a huge ulcer in the leg, the ulcer is red with raised
edges)). Best option of management:
-Topical steroids
- Biopsy
- Radiotherapy
- Topical antibiotics
226- pateint felt fatigue, SOB , angina like pain after doing excercise
there is no thrombus done by cath. , he had aortic valve stenosis with
less than ,7 cm what would u do :
a- avoid exersion
b- aortic valve replacement
c- ttt with medication only
22722y o female there is no breast development , amenorrhea , deep
voice O-E mass over the ovary Dxx:
a-thecoma
b- germ cell tumor
c- lyding cell tumor
d- ovarri storma

228- pt with CHF and atrial fibrillation u add digoxin what is the effect
of it in this case :
a- decrease ventricular effecacy
b- unchanged COP
C- decrease HF

36
229- infant swallow coeeosive material came within half an hour to
ER drooling, crying what is the initial thing to do
activated
charcoal
endoscopy
secure airway
2 cups of
milk 230-svt
ttt:
Digoxin
adenosine
231- malaria in a child:
a- crescent shape gametocyte of vivex is diagnostic in the stool
b- the immediate ttt primquine for 3 d
c- 72h tt t of malaria is suffeceint
d- the most common cause is falciparum
232-scaly purpule lesions in the face of a child the
cause a- staf. Aureus
b- beta haemolytic
srept.coci c- H. influenza
233- child >90% of the normal . < persentile hight with sever bowing of
legs what help u for diagnosis: (same question mentioned but
different choices)
a- lower extremeties x-ray
b- pelvic x-ray
c- cbc
d- alkaline phosphatase
234- ttt PE:
a- iv heparin
b- iv tpa
c- streptokinase
235- The most common side effect of long use of
systemic corticosteroids:
a.Asthma
b.Weakness in pelvic muscles
c osteoporosis

236- pt taking digitalis he developed sudden disturbance in vision


yellow discoloration and light flashes (that’s what I remember from the
question)
a.digitalis toxicity
b.retinal detachment
237- Pt has carotid bruit with occlusion 60% of the left carotid artery
37
what well u advice the pt
a.Asprin daily

38
b.Angiograpy
c. endartectomy

238-) What is the most specific test for syphilis: ‫للتختصارات ارجعوا‬
a. TPI
b. FAAT
treponema antibody absorption test
239-) pt had history of hypertension and no medication taken he eats a
lot of meat with no fruit and vegetables on examination he was obese
BP:130/98 investigations she high cholesterol ,high trigelcride, low HDL
in which category u well put the pt for risk of IHD:
A B C d
I forgot I forgot  High High
cholesterol cholesterol
Sedentary life obese
High BP

240- pt came with PND and orthopnea an examination he has


bilateral basal crepitation and pulmonary edema what is the
diagnosis:
a.left heart failure
b.right heart
failure
241- Likelihood ratio of a disease incidence is 0.3,
mean: 1.large increase
2. small increase
3. no change
4. small decrease
5.large decrease
242- I study done on 10,000 people for about 3 years in the beginning
of the study 3,000 developed the disease and 1,000 on the end of the
study what is the incidence:
a. 10.3
b. 12.5
c. 30

243- the most useful test to detect early pregnancy:


a.urine pregnancy test (my answer) there was no serum BHCG in the
chooses
b.ultrasound
244- Pregnant lady which is hypertensive regarding methyldopa
what well u tell her
39
Methyl dopa better then lisnopril

40
(I couldn’t remember the other chooses)
245- treatment of gonrehea:
Ceftriaxone
246-44 lady has previous history of DVT her husband doesn’t want to use
condom what well u advice her:
a.OCP doesn’t increase the risk.
b.IUD is preferred in this case.
c.she is unlikely to become pregnant
247-Pt covers the tv because he says that they see hem and well
split on his face…… diagnosis:
a.SCZ

248-A man has excessive worry form germs on his hand


a. Specific phobia
b.Agrophopia
c.OCD
249- Scenario about premenstrual dysphoric disorder. (straight
forward and they asked about the diagnosis

250- regarding group Astrept. Infection have lead to rhematic fever


: a.blood dissemination
b.by causing pharyngitis, tonsillitis.
c.joint invasion.
d.affect skin.
e-reach endocardium

251- Child with positive gower sign which is the most diagnostic test
: a.Muscle biopsy

252- child has allergy to dust what well u advice the family
a.keep humidity of the house about…..
b.cover his pillow with….
c.clean his clothe with warm water??
253- child with congenital; heart disease his parents doesn’t know
the name of the disease he has peripheral and central cyanosis:
a.PDA
b.tetrolgy of fallot.
c.VSD
D.left ventricular hypoplasia.

254-
41
Treatment of papillary thyroid cancer: (read about it)
a radioactive iodine uptake scan
b surgery
255- picture of herpes zoster (the same picture)

256-15y boy appear patch in rt lower leg these patch is clear center ,
red in peripheral, no fever no other complain so diagnosis (there was a
picture with lesion in the groin area)
a-c ontact dermatitis
b-tinea corpora
c- lyme disease
d-psiorosis

257- Man is complaining that he doesn’t see the traffic signs well what
is the best way to measure the distance vision:
Snellin chart

258- question about pt had pterygium what well you tell the pt:
-it is malignant
- needs surgery
in another word :regarding ptergium :
🗷 It indicate systemic disease
🗷 Will cause loss of vision
🗷 Treatment is surgery
I forgot the other choices read about the topic
259- Compelete loss of vision Lt eye, in pt with recent infarction?
a) Frontal
b) Parital
c) Cortex
d)
Occipital
42
260- the commonest initial manifestation of increased ICP in
patient after head trauma is
1. Change in level of consciousness
2. ipsilateral pupilary dilatation
3. contralateral pupilary dilatation
4. hemiparesis
5. hypertension
261- Most accurate test for CTS ? carpal tennel syndrome
a) Tinels test
b) Phalens test
c) Nerve tapping numbness
There was no nerve conduction velocity.
262- Hx of Child has itching in his RT hand which increase in the
night described as linear fissures at their top there is blacknish
Scabias
263- Which of the following method is rapid and best for
complete gastric evacuation ? ‫!!!! محدد وقت ماعطاك‬...
a) G lavage
b) Manual induce V
c) Syrupe
d) Active charcoal
264- old male pt with hx of IHD, DM , HTN , dyslipidemia
, family hx of heart disease ,,,,‫كثيره ومصايب‬
lab showed :- LDL : 199 ,,,, HDL : 37
so , in this pt what is most dangerous Risk factor
? a) Increased LDL
b) Decreased HDL
265- Old man psych pt , has halosination , aggressive bebaviour
,loss of memory ,Living without care , urinate on him self , what is
next step to do for him ?
a) Give antipsychotic
B) Admit him at care center for elderly .
266-60 y/o male known to have ( BPH) digital rectal examination
shows soft prostate with multiple nodularity & no hard masses , the
pt request for ( PSA) for screening for prostatic ca what will you do
?
a) Sit with the pt to discuss the cons & rods in PSA test
b) Do trans-rectal US because it is better than PSA in detection
c) Do multiple biopsies for different sites to detect prostatic ca
267- Female com with lump in breast, which one of the following
make you leave him without appointment ?

a) Cystic lesion with serous fluid that not refill again


43
b) Blood on aspiration

44
c) Solid
d) Fibrocystic change on histological examination
268- Infant newly giving cow milk in 9 months old , closed posterior
fontanel, open anterior fontanel with recurrent wheezing and cough ,
sputum examination reveal hemoptesis , x-ray show lung infiltration ,
what is your action ?
a) diet free
milk
b)corticosteroid
c) antibiotics
heiner syndrome (milk induced pulmonary disease in infants) is a food
hypersensitivity pulmonary disease that affects primarily infants.
269- In a certine study they are selecting the 10th family in each
group,ahat is the type of study: imp.
🗷 systemic study
🗷 non randomized study
🗷 stratified study

In statistics, stratified sampling is a method of sampling from a


population.
When populations vary, it is advantageous to sample each
subpopulation (stratum) independently. Stratification is the
process of members of the population into homogeneous
subgroups before sampling.

270- delusions definition:

271- giardiasis treatment:


metronidazole

272- TTT of refractory hiccups?

Chlorpromazine

273- TTT of
miagrine?
Sumatriptan
If BB is
274- most common psychiatric condition come with mania
? paranoid
grandiosity

45
275- patient with fever and fatigue prior to develop maculopapular then
vesicle and pustule ?

46
HSV1
HSV2
Varicella

276- postmenopausal women at high risk of:


osteoporosis

277- why SSRI best TTT?


Effective and tolerable

278- compliance of prophylactive antiasthmatic drugs important to


reduce airway inflammation
reduce esinophil…

279-2 months amnorrhea refuse examination because she is tense and


anxious what will do for her :
FSH and LH
US pelvis
280-60 years old patient has only HTN best drug to start with:
ACEI
ARB
diuretics
beta blocker
alpha blocker

281- structure normally not


palpable? LN

282- COPD pt not responding to bronchiodilator what well u add for hem
: a.aminophiyline
b.methylpredinselone

283- female complain of colourless itching vagina ,her partener


complain of uretheral discharge ,cervical examination shows strawberry
spots treat with:
mecanozole cream
estrogen cream
progesterone cream
douch

47
284- four year old child complain of bleeding from 4 months ,intermittent
painless bowel movement good appetite after examination the
examiner found blood on his digits:
uc
mickels diverticulitis
juvenile polyps

285- high grade fever, rigors ,painful


hepatosplenomegaly: malaria
visceral leshmania
toxoplasmosis
286- appropriate way to prevent spread of
disease : change human behavior
screening tests
287- first few
words:
12 months
18 months
36 months
288- which of the following true about headache :"
-increase ICP at last of day
-normal CT may exclude subarachnoid hemorrhage
-amnursus fugax never come with temporal arteritis .
- neurological sign may exclude migraine
289- patient with typical sign of infections mononucleosis come with
abdominal pain and hypotension next step :
-abdominal CT and IV fluid
-antibiotic and IV fluid and and observation
290- patient with hypersensitivity skin at back take paracetamol and
develop vesicle at back extend to abdomen Dx :
Herpes zoster
291- picture of viral warts

292- pt in burn wll die due to :


-smoke inhalation
- trauma
293- elderly patient bedridden for long time what will you do :
-include family support
-IV valum
294- pt with dysphagia , weakness ,fasciculation …..:
-motor neuron disease
-polyneuropathy
48
295- Young male c/o pleurisy pain at rt side On EX there is only
decrease breath sound
tachypnia other wise normal and there is CXR I don’t know if it is normal
or not But it seems to me normal what will you do?
a-d ischarge pt bez it is only viral
plurzy b-discharge him on Augmentine
C- I think refer him to pulmonologist
296- aseptic meningitis early will found:
a-l ymphocytosis
I can't remember the other choices sorry but you must read about the
CSF analysis in aseptic meningitis and which cells present
297- Kernig's sign:Definition:

298- Diagnosting peritoneal lavage positive


when 1000 RBC
50WBC
‫وخيارات للفسف مافتكرها كلها أرقام‬

299- attributable Risk definition:

300- female with irregular cycle month and absent for two month with
heavy bleeding:
a-m etroohaia
b-menorraghe
c-menometrogia
d-polymenorrhagia
301-8month complaining of gastroenteritis loss of skin truger, sunken
eye depressed anterior fontanel his dehydration is:
10%
20%
5%

302-Patient admitted as a case of emphysema, according to the


vaccine what you will do
a) give pneumococcal vaccine now
b) give flu vaccine now
c) give all vaccine 2week after discharge
d) give flu vaccine now and pneumococcal vaccine 4week after discharge

303-17 years male while play football felt in his knee (turn over ) what
injury

49
medial meniscus lig
lsteral meniscus lig
medial collateral lig
lat collateral lig
anterior crussate lig

304- pt. complain of joint. Stiffness, and high ESR, CRP:

- Inflammatory condition of the soft tissue.


- Immune complex deposite.

305- patient has tangential thought, circumstantial, what is the type of


this condition:

- Form
- content

306- pigmentation of OCP called:

melasma

307- old man did femoral popliteal bypass has 2 days of forgetting

: alzhiemer

vascular alzhiemer

308- clear scenario of turner syndrome (read about features of turner

) 309- cause of death in inflamed burns :


-injuries
-inhalation of smoking

In flame burn , the most common cause of immediate death


1.hypovolemic shoke
2. septic shoke
3. anemia and hypoalbumin
4. smoke inhalation
5. associated injury

310- Atrial fibrillation +narrow complex+unstable pt (hypotensive):


cardioversion

50
311- epdiomology definition:

312- epdimology curve:


graphic registration of disease through a period of time
other choises :
a- Plotting number of cases on time
line b- Geographical places
c- Case with similar diagnosis

313- question in digoxin toxicity (read about):

314- child was playing and felt in the toy, his leg rapped and twisted
he don’t want to walk since yesterday:
- ankle tissue swelling
- spiral tibial fracture 100%
- chip tibial fracture
- femur neck of the tibia freacture
315- which medication increase survival in COPD pt.
: a-b agonist inhaler
b-corticosteroid inh.
c-oral corticosteroid
D- continuous oxygen

316- How much Na in 0.9 normal saline


? a- 30
b- 75
c- 90
d- 155

317-- Female with yeast vaginal discharge the treatment is:


a. Meconazole cream for 7 days
b. Fluconazole orally for one day
c. Metronisazole orally for 7 days

318-- Secondary dysmenorrhea is:


a) rare due to anovulation.
b) due to gonadal agenesis
c) always pathological

51
d) part of sheahan syndrome my answer

319- for with aggressive patient with rheumatoid arthritis:

Methotaxtrate my answer

320- Greatest thing to prevent


disease Genetic consulting
Immunization
Prevent environments ..
Personal behavior??

321- PT WITH POLYCYTHMIA VERA COMPLIN OF GENRALIZED


PRURITTUS AFTER BATHING THE CAUSE IS :

due to abnormal histamine release

322-- unwanted effect of antichlnrgic drugs :

a- Diarrhea

b- Urine incontince

c- Decrease intraocular pressure

d- Blurred vision

323- question about stratified simple study:

324- Rubella Incubation Period


14 to 21 days sure
325- De Quervain Thyroiditis histopathology :
the classic changes of granulomatous thyroiditis develop. This is
characterized by aggregations of lymphocytes, large histiocytes, and
plasma cells among damaged thyroid follicles. Multinucleated giant cells
enclose pools or fragments of colloid, from which stems the designation
giant cell thyroiditis.

326- Nodular sclerosis Hodgkin disease

52
In nodular sclerosis Hodgkin disease (NSHD), which constitutes 60-80%
of all cases of Hodgkin lymphoma, the morphology shows a nodular
pattern. Broad bands of fibrosis divide the node into nodules. The
capsule is thickened. The characteristic cell is the lacunar-type Reed-
Sternberg cell, which has a monolobated or multilobated nucleus, a
small nucleolus, and abundant pale cytoplasm.
327- Thyroid cancer associated with:
Euothyroid
Hyper
Hypo
graves

328- patient with recurrent pneumonia and productive cough , foul


smelling sputum increase with lying down + clubbing:

bronchectasis

BA

Pneumonia

329-) flu like sx since to days and now has red eye ( pic ) Dx:

Viralconjunctivitis / bacterial conjunctivitis / uvitis / glaucoma

330- young pt came to ER with dyspnia and productive tinged


blood frothy sputum , he is known case of rheumatic heart dz , AF
and his cheeks has dusky rash dx :
Mitral stenosis
CHF
endocarditis
331- You r supposed to keep a child NPO he's 25 kgs, how much you
will give for maintenance >> 1600 ml .
First 10 kg X 100ml >> 1000
ml Second 10 kg X 50ml >>
53
500ml

54
Third 5 kg X 20 ml >> 100
ml Total = 1600 ml
332-) old pt take hypercalcemic drugs and developed gout what is
responsible drugs >> frosamide
thiazide

333- In pt with moderately sever acne valgarus best ttt

Oral isotretinoin

topical Retinoids

Topical clindamycin

oral antibiotics

334- which of the following TTT contraindication in asthmatic pt

: Non-selective B blocker

335- case with 60 years old male with RT upper quadrant pain
after dinner , most likely DX gallstone ;
What is most appropriate inx to DX gall stone ?
US
Xray
Barium
336- human bite to hand , most common hand position that proposed
to infection ?
Clenched hand
dependent
extended thump
extended fingers
337- In chlamedia infection ttt is ? ( not mention pregnant or not )
Doxcycline
Azithromycine
Metroniadizole
( also, doxcy used )
338- case cord like cheesy white adherent odour less vagina after use
of antibiotic DX >>Candidiasis

55
339-)- malaria case , beside antiobtic how to prevent ?
Kill the vector

In another way :
What is the most important prevention measures to be taken in the
outbreak of malaria:
a- Clothing disinfected & inspect for insect bite
b- Clothing disinfected & prevent insect bite
c- Eradicate the vector & inspect for insect bite
d-Eradicate the vector & prevent the insect bite

340- Positive predicitive value : Definition


? " pt who has high Risk factor & +ev
test "

341- Most difficult method to prevented in transmission:


Person to person / Vector / Droplet /Air flow

342- old pt, bedridden , with bactermia , organism is


enterococcus fecalis , what the source of infection:
UTI
GIT
pneumonia
bed sores

343-) 4y girl, decrease head growth, decrease social intraction,


decrease in language …etc:
Rett's syndrome

344- case of Raynaud's phenomenon it was direct >> pallor


then cyanotic then red finger without other clinical features .

345- read about rebound hyperglycemia in DM ?? somogi and


down phenomenon

346- During heart contraction,heart receive more blood by:


_ coronary artery dilatation??
_ IVC dilatation
56
_ pulmonary vein constriction
347- Pt. with 1st and 2nd degree burn involving face and neck:
All choices with no hospital admission except one which I choosed as
the
burn involves the face
348- In patient with rheumatoid arthritis:
_ cold app. Over joint is good
_ bed rest is the best
_ exercise will decrease postinflammatory contractures
349- Pt after swimming pool(clear Dx of otaitis externa) Rx:
_ nothing
_ amphotericin B
_ steroid
_ ciprofloxacin drops
350- Patient with continous seizures for 35 min. despite taking 20 mg Iv
diazepam..what to do??
_ give 40 mg IV diazepam
_ give IV phenytoin
_ give IV Phenobarbital
351-16 wk pregnant not known to have illness before has high BP..DX:
_ preeclampsia
_ chronic HTN
_ gestational HTN
35 years prime 16 wk gestation PMH coming for her 1st cheek up she is
excited about her pregnancy no hx of any previous disease.
Her B/P after since rest 160/100 after one wk her B/P is 154/96
Most likely diagnosis :

a- Pre eclempsia
b- Chronic HTN
c- Lable HTN
d- Chronic HPT with superimposed pre eclampsia
e- Transit HPT

352- y/o with mild epigastric pain and nausea for 6


months..endoscopy>lossof rugeal folds, biopsy> infiltration of B
lymphocytes..treated with abx..cause:
_ salmonella

_ H.pylori??

353- Young suddenly develops ear pain, facial dropping..what to do:


50
_ mostly will resolve spontaneously
_ 25% will have permenant paralysis
_ no role of steroids
in another word: male old patient has S&S of facial palsy ( LMNL) ;
which of the following correct about it ;
A- almost most of the cases start to improve in 2ed
weeks b- it need ttt by antibiotic and anti viral
c- contraindicated to give corticosteroid
d- usually about 25 % of the cases has permanent affection

354-2 month infant with white plenched papules in the face what to do:
_ reassurance
_ topical steroids
_ abx
355- Recurrent watery discharge of eye, pain, sensitivity to light..on
exam.> inflammation,ulceration of eye..cause:
_ dust&pollens
_u/v light
_ stress
_ night accommodation
356- Patient with ARDS on ventilation developed pnemothorax..cause:
_ -ve pressure ventilation
_ central line
_ 100% O2
357- Lactational mastitis..Rx:
_ doxycycline
_ ciprofloxacin
_ ceftriaxon
_ gentamyecin
_ cephalexin
358- OCP that causes hyperkalemia:
cant remember the choices.
drospirenone
359- All are 1ry prevention of anemia except:
_ health education about food rish in iron
_ iron fortified food in childhood
_ limitation of cow milk before 12 month of age
_ genetic screening for hereditary
anemia?? 360-+ve leichman test:
ACL injury
361- Waking up from sleep..cant talk, no fever, can cough, normal vocal
cords…Dx:
51
Functional aphonia
362- Patient with CML taking imatinib mesylate and odansetron for
nausea and vomiting presented with tachycardia,fever
Diphoresis and hyperreflexia… Dx:
_ neuroleptic malignant syndrome
_ imatinib toxicity
_ odansetron toxicity
363- What is the most effective method to prevent the brucellosis
infection:
a- Treat the infected people
b- Immunize the farmers & those who deal with the
animals
c- Get rid of all the infected animals
d- Pastralization of the diary products
‫ـــــــــــــــــــــــــــــــــــــــــــــــــــــــــــــــــــــــــــــــــــــــــــــــــــــــــــــــــــــــــــــــــــــــــــــــــــــــــــــ‬
364- PTS with history of infertility the first line of investigation for this
couple is >>>
semen analysis
365- PTS with depression manifestations , what is the mechanism of
the drug that you will prescribe >>>>
increase availability of serotonin

366- women 52 year complaint of hot flush , dry vagina, loss of libido
, loss of concentration , wt gain since hot flush , affect marital state
>>>>
estrogen
Progesterone
fluxatine

367- Old pts with history of bilateral pain and crepitation of both knee
for years now come with acute RT knee swelling , on examination
you find that there is edema over dorsum and tibia of RT leg ,what
is the best investigation for this condition >>>>
Rt limb venogram
368- Standard deviations >>
▪ Measure central tendency or variability
▪ Measure extreme value
▪ Measure validity

369- PTS 18 yrs , you prescribe for him retinoid gel will counsel him for
>>>>>
make your skin sensitive for sun light
52
370- What is the name of questionnaire that differentiate b/w primary
and sleep apnea.???????

371- Post partum female with recurrent attack of hearing loss , which
diagnosed as conductive hearing loss , on CT the is dehesion in the
of semi circular canal diagnosis >>>>
otosclerosis
miner's
Tuberus sclerosis
372- most important investigation to diagnose maxillary sinusitis:
CT
Xray
373- child with bleeding from nose ,history no evidence of clotting
abnormalty ,vitally stable .on eaxamination slow bleeding from posterior
septum whats your action:
do clotting study
spray vasoconstrictive agents.

374- Child with posing head , bowing tibia ,,,, rickets ,,, what is
the deficiency >>>
vit D deficiency.

375-80 year old man complain of sever itching mainly in the wrist and
b/w fingers , with excoriation mark linear and superimposed by
secondary infection disturbing sleep, the pts newly finish 10 days
course of Antibiotics >>>>>>
Monilia
eczema
icythiosis

376- Live guard come to annual examination , no compliant ,


macular dicloration, painless over the face , thers is history for
exposure unproductive to sun rays >>>>>
Sqamous cell carcinoma
377- Community problem of multiple chlymedia infection in the eye ,
best prevention method is >>>>
good water and good sanitation supply

378- Which IS considered abnormal & indicate fetal distress<<<


late deceleration
379- What is the vector for leshmania disease
<<<< sand fly
53
380- Child with URTI then complained from ear pain on
examination there is hyperemia of TM &+ve insufflations test he tri
2 drug no benefit what is the bestTTT>>>
agmentine
azythromycin
ciprofloxacin
steroid
381- Infant in respiratory distress ,hypercapnia , acidosis & have rhinitis
, persistent cough +ve aglutenation test & the doctor treat him by
ribavirin DX>>>
pertusus
RSV
382- Aluminum salt & …… salt will decrease absorption of <<<<
tetracycline
penicillin
383-40 yrs old male com with HX of smoking & alcohol intake for long
time complain of painless ulcer ,role out border on the lateral border of
the tongue DX<<<<<
SCC
lukoplakia
384- Old PTNs with osteoporosis TTT for HTN with diuretic that
prevent Ca loss complain of severe pain in big toe DX>>>>>
thiazide

385- Old male come with CHF & pulmonary edema what is the
best initial therapy>>>>
digoxin
frosamide
debutamine
386- Patient with HX of URTI & flash of light when he sneeze the
cause is>>>>
chemical irritation
mechanical irritation of retina
387- Adult with HTN what is the most common cause
>>>>> renal
essential
388- All of the following exaggerate the gastric ulcer except<<<<
decrease gastric empty time
use of tricyclic antideppresent

389- Sever pain in anatomical snaph box >>>>>


54
scavoid fracture

390- Child come to ER after ingestion of multiple iron tablet of his


relative &iron conc. In blood 700ml???? what is the best
intervention>>>
gastric lavage
charcoal oil
iv defrroxamin
391- Patient with lacremation ,salivation, diarrhea, what
is antidote<<<<
atropine
pralodexam

392- Child with recurrent UTI how to counsel


him>>> increase fluid intake
393- group of patient with lung cancer divided In tow group disease
&control>>>
case control study

394- Gouty arthritis -ve pirfringes crystal what is the


mechanism>>>> deposition of uric acid crystal in synovial fluid due
to over saturation
395- Patient have of urethritis now com with lt knee urethral swap
+ve puss cell but –ve for N.M, chlymedia>>>>
RA
riters disease
gonococcal
396- Pulled Elbow scenario the
TTT>>> immediate reduction
supination
397- Which of the following indicate benign thyroid
lesion>>>> lymphadenitis

398- Young female, k/c of vitligo, came to the clinic with hx of hair loss,
on exam you found an area of4 cm devoid from hair, but the skin is
normal no redness or any abnormality in this area except hair loss,
what does she has:
a- Alopecia
totalis b- Alopecia
areata
399- Young female came to your clinic complaining of skin rash
55
involving the whole body including the palms & soles after unprotected
sexual hx, what is your DX:
Secondary syphilis

56
400- Old man came to you with hx of anal pain, spastic in nature
associated with diaphoresis, tachycardia, which last for a few
min. bothering him more in the night, what does he has:
a- Thrombosis of external hemorrhoid
b- Goy syndrome
c- Proctalgia fugax

401- Female came with hx of sever abdominal pain, vaginal bleeding


for 6 hours, amenorrhea for 8 wk, , O/E tachycardiac, hypertensive,
tense abdomen. what is the most likely site of the ectopic pregnancy:
a- Fallopian
tube b- Ovary
c- Peritoneum
d- Fimbria
402- Young pt was operated 1 year ago for small bowel perforation,
presented with hx of vomiting, constipation, abdominal distention,
colicky pain, what will be the appropriate investigation you will
choose:
a- Barium swallow
b- Barium enema
c- Barium follow through
d- Double contrast study
403- Male came with hx of acute onset of knee pain with swelling,
what will be the most important investigation to do:
a- Arthrocentasis
b- Full CBC
c- Joint US
d- ????
404- Long scenario about child has sore throat & the culture showed
group A strepto, then he develop coca-colored urine, periorbital
edema, and headache what is the single most specific test to diagnose
him with post-strept acute GN:
a- BP more than 95% of normal
b- +ve strepto enzyme
c- Low C3
d- High creatine & urea
(the question is not new but here are the full
choices) 405- Regarding case-control study which is
true:
a- The number of control should be equal to the number of
diseased
b- It is forward study to the risk factor
c- It is backward study to the risk factor
57
406- Young pt came with hx of acute painful swelling of the first
metatarsophalangeal joint, redness, tenderness, fever 38c, what is
the etiology:
a- Staph aureus
b- Sodium urate deposition
c- Pyro phosphate calcium deposition

407- Child with barking cough dx to have croup, what is the


causative organism :
a- Parainflunza

408- Long scenario about 5 year old child otherwise healthy has cough,
fever, chest x ray shows infiltration of the middle & lower Rt lob, after 24
hour of receiving cefotaxime he develop complete Rt lung opasifcation,
what is the most likely causative organism:
a- Staph aureus
b- Strepto, pneumonea
c- Hib
d- Pseudomonas argenosa
In another word :
child pt. came with scenario of chest infection , first day of admission he
treated with cefotaxime , next day , pt state became bad with decrease
perfusion and x-ray show complete rt. Side opcifaction + hydrothorax ,
causative organism :
Strepto. Pnem
Staph. Aureus true if pnumothorax
Hemophilus influenza type b
Pseudomonas
-child presented to ER with SOB on x-ray there is filtration
on mid & lower zone on RT side after
24h of antibiotic pts become cyanosis the x-ray total lung
collapse with medastinal shift what cause?
H-influenza
-pneummocystic carnia
-streptoccouse pneumona

409- Regarding face suture, when should you remove it:


a- After 3-5 days
b- After 7-10 days (my answer)
c- Only absorbable suture should be used
410- Pt with dysmenorria, infertility, not responding to naproxen, what
cauld be the cause:

58
a- Endometritis
b- Endometriosis
411- Pt complain of central lower back pain when he wake up at the
morning, stay for 30 min without medication, with slight improvement
brought by NSAID, investigation shows lumber spinal stenosis, and O/E
we just found Para spinal muscle spasm, otherwise normal, what will
you do:
a- Physical
therapy b- Steroid
injection
c- Surgery
412- Pt had hx of trauma to his nose 2 days ago, he came with hx
of nasal obstruction & pain, O/E he has bilateral swelling (septal
hematoma), what will you do:
a- I &D
b- Antibiotic
413- pregnant woman with past hx of DVT, what will you do for her:
a- warfarin
b- heparin
c- enoxaparin
d- no anticoagulant need
e- aspirin
414- pt post cardiac arrest, poor tissue perfusion, inadequate
ventilation, ABG shows low PH, normal pCO2, low HCO3, what is the
explanation:
a- Resp. acidosis
b- Resp. alkalosis
c- Metabolic acidosis
d- Met. Alkalosis

415- Long scenario about Pt with hyperkalemia 7.5, what will be


your FIRST action:
a- IV calcium gluconate
b-IV HCO3
416- Treatment of recurrent otitis media after multiple Abx course, with
bulging tympanic membrane:
a- Amoxicillin
b- Amoxicillin & clavulanic acid
c- Tazocin
d- IV ????
417- What is the serious sign in pregnancy:
a- Abdominal pain
59
b- Back pain
c- Leg swelling

60
d-H yperacidity
418- Pt came to ER with his hand vist in the mid of his chest, what is
the most likely Dx:
a- Ischemic heart disease
b- Esophageal spasm
c- Pneumonia
419- After head trauma, the pt start to have disinhiption, agitation, and
restlessness what is the affected lob:
a- Prefrontal
area b-
Temporal area
c- Occipital area
d- Parietal area
420- Which of the following suggestive of ovarian cyst rather
than ascites in percussion:
a- Dullness centrally & tympanic peripherally
b- Dullness peripherally &tympanic centrally
c- Dullness all over
d- Fluid wave
e-Decrease bowel motion
i) Shifting dullness

421- Q about sub dermal implantable contraception:


a- Is has low compliance compared to OCP
b- More side effect
c- No local reaction

422- Ptpresented with multiple attacks of palpitation, chest pain,


numbness & tingling of the upper limbs & fearing from dying, no
abnormality detected by physical examination. What is the Dx:
Thyrotoxicosis
Panic attacks
Agoraphobia
Peochromocytoma
a 28 yrs. old lady , C/O: chest pain, breathlessness and feeling that
she'll die soon .. O/E : just slight tachycardia .. otherwise
unremarkable .. the most likely diagnosis is:

a- panic disorder
61
423- which of the following medication is safe during lactation:
Tetracycline
Chloramphenicol
Erythromycin
424- small child presented to ER with hx of sudden onset of groin mass,
painful, associated with vomiting, nausea, O/E groin mass tender,
scrotal swelling with multiple bleeding dots, cremastric reflex is
preserve, what is your dx:
a-Testicular torsion
b- torsion of appendix testis
c-scrotal hematoma
e- incarcerated hernia.
425- what is true about marasmus disease:
a- In contract to kwashorcoir, it affect the low socioeconomic stat
b- It is due to late weaning
c- It leads to growth retardation & wt loss

426- picture of pt with htperpigmentation of the axilla, under


the flourcene wood's light it become pink what is the dx:
a- Fungal
infection b-
Erthrasma
c- Acanthosis nigricans
d- ??????
(picture in kumar)
427- the most common cause of 2dry amenorrhea with high FSH & LH is:
a- Menopause
b- Pituitary adenoma
c- Pregnancy
d- ???????
a- 428-?????
428- female G3P3 post-partum 4wk treated for 3days with antibiotic ( I
forget it) for PID but no response & she still febrial , O/E by PV you
found a 10cm mass in the sac between the vagina & the rectum, tender,
fluctuating what will be your action:
a- Colpotomy
b- Laparotomy
c- Laparoscopy
d- D/C antibiotic & start another one
429- pt with hyperthyroidism ask you regarding the long term
complication of her disease, what will you do:
62
a- Bone density scan

63
b- Brain CT scan
c- ECG
d-Echo
430- pt with hypothyroidism, you start her on the levothyroxine, then
she came for follow up you found her TSH is high, what will you do:
a- Reduce the thyroxin and follow her after 6 months
b- Reduce the thyroxin and follow her after 2 months
c- Continue the same dose & follow up after 2
months
d- Stop the dose till the TSH become normal and follow up after 2
months
431- picture of large cervical lymph node, O/E multiple enlarged
lymph node, matted, non tendr, no skin changes, what is the dx:
a- TB
lymphadenitis b-
Metastatic tumor
c- Lymphoma
d- Infectious mononucleosis
432- pt fall from 10 stairs, he develop nasal swelling, tenderness, skull
X- ray shows non displaced fracture of the nasal bone, what will you do:
a- Refer to the surgeon immediately.
b- Do brain CT
scan c- Reassure
him
433- long scenario about pt with GERD responding partially to PPI,
endoscopy biopsy shows barret esophagus with mild severity, what
will be your action:
a- Esophagus resection.
b- Fundoplication
c- Increase the dose of PPI
434- child came with hx of one attack of tonic-clonic convulsion, & his
mother said that he has multiple attacks of febrile convulsion, what will
give her if her child develop another attack at home:
Phenytoin
Diazepam
Clonazepam
Phenol barb.
435--- 35 year old smoker , on examination shown white patch on
the tongue, management: ‫ حالت‬leucoplakia
a. Antibiotics
b. No ttt
c. Close observation
d- excision biopsy ( may answer ) the choice is not in
64
alqasem question
436-male singer with colon cancer stage B2 ; which of the following
correct ?

65
a- no lymph node
metastases b-one lymph
node metastasis c-2 ===
d-lymph node metastasis + distant metastasis
437--child swallowing battery in the esophagus management :
(the choices are complete)

-bronchoscope
- insert fly catheter
- observation 12hrs
Remove by endoscope
438-young male patient present to ER due to RTA with poly trauma ;
the beast way to maintains airway in responsive poly trauma patient is ;
A-orophargenial airway
b-nasophargenial airway
c-trachastomy
d-endotracheacheal intubations
439- Young patient with congested nose, sinus pressure, tenderness
and green nasal
discharge, has been treated three times with broad spectrum antibiotics
previously, what is your action? (chronic sinusitis)
a) Give antibiotic
b) Nasal corticosteroid
c) Give anti histamine
d) Decongestant
e)observation
440-long case patient with RTA with Blount trauma to abdomen . patient
undergo remove of distal small intestine and proximal colon , patient
come after 6 month with chronic diarrhea , SOB , sign of anemia , CBC
show megaloblastic anemia
What the cause of anemia :
A-folic acid
deficiency b-vit B12
deficency
c-alcohol
441- case infant has genital rash ( the rash spares genital fold ) not
response to antibiotics , most likely Dx;
A-candida albicans
b-napkin dermitis
c-contact dermatitis
d- atobic
dermatitis
e- sebborich dermatitis

66
442--long senior patient came with chest pain , burning in character ,
retrsternal , increase when lying down , increase after eating hot food ,
clinical examination normal DX ‫ مع رايح السيناريو‬GERD
a-MI
b-peptic ulcer
c-GERD
d-

443- about fetal alcohol syndrome (read about)

444- the beast way to ttt pinged induce nervosa ( ‫ يقصد ممكن‬bullima
nervosa )
a-interpersonal psychotherapy
b-cognitive behavior therapy
c-pharmacotherapy
d-
445- old female came with scales aroud the areola ,she took steroid
but no benefit on examination normal and no masses what is your next
step?
A-Antibiotics
B-anti-fungal
C- Mammography
446- patient come with diarrhea , confusion , muscle weakness he
suffer from which ? (hyponatremia)??
A-hypokalemia
B-hyperkalemia
c-hypercalcemia
447- Patient come with jundice , three days after the color of
jundice change to greenish what is the cause?
oxidation of bilirubin

448- Patient with Rhumatoid Arthritis he did an X-Ray for his fingers
and show permanant lesion that may lead to premnant dysfunction ,
what is the underlying process?
substance the secreted by synovial

449- prevention of lyme disease , what is best advice to


parents insect repellents

prevention of Lyme disease :


Treat early disease with doxycycline , Prevent with tick bite avoidance
✓ Light-colored clothing makes the tick more easily visible before
it attaches itself. People should use special care in handling and
67
allowing outdoor pets inside homes because they can bring ticks
into the house.
✓ A more effective, communitywide method of preventing Lyme
disease is to reduce the numbers of primary hosts on which the
deer tick depends, such as rodents, other small mammals, and
deer. Reduction of the deer population may over time help break
the reproductive cycle of the deer ticks and their ability to flourish
in suburban and rural areas.
✓ Backyard patios, decks, and grassy areas that are mowed
regularly are unlikely to have ticks present. This may be because
of the lack of cover for mice from owls and other raptors that prey
on mice. The ticks also need moisture, which these areas do not
provide.
✓ -The areas around ornamental plantings and gardens are more
hospitable for mice and ticks. The highest concentration of ticks is
found in wooded areas.
✓ Individuals should try to prevent ticks from getting onto skin and
crawling to preferred areas.
✓ Long hair should be worn under a hat.
✓ Wearing long-sleeved shirts and tucking long pants into socks
is recommended.
✓ Guineafowl

450--30-40 year male suffer from tenitus , vertigo , sensorconductal


hear loss diagnosis is
Miner's disease

451--patient 20 year old come with palptations ECG show narrow QRS
complexes and pluse is 300 bpm what is the true
Amidarone

452-40 years female complaining of thinking a lot in his children future,


she is alert, anxious, cant sleep properly, poor appetite, she always
make sure that doors in her home are closed, in spite of doors already
closed, provotional Dx:
OCD,
GAD
Schizo

453- Long scenario of restless leg syndrome(he didn’t mention Dx in


scenario), 85 old male many times awake from his sleep bcz leg
pain,
68
this pain relieved by just if he move his foot, but it recure, at rest,…best
management:
Colazpin
Haloperidol
lorazepam,
one drug from dopamine agonist group forgot its name, it’s the right
answer.
Bromocriptin,opomorphine,cobegoline

454- best drug for von willbrand disease is:


fresh frozen
plasma
cryoprecipitate
steroids
(he didn’t mention vasopressin in choices)

455- Best fast management of acute hyperCalcemia is:


Iv fluid
frusamide
dialysis

456- Null hypothesis definition

457- Long scenario of a pt with melanoma in back, he is afraid


of malignat change, which one of the following indicate
malignant melanoma:
>6mm,
irregular and invade the skin,…

458- Best inv to visualize the cystic breast masses


is: MRI
CT
Mammogram,
US
459- Female pt new diagnosed as HTN pt, BMI 28, U sld advise her by:
Body wight reduction ALONE doesn’t benefit her HTN
wt reduction and exercise may benefit him,…
salt restriction well help to reduce the BP

460- Long scenario for pt smokes for 35 y with 2 packets daily, before
3 days develop cough with yellow sputum, since 3 hours became
blood tinged sputum, X ray show opacification and filtration of rt
hemithorax, DX:
69
Bronchogenic CA

70
acute bronchitis
lobar pneumonia
461- Pt with hypercholestrelemia, he should
avoid: Organ meat
Avocado
Chicken
white egg
462-5 y child diagnosed as UTI, best inv to exclude UTI comp:
Kidney US
CT
MCUG
IVU
463-6 y old boy, eat the paper and soil, best initial ttt is:
Fluoxetine
behavioral therapy,….

464-Baby said baba mama, pincer grasp, creeping well, sitting wth
support, estimate age:
6m
7m
8m
10m
465- Drug of choice for a schistosomaisis is:
Praziquanetilo
xaminiquine,
artemether
466-50 YEARS OLD FEMAL HAS HYPERTENSION ,COMPLAINING OF RIGHT
EYE PAIN ,HEADACH,FLUSHES LIGHT ,FLOTERS,SHE DID NOT TAKE HER
MEDICATIN TWO WEEKS BACK THE BLOOD PRESSURE IS 140\90
,ON EXAMINATION NON DAILATED PUPLE REACTIVE ,NO DECREASE EYE
VISION ,THERE IS CUPPING AND SLIGHT ARTERIOVENOUS NIPPING WHAT
IS YUOER MANAGEMENT:
A) REFERAL TO OPTHALMO
B) REASSURANCE AND TELL HER YOUR PROBLEM
BENINGN C)GIVE BEXOLO EYE DROPS
D)TREAT HER AS MIGREN HEADACH

467-55 ys old male pt, presented with just mild hoarsness, on exam,
there was a mid cervical mass, best inv is:

Indirect laryngioscopy??
71
CT brain
CT neck
Biopsy
aspiration

468-4 years old child, was diagnosed as SCD,so many times came to
hospitals with, dyspnia, dactylites , ( he put sign of acute crises ), the
best strategy for prolonged therapy is:
IV hydration fluids with analgesia
follow in Out pt clinic
refer to tertiary haem center.
469- Long scenario for a pt came to ER after RTA, splenic rupture
was clear, accurate sentences describe long term management:
We give pneumococcal vaccine for high risky people just,
we sld give ABs prophylaxis if there Hx of contact even with
vaccination against pneumococcal,
pneumoccal vaccine should not be given at same time with MMR

470- Female pregnant, 32weeks of gestational age, diabetic, and she


has a Hx of full term fetal demise, but her DM now well controlled,
and BPP show no fetal distress,best management:
Wait for SVD
report a CS in 36
weeks Teminate
pregnancy

471- Triad of heart block, uveites and sacroileatis,Dx:


Ankylosing spondylites,
lumbar stenosis,
multiple myeloma

472-72 years old Man with loss of vision in one eye , jaw claudication :
Temporal arteritis .

473- GDM with diagnostic GGT what will u do:


-repeat GTT
-diet
-start with monotherapy
-insulin then change to monotherapy
-insulin test

474- Lt sternal border murmur,ejection systolic, but not radiation to


carotid:
72
73
>AS
> PS .
475- commenst cause of failure to thrive:
psychosocial??
allergy to milk and protein

476- child with umblical hernia:


-sponteneous recovery before school age(: 90% disappear
spontaneously during the first 5 years of life

477- commenst cause of hearing loss in children:


-chronic otitis media
-dysfunction eschian tube
-antenatal causes.
478- child with meningitis symptoms and no nuchal rigidty ,whats the
next diagnostic investigation:
CSF

479- patient given 3 liter 10% dextrose then started to develop confusion
and leg cramps:
hyperkalemia
hypokalemia
hypernatremia

480- table shows only hyponatremia and low plasma osmolality with
normal urine osmolality :
_inappropriate secretion of ADH
-cushing syndrome
-addison
481- german measles cause what:

482- man had gun shot to left lung with decrease breath sound, what
well u do :
-2nd midclavicular needle.
-5th midaxillary needle
-5th midaxillary tube.
-ab.

483- > How did we differentiate between snoring and sleep apnea ? I
think !!!!! Read about Sleep study chart !

> Michegan chart . My answer


74
> Different names of chart -_-"

484- > Pt K/C of crohns , present with hip and Back pain , normal Ab Ex
and hip ROM , what to do next :

> Hip CT

> Ab US

> IVP

> Abdominal CT

> Renal US

485- Patient with around Rt eye pain with tearing and headache , lid
swelling , Dx :

> Migrane with aura

> Tension headache

> Cluster headache .

> Glucoma

486- Recurrent swelling in the natal cleft with skin tract and recurrence ,
Dx :

> Hydrandinitis suuporitiva .

> Frunclosis .

> F..dermatic

487- > Recently diagnosed with DM type II , 32 years old , exercise for 8
weeks and BMI changed from 32 to 31 ..
Labs shown on table ? But no table !!!!!

> Continue exercise . My answer


75
> Start medication
488-74-year old female patient of Cushings syndrome, had hip fracture
falling off stool, what will you screen for while also treating her fracture:
Hyperparathyroidism
Osteomyelitis
Osteoporosis
Osteomalacia
489-Very long scenario about middle age man (50 years) with family
history of heart disease, active lifestyle, on self induced diet with 50%
fat, 35% protein and 15 % carbohydrates, table showing labs, elevated
LDL, low HDL, elevated triglycerides and cholesterol, normal RFTs and all
other labs.
No risk of heart disease
Heart disease risk can be avoided by taking statins
Heart disease can be prevented by decreasing calorie intake
490-18 month old patient, parents were treating baby for flu-like illness
with fever with increased water intake at home, patient developed
generalized tonic-clonic convulsions, presented to you after 1 hour with
(description of post-ictal stage). Now baby becomes fully conscious.
What is the next most appropriate step?
rapid sponging to reduce fever
Give acetaminophen and antipyretics
Treat fever, Get MRI done and treat after result
491-9. Middle aged female patient with history of Stage 2 breast cancer
treated successfully, now presents with moderate to severe pain in left
leg, not relieved by lying down, pain on extension of leg and walking,
O/E Tender region in L3-L4 lower back. No Physical sign of cancer
recurrence. Last saw oncologist 2 years back. What is most appropriate
scenario:
Refer to oncologist
Do DEXA Scan
Do MRI
Hospitalize and do neurology and oncology consultations
492-11. Baby present with weeping, shiny and crusting lesions
around mouth :
Impetigo
493-Male patient was advised to undergo Arterial Graft Bypass surgery
at other clinic after having episode of pain in leg, now is asymptomatic.
Came to you, Non-smoker, elevated cholesterol and early
atherosclerotic plaques on some descending aortal branches. What will
you advise: Undergo Bypass Grafting
Take medication to prevent formation of Arterial plaques

76
To undergo frequent arterial scans to see extent of
disease. 494-1st line class of drugs against Post partum
hemorrhage: Uterine Contractile
Uterine Relaxant
495- Patient recovering from Viral Gastroenteritis, vomiting and
diarrhea abated but still having Anorexia. What will you advise:
Bananas
Rice cereal and apple juice Chopped pears
yougurt and .
Granola, .
496- Patient with idiopathic anovulation. What drug to give:
Clomiphene
Progesteron
LH
FSH
497- Description of PCOS. Mechanism of PCOS:
Androgen Excess
498- Prostitute with multiple sex partners presents with history of
painless vaginal sore which healed and did not leave scar. O/E has
generalized lymphadenopathy. What is your diagnosis:
Syphilis

_
499- Female patient with wide-open eyes, tremors in hands that do
not diminish with intention, What investigation will you do:
1 Pituitary Scan
2 T4 Levels
500- Middle age patient alcoholic with H/O fullness in epigastric region
and mild pain, History of nausea and vomiting. Labs: Increased
Serum Amylase, Diagnosis:
Pancreatic Pseudocyst
Pancreatic Cystadenoma
Choledochal Cyst
Liver Cirrhosis
501- Which one of these patients with pneumonia will you treat
as outdoor patient:
a. 80 Year old with 104 F temperature, BR 24/min PR 126/min, BP 180/110
b. 60 year old with 102 F temperature BR 22/min PR 124/min, BP 160/110
c. 50 year old with 98 F temperature, BR 20/min. HR 110/min, BP 180/110
d. 80 year old with 96 F temperature, BR 18/min, HR 70/min, BP 110/80

http://pda.ahrq.gov/clinic/psi/psicalc.asp
77
according to pneumonia severty index calculator (class IV and V need
hospitalization class III depend on clinical judgment) the high blood
pressure is not involved in calculation.
a-classIII
B-classII
c-class I
D-class
III
502-Long scenario of 28 year old male patient with symptoms of
Ulcerative Colitis+ anemia related to UC. Sigmoidoscopy revelaed
multiple polyps, Biopsy of polyps Carcinoma in situ. What is the most
3. definitive therapy that will be effective in the long-term:
4. Correct Anemia
Left hemicolectomy and Colostomy
5. Total Colectomy and Ilectomy
6. Removal of all polyps by Colonoscopy

503-Female patient came with hypertension, azootemia and GFR of 44.


What is her condition due to:
a. Pheochromocytoma
b.Renal artery stenosis
c.Renal Parenchymal Disease
(pheochromocytoma is excluded from the scenario
renal artery stenosis :1- Significant functional impairment of autoregulation,
leading to a decrease in the GFR, is not likely to be observed until arterial luminal
narrowing exceeds 50%
2- Patients with documented or possible renovascular hypertension may experience
progressive azotemia as a consequence of the renal ischemia and/or the persistence of
significant hypertension.
3- Refractory hypertension (ie, poor control of blood pressure despite treatment with 3 or
more antihypertensive agents) may occur. (emedicine)
renal parenchymal disease: 1- present with HTN.
2-the GFR OF 44>>>means chronic kidney disease stage
3 0)Normalkidneyfunction–
GFRabove90mL/min/1.73m2andnoproteinuria
1)CKD1–GFRabove90mL/min/1.73m2withev
idenceofkidneydamage
2)CKD2(Mild)–
GFRof60to89mL/
min/1.73m2withevidenceofkidneydamage
3)CKD3(Moderate)–GFRof30to59mL/min/1.73m
2
4)CKD4(Severe)–GFRof15to29mL/min/1.73m2
5)CKD5Kidneyfailure-
GFRlessthan15mL/min/1.73m2Somepeoplea
dCKD5Dfor
thosestage5patientsrequiringdialysis;manypat
ientsinCKD5arenotyetondialysis

504- Patient comes with attack of Strep Throat, had history of previous
attack(RF), what is his chance of getting RHD now?
Nothing, he is immune due to previous infection.
72
100%
Needs Immunoglobulin to prevent re-infection.
50% chance of re-infection.

In the United States, rheumatic fever rarely develops before age 3 or after age 40 and is
much less common than in developing countries, probably because antibiotics are widely
used to treat streptococcal infections at an early stage. However, the incidence of
rheumatic fever sometimes rises and falls in a particular area for unknown reasons.
Overcrowded living conditions seem to increase the risk of rheumatic fever, and heredity
seems to play a part. In the United States, a child who has a streptococcal throat infection
but is not treated has only a 0.4 to 3% chance of developing rheumatic fever. About half
of the children who have had rheumatic fever develop it again after another streptococcal
throat infection if it is not treated. Rheumatic fever follows streptococcal infections of the
throat but not those of the skin (impetigo) or other areas of the body. The reasons are not
known.
http://www.merckmanuals.com/home/childrens_health_issues/bacte
rial_infections_in_infants_and_children/rheumatic_fever.html

505-pediatric patient from developing country presented with muscle


wasting, weight loss and absent edema. What is the diagnosis:
1 Marasmus
2 Kwashiorkor
3 Muscle wasting syndrome
Marasmus:1-present of muscle wasting
2- body weight less than 80%of average weight.
3- absence of edema
4- increase prior to age 1
kwashiorkor: 1-presence of
edema 2-increase in >18month
muscle wasting syndrome:1-loss of weight.
2-muscle atrophy.
3-in older pt with chronic disease.
(ref. wikepidia)
The most suitable answer is :marasmus

506-Patient with decreased vision, also peripheral vision decreased,


using tonometer pressure in right eye 24 mm and left eye 22 m. What is
the mechanism:
a.Obstruction in trabecular meshwork and ciliary muscle leads to
pupillary blockage and drainage of aqueous humor.
b.Obstruction at ciliary muscle leads to blockage in drainage of
Aqueous Humor.
In cases where POAG is associated with increased IOP, the cause for the elevated IOP
generally is accepted to be decreased facility of aqueous outflow through the trabecular
meshwork. Occurrence of this increase in resistance to flow has been suggested by multiple
theories
73
http://emedicine.medscape.com/article/1206147-overview#a0104
507-Picture of optic disc. (Looks like this:
http://www.revophth.com/content/d/cover_focus/i/1315/c/25316/)
What does it show?
a. Normal disc
b. Cupping of optic disc
508- Picture of Patients legs (calves) showing maculopapular rash.
H/O red rah appearing on extensor surfaces. Rash is tender to
palpate but does not blanch on pressure. What is the diagnosis:
4 Henoch-Schnolein Purpura
5 Polyarteritis nodusa
Henoch-Schoenlein purpura begins with a symmetrical erythematous macular rash on the
lower extremities that quickly evolves into purpura. The rash may initially be confined to
malleolar skin but usually extends to the dorsal surface of the legs, the buttocks, and the
ulnar side of the arms. Within 12-24 hours, the macules evolve into purpuric lesions that are
dusky red and have a diameter of 0.5-2 cm. The lesions may coalesce into larger plaques
that resemble ecchymoses. Several cases of Henoch-Schoenlein purpura have been
observed after varicella infections.
6 (emedicine)

509- Female patient comes with history of periorbital swelling, itching all
over body, O/E there is lymphadenopathy. Liver and spleen are
enlarged. What is the diagnosis?
a.Urticarial
b.Angioedema
c.Lymphoma??
510- Picture of base of mouth showing a white patch with sharply-
demarcated edges. Patient is male, long- term smoker and chews
tobacco, presents with painless lesion in mouth. What is the next
most important step:
Topical Fluconazole
Biopsy
Wide surgical excision
Diagnosis is :leukoplakia:
1- painless white plaque
2- associated with smoking
3- onthemucousmembranesoftheoralcavity,incl
e
udingthetong ,u ,butalsootherareasof
t
thega ostro-
intestinaltracturinarytrac andthegenitals.
4- Tobacc,eithersmokedorchewed,isconsideredtobet
hemainculpritinitsdevelopment
t
5-5% to25% ofleukoplakiasarepremalignan
lesions;therefore,a leukoplakiasshouldbe
treatedaspremalignantlesionsbydentistsandph
74
ysicians-theyrequirehistologicevaluationor
biopsy(ref.wikepidia)

511- What drug is likely to cause heat-stroke as it inhibits sweating :

75
Orphanedrine
Hyoscamine Sulfate

Hyoscamine sulfate Warnings:

In the presence of high environmental temperature, heat prostration can occur with drug use
(fever and heat stroke due to decreased sweating)

http://www.drugs.com/pro/hyoscyamine-sulfate-elixir.html

other drugs:
Anticholinergics

Cogentin and Artane are examples of anticholinergic drugs. They are both used in the treatment of
Parkison's disease. Medscape.com suggests that anticholinergic medications are involved in the
development of heat stroke. These drugs inhibit the body's sweating mechanism, leading to inadequate
heat elimination.

Thermal Analyzer Leaders In Analytical Instruments Contact Us For Quality


Products. www.Scinco.com
Sponsored Links
Neuroleptics

Neuroleptic drugs are also known as antipsychotic medications. They are used to treat mental health
problems like schizophrenia and bipolar disorder. Some drugs in this class of psychiatric medications are
Haldol, Prolixin and Thorazine. According to "Goldfrank's Toxicologic Emergencies," antipsychotic drugs
inhibit the body's normal response to heat. They reduce the body's ability to increase blood supply to the
skin for the purpose of heat elimination. Due to this, individuals taking medications from this drug class
may be at risk for heat stroke.

Diuretics

Robert Wood Johnson University Hospital reports that dehydrated individuals who cannot cannot sweat
enough to cool their body may experience heat stroke because their internal temperature may rise to
dangerously high levels. Diuretics are drugs that put individuals at risk for heat stroke because they
promote dehydration, according to "Irwin and Rippe's Intensive Care Medicine." They work by making
the body eliminate fluids through urine. Furosemide and hydrochlorothiazide are diuretic medications.

Sympathomimetics

Sympathomimetic drugs, as stated in "Synthesis of Essential Drugs," mimic the actions of the body's
sympathetic nervous system. They increase heart rate, open up the airways and constrict the blood vessels.
Sympathomimetic drugs such as amphetamines, cocaine and ephedrine can lead to heat stroke, because
they constrict the blood vessels and do not permit heat loss through blood vessel dilation. This class of
drugs are also risk factors for heat stroke because they increase the amount of heat produced within the
body

76
Antihypertensives

Antihypertensive medications are used to treat high blood pressure and also put individuals at risk for heat
stroke. Examples of drugs in this class are beta-blockers such as propanolol and calcium channel blockers.
These drugs reduce heart rate, the strength with which the heart contracts and the amount of blood pumped
out to the body. This leads to decreased blood flow to the skin and a reduction in the body's ability to
eliminate heat.

Read more: http://www.livestrong.com/article/118891-medications-predispose-heat-


stroke/#ixzz1jMAxCHRd
http://www.livestrong.com/article/118891-medications-predispose-heat-
stroke/
512- CT of Brain picture. Scenario: Patient with sudden severe occipital
headache came to emergency.
Subarachnoid Hemorrhage
Intracerberal Hemorrhage
Meningitis

Theclassicsymptom s
]
"
ofsubarachnoidhemorrhageit hunderclapheadache(ah
, t
eadache describedas"likebeingkickedinthehead [4

orthe"worstever",developingoversecondsto
minutes).Thisheadacheoftenpulsatestowardstheocci
pu (thebackofthehead).[
(wikepidia)
.
intracerberalhem :Patientswithintraparenchym
.]
albledshavesymptomsthatcorespondtothe
functionscontroledbytheareaofthebrainthatis
.]
damagedbythebled [3
Othersymptoms
s
includethosethatindicateariseini
s ntracranialp
ressureduetoalargemassputingpressureon
thebrain[3
Intracerebralhemorrhagesareoftenmisdiagnos
eda subarachnoid
hemorrhage
duetothesimilarityinsymptomsandsigns.Aseverehead
achefolowedby
vomitingisoneofthemorecommonsymptomsofintracer
ebralhemorrhage.Somepatientsmay
alsogointoacomabeforethebledisnoticed.

513- Patient with severe pain in forehead, over nose and sides of
77
face, also present are shiny blisters on surface of forehead, face and
nose.
What is the diagnosis:
Post-herpetic neuralgia
Varicella
Herpes Simplex
Herpes Zoster

Herpeszosterisaviraldiseasecharacterizedbyapai
nfulskinrashwithblistersinalimitedarea
ononesideofthebody,ofteninastripe.Theinitialinf
ectionwithvaricelazostervirus(VZV)
causestheacute(short-
lived)ilnesschickenpoxwhichgeneralyoccursinc
hildrenandyoung
people.Onceanepisodeofchickenpoxhasresolved,t
hevirusisnoteliminatedfrom thebody

78
butcangoontocauseshingles—
anilnesswithverydifferentsymptoms—
oftenmanyyears
aftertheinitialinfection.Herpeszosterisno
thesamediseaseasherpessimplexdespitethe
namesimilarity(boththevaricelazostervirusandherpe
ssimplexvirusbelongtothesameviral
subfamilyAlphaherpesvirinae).

Varicelazosterviruscanbecomelatentinthenervecel
bodiesandlessfrequentlyinnon-
neuronalsatelitecelsofdorsalrot,cranialnerveorauton
omicganglion,[1]withoutcausing anysymptoms.
[2]
Yearsordecadesafterachickenpoxinfection,thevi
rusmaybreakoutofnerve
celbodiesandtraveldownnerveaxonstocauseviralinfe
ctionoftheskinintheregionofthe
nerve.Thevirusmayspreadfrom
oneormoregangliaalongnervesofanaffectedsegmenta
nd
infecthecorrespondingdermatome(anareaofskinsu
pliedbyonespinalnerve)causinga painfulrash.[3]
[4]
Althoughtherashusualyhealswithintwotofourw
eks,somesufferers
experienceresidualnervepainformonthsoryears,acond
itioncaledpostherpeticneuralgia.

Herpessimplex:1-
blisterscontaininginfectiousparticles.

2- commoninfectionmayaffect:1-
facemouth(orofacial)2-
genitalia(genitalherpes)3-
hands(herapticwhitlow)

3- cancauseherpeskeratitis.

Varicela: isahighlc
y s ontagiou y
ilnesscausedbyprimar
s ) ]
h r
i n f e ct
. i o n withv a ri c e l a zo st e r
[1
s
viru (VZV Itusualystartswitvesicula
Post-herpeticneuralgia:

saneuralgiacausedbythevsaricelazosterviru .Typi
79
f
caly,theneuralgiaisconfinedto r
s erms
ad .
atomicareaoftheskinandfolowsanoutbreak
o herpeszoste (HZ,commonlyknown a shingle
s l
, r

Themostlikelyanswerisherpeszoster.

514- Patient had fly in his eye. On removal of the foreign object what will
he need:
7 Topical corticosteroids
8 Topical Antibiotics
9 Oral corticosteroids
10Oral Antibiotics
11Answered based on the sle studing group couldn’t find a ref.

80
515- Patient complains of discomfort in the eye. There is no
discharge. O/E with dye, a dendritic shaped ulcer is seen on the
surface of the cornea. What is the diagnosis:
Keratitis
Uveitis
Acornea
l ulce,o ulce
r r s r a t i v e k e ra t i t i , o
eyesoreisaninflammatoryormoreseriously,
s l
infectiveconditionofthec
l orneainvolvingdisrup
tionofitepithelia layerwithinvolvementofthe
corneastroma.
(Wikepidia)

516- A patient complains of 2  day history of stuck together lashes on


waking up. There is muco- purulent discharge. Anterior Chamber, uvea
and iris are clear. What is the diagnosis?
a. Bacterial Infection
b. Viral Infection
c. Allergy

Bacterial conjunctivitis is usually a benign self-limiting illness,1 although it can sometimes be


serious or signify a severe underlying systemic disease. Occasionally, significant ocular and
systemic morbidity may result.2

Epidemiology
 This is one of the most common ocular problems seen in the community.3
 In adults, bacterial conjunctivitis is less common than viral conjunctivitis; although
estimates vary widely, it is thought to account for no more than half of all cases of
acute infective conjunctivitis.4
 It is most commonly caused by Staphylococcus spp., Streptococcus
pneumoniae, Haemophilus influenzae, and Moraxella catarrhalis.5
 In children, bacterial conjunctivitis is more common than viral and is mainly caused
by H. influenzae,S. pneumoniae and M. catarrhalis.
Presentation
History

Relevant aspects of the history include:

 Nature of the problem:


o Discomfort - burning or gritty but not sharp.
o Pain - should be minimal; significant pain suggests a more serious diagnosis.
o Vision - usually normal, although 'smearing', particularly on waking, is common.
o Discharge - this tends to be thick rather than watery.
o Associated symptoms such as photophobia which should be absent or, at
most, mild. Significant photophobia suggests severe adenoviral conjunctivitis or
some degree of corneal involvement.
 Contact lens wear: could this be (or lead to) a problem of the (vulnerable) cornea?
 Time course: onset, duration - where 81 this is chronic, you may have to consider
venereal disease in people at a sexually active age.
 Use of over-the-counter medication: could this be a reaction to previously administered
drops or ointment?
 Social aspect - has anybody else had it (family, school, work?) and are there
issues about staying at home during the course of the illness?
Findings
 'Red eye' with uniform engorgement of all the conjunctival blood vessels.
 Bacterial conjunctivitis may often be distinguished from other types of conjunctivitis by
the presence of a yellow-white mucopurulent discharge. Eyes may be difficult to open in
the morning, glued together by discharge.
 There is also usually a papillary reaction (small bumps on the palpebral conjunctiva,
appearing like a fine velvety surface). The presence of follicles is more likely to indicate
viral conjunctivitis.
 Bacterial conjunctivitis is usually bilateral (but often sequential).3
 Check visual acuity - this should be normal, other than the mild and temporary blur
secondary to the discharge which can be blinked or wiped away.
Is this bacterial?

It is not always easy to determine whether the patient's simple, acute conjunctivitis is
bacterial or not but this is important as it may determine the subsequent management plan.
Ultimately, swabbing the eye provides the most accurate diagnostic answer but it is clearly
not practical to do this for every patient. A study has shown that, in adult patients, there is a
significant chance that the infection is bacterial when there is a combination of: 4

 A positive previous history of infectious conjunctivitis.


 An itch present.
 A mucopurulent discharge ('glue eye').

However, in severe, resistant, atypical cases or in immunosuppressed patients, swabbing for


culture and sensitivities is important.3
When patients describe their eyes glued together in the morning, this doesn't necessarily mean
that there is a purulent discharge. Viral and allergic conjunctivitis often result in lids that are
matted shut in the morning with mucopurulent material. However, these patients actually
have crusting of the lashes due to drying of tears and serous secretions, not the wet, sticky,
mucopurulent matting characteristic of bacterial conjunctivitis.

http://www.patient.co.uk/doctor/Bacterial-Conjunctivitis.htm

_
517-Patient presents with red, peeling rash at back of ears, on limbs
and over body. What is the first line treatment?
Topical
steroid Oral
Steroid Oral
Antibiotic

The diagnosis:scarlet fever :


82
83
The cutaneous rash, , lasts for 4-5 days, followed by fine desquamation, one of the most
distinctive features of scarlet fever. The desquamation phase begins 7-10 days after resolution
of the rash, with flakes peeling from the face. Peeling from the palms and around the fingers
occurs about a week later and can last up to a month or longer. The extent and duration of this
phase are directly related to the severity of the eruption.
Antibiotic therapy is the treatment of choice for scarlet fever.

Penicillin remains the drug of choice (documented cases of penicillin-resistant group A


streptococcal infections still do not exist). A first-generation cephalosporin may be an
effective alternative, as long as the patient does not have any documented anaphylactic
reactions to penicillin. If this is the case, erythromycin may be considered as an alternative.

??518- 50-year old accountant, sedentary lifestyle, BMI 30, takes


irregular meals; arteries show signs of early atherosclerotic changes.
What will you advise?
12No meds necessary
13Prescribe diet of 600 kcal/day and reevaluate in 4 months
14Prescribe over weight diet and reevaluate in 6 months
??519-Mechanism of Action of drugs that inhibit Conversion of estriol to
estrogen. (I forgot the exact question but it mentioned about ovulation
and who inhibits conversion of esterone to estrogen?) Options were:
a. Aromatase inhibitors??
520-What drug reverses the effect of Benzodiazepines:
a. Flumazanil
Flumazen
i r t
l
(Anexate)isacompetitivebenzodiazepinerecept
o antagonis thatcanbeused
521- pateint C/o ictrus in skin and eye on investigation WBC 2500
plt 70,000 HG 7 lekocytosis 17% total bilirubin 51 and direct bilrubin 12
what is the test most likly positve
+ve coomb test
In us obstructive billiary duct
antiparietal cells antibodies

522-Pts have history of cervical incompetence pregnant at 8w what the


management?
circulage at 14-16w

523--child presented with anemia he have family history of thalassemia what


the most diagnostic test?
-measuring of HB A2
-bone marrow
-serum feriten

84
524-Fracture of elbow common injury of ?
ial

-_pts presented with pruritis hepatosplenomegaly lef supraclavicular LN ?


Cholinergic pruratis
Lymphoma
526- the antipsychotic drug have less pyramidal side effect is ?
There was significant optimism when they were first developed and it was thought
that they represented a breakthrough in the treatment of schizophrenia due to
having less extra-pyramidal side effects at therapeutic doses. The extra-pyramidal
side effects has been the one significant set of side effect that has led to poor
compliance with antipscyhotic medication.
he common atypical antispychotic drugs include risperidone, olanzapine, quetipaine,
aripiprazole, zyprasidone, clozapine and amisulpiride.
http://www.understand-schizophrenia.com/atypical-antipsychotics.html

527-_old age presented with vesicular rash on thoracic tell the midline
?
Herpes zoster

528--female came with her baby with history of fatigue palpitation and tremor
due to?
Postpartum anemia
Hashimotos thyroiditis

a very short scenario)Female come to the clinic with her baby of 6


month , she had tremor and other sign I forgot it, which of the following
is most likely dx
... Hashimoto
Postpartum thyroiditis

hypertyrodism,
sub acute tyroditis
hypothriodism
]
Postpartum
thyroiditisisaphenomenonobservedfolowingp
, regnanc
[1
y andmay involvehyperthyry
oidism hypothyroiidism
orthetwosequentialy.Itaffectsabout5%
s d
ofalwomen
withinayearaftergivingbirth.Thefirstphaseistypical
hyperthyroidism.Then,thethyro either
returnstonormalorawomandevelop hypothyroidism.
85
Theinitialphaseofhyperthyroidsymptomsoccurstran
2
sientlyabou
] twotosixmonths , s
postpartum[ .Typicalsymptomsincludefatigue,irritab
ility,nervousness palpitation ,andheat

86
intolerance.Hormonaldisturbancesduringthisphaset
endtooccurwithlowerintensity
comparedwiththehypothyroidphase[2

529-antidote of acetaminophen:
N-acetylcysteine.
??530-_pts with hypertension and cp of aortic dissection what the
management ?

531-wich true about hepatoma?(read about it)


Is common in female
Comes with Chronic liver disease
http://emedicine.medscape.com/article/197319-overview

532-The most powerful epidemiologic study is:


d. retrospective case control study
e. cohort study
f. cross-sectional study
g. historic time data
h. secondary data analysis

533- Evidence base medicine:


practice medicine as in the book
practice according to the department policy
practice according to available scientific evidence
practice according to facility
practice according to latest publish data
534- Pt had fever in the morning after he went through a surgery (I
couldn’t remember the type of surgery). What’s your diagnosis:
Atelectasis if 0-2 days)
Wound infection
DVT
UTI

Post-operative fever
 Days 0 to 2:
o Mild fever (T <38 °C) (Common)
o Tissue damage and necrosis at operation site
o Haematoma
o Persistent fever (T >38 °C)
o Atelectasis: the collapsed lung may become secondarily infected
o Specific infections related to the surgery, e.g. biliary infection post
biliary surgery, UTI post-urological surgery
87
o Blood transfusion or drug reaction

88
 Days 3-5:
o Bronchopneumonia
o Sepsis
o Wound infection
o Drip site infection or phlebitis
o Abscess formation, e.g. subphrenic or pelvic, depending on the surgery involved
o DVT
 After 5 days:
o Specific complications related to surgery, e.g. bowel anastomosis
breakdown, fistula formation
o After the first week
o Wound infection
o Distant sites of infection, e.g. UTI
o DVT, pulmonary embolus (PE)

535-23 yrs old female has mobile breast lump in the upper outer
quadrant of the left breast. Size= 2 cm and doesn’t change with
menstrual cycle. What’s the most likely diagnosis:
Fibroadenoma

1-the most common benign breast mass in


adolescent 2-easy to move with well defined edges
3- often in upper outer quadrent
4- not affected with menstrual cycle ,the affected with menstrual cycle
is cystic breast changes

536- Breast feeding in the full term neonate:


a. Increase URTI rate
b. No need for vitamin supplementation
c. Food introduce at 3 months
d. Increase GE rate
537- Which of the following organisms can cause invasion of
the intestinal mucosa, regional lymph node and bacteremia:
a. Salmonella??
b. Shigella
c. E. coli
d. Vibrio cholera
e. Campylobacter jejeni
538- Pt has saddle nose deformity, complaining of SOB,
hemoptysis and hematiuria. The most likely diagnosis is:
Wagner’s granulomatosis

http://en.wikipedia.org/wiki/Wegener's_granulomatosis

539- OCP:

89
a. Changes the cervical mucus
b. increase pre menstrual tension
c. Have a failure rate of 3 %

b. Combinedoralcontraceptivepilsweredevelopedto
preventovulationbysu pressingtherelease
ofgonadotropins.Combinedhormonalcontraceptives
,includingCOCPs,inhibitfolicular
developmentandpreventovulationastheirprima
rymechanism ofaction.[5][24][71][72][73]

c. Progestogennegativefedbackdecreasesthepulsef
requencyofgonadotropin-releasing
hormone(GnRH)releasebythehypothalamus,whi
chdecreasesthereleaseo folicle-stimulating
hormone(FSH)andgreatlydecreasesthereleaseofl
uteinizinghormone(LH)bytheanterior
pituitary.DecreasedlevelsofFSHinhibitfolicula
rdevelopment,preventinganincrease
inestradio levels.Progestogennegativefedback
andthelackofestrogenpositivefedbackon
LHreleasepreventamid-cycleLHsurge.Inhibitiono
foliculardevelopmentandtheabsenceofa
LHsurgepreventovulation.[5][24][71]

d. Estrogenwasoriginalyincludedinoralcontracepti
vesforbetercyclecontrol(tostabilizethe
endometrium
andtherebyreducetheincidenceofbreakthroughbl
eding),butwasalsofoundto
inhibitfoliculardevelopmentandhelppreventov
ulation.Estrogennegativefedbackonthe
anteriorpituitarygreatlydecreasesthereleaseo
fFSH,whichinhibitsfoliculardevelopmentand
helpspreventovulation.[5][24][71]

e. Asecondarymechanism
ofactionofalprogestogen-
containingcontraceptivesisinhibition ofsperm
penetrationthroughthecervixintotheu
pergenitaltract(uterusandfalopiantubes)by
decreasingtheamountofandincreasingtheviscosi
tyofthecervicalmucus.[73]

(wikipedia)
540- Pt has 2 cm dome shaped mass in the dorsum of his hand. It’s covered
by keratin. What’s the most likely diagnosis:
Basal cell carcinoma
Malignant melanoma
90
Keratoacnathoma
KAisthatitisdome-shaped,symmetrical,suroundedbyasm
othwalofinflamedskin,andca pedwithkeratin scalesanddebris.
(Wikipedia)

541- Pt has hemorrhagic lesion in the mouth and papules in the face
and back. He had SOB, fever, cough and mediastinal mass. What’s the
diagnosis:
a. Kaposi sarcoma

b. KSlesionsarenodulesorblotchesthatmayb
ered,purple,brown,orblack,andare
usualypapular(i.e.,palpableo raised).

c. Theyaretypicalyfoundontheskin,butspreadels
ewhereiscommon,especialythe
mouth,gastrointestinaltractandrespiratorytra
ct.Growthcanrangefrom veryslowtoexplosively
fast,andisassociatedwithsignificantmortalityand
morbidity.[16]

91
d.[edit]Skin

e.
Commonlyaffectedareasincludethelowerlimbs,ba
ck,face,mouth,andgenitalia.Thelesions
areusualyasdescribedabove,butmayoccasion
alybeplaque-like(oftenonthesolesofthe
fet)oreveninvolvedinskinbreakdownwithresulti
ngfungatinglesions.Associatedswelingmay
befrom
eitherlocalinflammationorlymphoedema(obstruct
ionoflocalymphaticvesselsbythe
lesion).Skinlesionsmaybequitedisfiguringfort
hesufferer,andacauseofmuchpsychosocial
pathology.

f. [edit]Mouth

g.

h. IntraoralAIDS-
a sociatedKaposisarcomawithanoverlyingcandidiasisinfe

i.
Isinvolvedinabout30%,andistheinitialsitein1
5% ofAIDS-relatedKS.Inthemouth,thehard
palateismostfrequentlyaffected,folowedbythegu
ms.[17]Lesionsinthemouthmaybeeasily
damagedbychewingandbledorsuffersecondaryinfec
tion,andeveninterferewitheatingor speaking.

j. [edit]Gastrointestinaltract

k. Involvementcanbecommoninthosewithtransplant-
relatedorAIDS-relatedKS,anditmayoccur
intheabsenceofskininvolvement.Thegastrointesti
nalesionsmaybesilentorcauseweight
loss,pain,nausea/vomiting,diarhea,bleding(eitherv
omitingblodorpassingitwithbowel
motions),malabsorption,orintestinalobstruction.[18]

l. [edit]Respiratorytract

m.Involvementoftheairwaycanpresentwithshortn
essof
breath,fever,cough,hemoptysis(coughingupblo
d),orchestpain,orasanincidentalfinding
onchestx-ray.
[19]
Thediagnosisisusualyconfirmedbybronchosco

92
pywhenthelesionsare
directlysen,andoftenbiopsied

(wikipedia)
542- In the neck, esophagus is:
Posterior to the trachea
Anterior to the trachea
Posterior to vertebral column
543- High risk factor in CLL :
Age
Smoking
History of breast ca
History of radiation
Factorsthatmayincreasetheriskofchroniclymphocyticleukemi
ainclude:

93
 Yourage.Mostpeoplediagnosedwithchroniclymphocyticleuk
emiaareover60.

 Yoursex.Menaremorelikelythanarewomentodevelopchronic
lymphocyticleukemia.

 Yourrace.Whitesaremorelikelytodevelopchroniclymphocyt
icleukemiathanarepeopleofothe races.


Familyhistoryofblodandbonemarrowcancers.Afamilyhist
oryofchroniclymphocyticleukemiaor
otherblodandbonemarowcancersmayincreaseyou risk.


Exposuretochemicals.Certainherbicidesandinsecticid
es,includingAgentOrangeusedduringthe Vietnam
War,haveb
enlinkedtoanincreasedriskofchroniclymphocyticleukemia.

http://www.mayoclinic.com/health/chronic-lymphocytic-
leukemia/DS00565/DSECTION=risk-factors
There are no other proven risk factors for CLL. The risk of getting CLL does not seem to be affected by smoking, diet, exposure
to radiation, or infections.
http://www.cancer.org/Cancer/Leukemia-
ChronicLymphocyticCLL/DetailedGuide/leukemia-chronic-lymphocytic-risk-
factors
544- Which of the following medications is considered as HMG-CoA
reductase inhibitor:
Simvastatin
Fibrate

Alstatinsactbyinhibiting3-hydroxy-3-
methylglutarylcoenzymeAHMG-CoAreductase,
y y
therate-limiting fenzymeoftheHMG-
CoAreductasepathwa ,themetabolicpathwa

_
545- Burn involved 3 layers of the skin
called: Partial thickness
Full thickness(entire dermis)
Superficial
Deep

546- Cherry red skin found in:


a. Polycythema
b. CO poisoning
94
http://www.thelancet.com/journals/lancet/article/PIIS0140-6736(05)79807-
X/fulltext

547- Most serious symptom of CO poisoning is:


Hypotension
Arrhythmia
Cyanosis

95
Seizure
548- Patient with subconjuctival hemorrhage. What you will do for him:
Reassurance sure 100%
Send him to the ophthalmologist
549- Patient with renal transplant, he developed rejection one week
post transplantation, what could be the initial presentation of rejection:
Hypercoagulability
Increase urine out put
Fever
Anemia
Signs and Symptoms of Kidney Rejection
Fever over 38°C or 100.4°F
Decreased urine output
Weight gain over 2 pounds per day
Increased blood pressure
Pain over kidney
http://www.uihealthcare.com/topics/medicaldepartments/surgery/rejection/in
dex.html
550- Pregnant lady in her 30 wks gestation diagnosed as having swine
flu. She has high grade fever and cough for 4 days and her RR= 25/min.
what will you do for her:
Give her Tamiflu 75 mg BID for 5
days Refer her to ER for admission
Give her antibiotics
Refer her to OBGY doctor
551- Female patient has morning stiffness and pain involving the
metacarpophalengeal and proximal interphalengeal joints. What’s the likely
diagnosis:
Rheumatoid arthritis
552- DPT vaccine shouldn’t given if the child has:
a. Coryza
b. Diarrhea
c. Unusual cry
d. Fever = 38
553-A female patient has clubbing, jaundice and pruritis. Lab results showed
elevated liver enzymes (Alkaline phosphatase), high bilirubin, hyperlipidemia
and positive antimitochondorial antibodies. What’s the most likely
diagnosis:
Primary sclerosing cholangitis
Primary biliary cirrhosis

_
554- Shoulder pain most commonly due to:
Infraspinatus muscle injury
Referred pain due to cardiac ischemia
96
In acute cholecystitis

_
555- Female patient with fatigue, muscle weakness, parasthesia in the
lower limbs and unsteady gait. Do:
a. Folate level
b. vitamin B12 level
c. Ferritin level

_
556- Patient developed lightheadedness and SOB after bee sting. You
should treat him with the following:
d. Epinephrine injection, antihistamine and IV fluid
e. Antihistamine alone
557- Patient is 74 yrs old female complaining of pain and stiffness in the
hip and shoulder girdle muscles. She is also experiencing low grade fever
and has depression. O/E: no muscle weakness detected (Polymyalgia
rheumatic). Investigation of choice:
RF
Muscle CK
ESR

_
558- 2 yrs old boy with coryza, cough and red eyes with watery discharge
(a case of measles). Most likely diagnosis of the red eyes is:
Conjunctivitis
Blepharitis
559- foot ball player gt hurt by the football to the knee , on examnation
+ve valgus stress test, -ve macmerry and lachman test . he has:
1) tibial fracture
2) lateral meniscus tear
3) medial meniscus tear
4) lateral collateral ligament tear
5) medial collateral ligament tear

. A foot ball player his knee was hit from the left lateral side, valgus
test was positive; drawer test & Laschman test were negative. Which
of the following was injured?
a) Anterior cruciate ligament
b) Posterior cruciate ligament
c) Medial collateral ligament
d) Lateral collateral ligament
e) Medial meniscus

97
_
560- Patient has bilateral abdominal masses with hematuria. Most likely
diagnosis is:
a. Hypernephroma
b. Polycyctic kidney disease

Polycyctic kidney disease: Pain— in the abdomen, flank, or back— is the most common initial complaint, and it is
almost universally present in patients with autosomal dominant polycystic kidney disease (ADPKD). The pain can
be caused by any of the following:

 Enlargement of one or more cysts


 Bleeding, which may be confined inside the cyst, or lead to gross hematuria with
passage of clots or a perinephric hematoma
 Urinary tract infection (eg, acute pyelonephritis, infected cysts, perinephric abscess)
 Nephrolithiasis and renal colic
 Rarely, a coincidental hypernephroma

In addition, patients with ADPKD may have abdominal pain related to definitively or presumably associated
conditions. Dull aching and an uncomfortable sensation of heaviness may result from a large polycystic liver.
Rarely, hepatic cysts may become infected, especially after renal transplantation.

Abdominal pain can also result from diverticulitis, which has been reported to occur in 80% of patients with ADPKD
maintained on dialysis, probably from altered connective tissue. However, this rate has not been demonstrated to
be higher than the rate among other patients on dialysis.

Patients with ADPKD may be at a higher risk of developing thoracic aortic aneurysms. Abdominal aortic aneurysms
are not increased among these patients.

Pain may also develop for reasons completely unrelated to the underlying disease; thus, abdominal pain in patients
with ADPKD may be a diagnostic challenge.

Hematuria

Hematuria frequently is the presenting manifestation and usually is self-limited, lasting 1 week or less. Polycystic
kidneys are unusually susceptible to traumatic injury, with hemorrhage occurring in approximately 60% of
individuals. Mild trauma can lead to intrarenal hemorrhage or bleeding into the retroperitoneal space accompanied
by intense pain that often requires narcotics for relief.

Hypernephroma(renalcelcarcinoma)widerangeofsym
:
ptomscanbepresentwithrenal .]
s
carcinomadependingonwhichareasofthebodyhaveb
enaffected[3 Theclassictriad
ihematuria(blodintheurine),flankpainandanabdominal
mass.
flank mass is uncommon (10%) and is indicative of advanced disease.
And I think the disese is unilateraral
The best choise is polycystic kidney

98
99
561- Male patient working in the cotton field, presented with 3 wks Hx of
cough. CXR showed bilateral hilar lymphadenopathy and biopsy (by
bronchoscopy) showed non-caseating granuloma. What’s your diagnosis:
Sarcoidosis
Amylidosis
Histiocustosis
Byssinosis
Pneumoconiosis
" r
Bysinosi,alsocaled"brownlungdisease"orM
s on
dayfeve",isanoccupationa lung
diseasecausedbyexposuretocotondustininadequ

_
562- Pt presented with severe epigastric pain radiating to the back. He has
past hx of repeated epigastric pain. Social hx: drinking alcohol. What’s the
most likely diagnosis:
MI
Perforated chronic peptic ulcer
If pancerititis come in choises I don’t know if it is better answer_

563- Erosive gastritisdon’t know


Happened within one week of injury
Happened within 24 hrs of injury !!!

564- In brainstem damage:


a. Absent spontaneous eye movement
b. Increase PaCO2
c. Unequal pupils
d. Presence of motor movement

Commonly, brain stem damage causes a loss of consciousness. It may be temporary or more extended.
People with severe brain stem damage can enter comas and persistent vegetative states with limited
probability of waking up again. Other people may be conscious and aware, but could have severe breathing
problems, abnormal heart rates, or balance disorders. More mild injuries may result in a staggering gait and
sensory impairments associated with interruptions to sensory signals.

Brain stem consisting of the midbrain, pons, and medulla, which extends downwards to become the spinal cord
Controls respiration and various basic reflexes (e.g., swallow and gag)
Absent pupillary light reflex
Corneal reflexes are absent
PaCO2 levels greater than 60 mmHg, ≥20 mmHg over baseline

565- fluxtin half life =

1-4 days

100
566-35 yrs old male has SOB, orthopnea, PND, nocturia and lower limbs
edema. What’s the most common cause of this condition in this patient:
a. Valvular heart diease
b. UTI
c. Coronary artery disease
d. Chronic HTN
Coronaryarterydiseaseandheartatack
.
Coronaryarterydiseaseisthemostcommonform of
http://www.mayoclinic.com/health/heart-
heartdiseaseandthemostcommoncauseofheartfail
failure/DS00061/DSECTION=causes
567-Female patient had carpopedal spasm after measuring her BP. This is
caused by:
Hypocalcemia

Causes of Carpopedal spasm:

The following medical conditions are some of the possible causes of Carpopedal spasm. There are likely to be other
possible causes, so ask your doctor about your symptoms.

 Hypocalcemia

 Multiple blood transfusions

 Parathyroidectomy

 Ileostomy formation

 Gastric resection with gastrojejunostomy

 Hyperventilation

 Tetany

 Following thyroid surgery due to damage to the parathyroid glands

568- Patient known case of DM type 2 on insulin, his blood sugar


measurement as following: morning= 285 mg/dl, at 3 pm= 165 mg/dl, at
dinner time= 95 mg/dl. What will be your management:
Increase evening dose of long acting insulin
Decrease evening dose of short acting insulin
Decrease evening dose of long acting insulin
Increase evening dose of short acting insulin
569-Diabetic patient on insulin and metformin, has renal impairment. What’s
your next step:
101
Stop metformin and add ACE inhibitor
DM HTN patient with MI receiving metformin and diltiazem and other
medication his creatine clearance is high .. you will do:
a) add ACE inhbitor
b) remove metformin (contraindicated in renal failure)
c) continue same medication

570-4 weeks old male child with acute onset forceful non-billious vomiting
after feeding. He is the first child in the family. He is gaining normal wt and
looks hungry. What’s your diagnosis:
Pyloric stenosis

571- 1 week old infant presented with repeated forceful vomiting. What’s the
diagnosis ???(the same history of pyloric stenosis but the age is 1 wk):
Pyloric stenosis
Duodenal atresia
Volvolus
Hirschprung
572-55 y/omale,, c/o angina and syncope on exertion ,, normal ejection
fraction ,, normal coronary arteries ,, there is only calcified aortic valve
with total area < .75 cm ,, the rest of examination and investigations are
normal ..
Wt is ur manag. :
Avoid exertion
Medical therapy
Aortic ballon dilation
Aortoc valve replacrment

The only definitive treatment for aortic stenosis is aortic valve replacement. The development of symptoms due to
aortic stenosis provides a clear indication for replacement. For patients who are not candidates for aortic
replacement, percutaneous aortic balloon valvuloplasty may provide some symptom relief.[4]

Medical treatment (such as diuretic therapy) in aortic stenosis may provide temporary symptom relief but is
generally not effective long term.

In truly asymptomatic patients with severe aortic stenosis, the issue of valve replacement is less clear.

The recommendations of the ACC/AHA 2006 valvular heart disease guidelines for aortic valve replacement in
patients with valvular aortic stenosis are summarized below, in Table 5.[10] In most adults with symptomatic, severe
aortic stenosis, aortic valve replacement is the surgical treatment of choice. If concomitant coronary disease is
present, aortic valve replacement and coronary artery bypass graft (CABG) should be performed simultaneously.

Table 5. Recommendations for Aortic Valve Replacement in Aortic Stenosis(Open Table in a new window)

102
Indication Clas
s

Symptomatic patients with severe aortic stenosis I

Patients with severe aortic stenosis undergoing coronary artery bypass I


surgery

Patients with severe aortic stenosis undergoing surgery on the aorta or other I
heart valves

Patients with severe aortic stenosis and LV systolic dysfunction (ejection I


fraction < 0.50)

Patients with moderate aortic stenosis undergoing coronary artery bypass IIa
surgery or surgery on the aorta or other heart valves

Patients with mild aortic stenosis undergoing coronary artery bypass IIb
surgery when there is evidence that progression may be rapid, such as
moderate-to- severe valve calcification

Asymptomatic patients with severe aortic stenosis and abnormal response


IIb
to exercise (eg, hypotension)

Asymptomatic patients with severe aortic stenosis and a high likelihood of


IIb
rapid progression (based on age, calcification, and coronary artery disease)
or if surgery might be delayed at the time of symptom onset

Asymptomatic patients with extremely severe aortic stenosis (valve area less
IIb
than 0.6 cm2, mean gradient greater than 60 mm Hg, and jet velocity greater
than 5 m per second) if the patient’s expected operative mortality is 1% or
less

AVR is not useful for prevention of sudden death in asymptomatic patients


III
with none of the findings listed under asymptomatic patients with severe
aortic stenosis

emedicine
573-Patient with untreated bronchogenic carcinoma has dilated neck veins,
facial flushing, hoarsness and dysphagia (SVC syndrome). CXR showed
small pleural effusion. What’s your immediate action:
a. Consult cardiologist for pericardiocentesis
b. Consult thoracic surgeon for Thoracocentesis
c. Consult oncologist(with radiotherapy symptoms
improve) the most common cause of SVC syndrome is bronchogenic
carcinoma 574-Fixs method in determining cardiac output ;??
1 BP
2 o2 saturation in blood

103
COP = PR/ BP

575-3 year old boy with acute UTI ….. first thing to do in such acute thing
;??
a-Indwelling foley cather drain
b – voiding cytctogram
c- cystoscopyd-US

576-- drug contraindication hypertrophic obstructive cardiomyopathy;


A_ digoxin
B_ one of b-blocker
Avoid digitalis because glycoside are contraindicated except in pts with
uncontrolled atrial fibirallation (emedicine)
577-- BPH pt with hypertension what to give;
alpha blocker
578-- posterior hip dislocation : ( from reconstruction)
A – flexion , adduction
b- flexion abduction
c- extension, adduction
_posterior hip dislocation :the hip is flexed ,internal rotation,adducted
(emedicine)

579-- action of ocp : ( from reconstruction)


A - inhibition of estrogen then ovulation
B – inhibition of prolactin then ovulation
d- inhibition of mid cycle gonadotropin then ovulation

580- 30 age women with sharp pain in the index finger increase with
using scissors or nail cut which cause sharp pain at the base of the
finger in matacarpophlyngeal joint and the finger become directed
downward in (mean flexed DIPj) and cause pain when try to extend the
finger..
1-trigger finger
2-tendon nodule
3-dupetren contracure
4- mullet finger
loss of extensor tendon continuity at the DIPJ causes the joint to rest on
an abnormaly flexed position
the classic mechanism of injury is a finger held rigidly in extension or
nearly full extension(emedicine)

104
581-- itching scale in pack of knee . face and ant elbow
: A – scapis
B –eczema
c- contact dermitis
eczema:the earliest lesion affect anticubital and popliteal fossa
lesions are ill defined erythematous,scaly, patches and
plaques(emedicine)
582-– mitral stenosis :
A – diastolic high pitch
B - systolic low pitch
C- diastolic low pitch

583- chylmedia non pregent treatment :


A – doxycycline
Non-pregnant:azithromycin-doxycycline. Pregnant:erythromycin.
584- – difference between unstable and stable angina :
A – necrosis of heart muscle
B - ……………………………..
Stable angina:the classic triad of angina consist of :substernal chest
pain that is provoked by exertion and relieved by rest or nitrates.
Secondary to myocardial ischemia not necrosis(necrosis in MI)
Prescence of st segment eleveation in the absence of cardiac
enzyme elevation.
ASA and B blocker (mortility benefit in stable angina)
Unstable angina:chest pain is newonset,accelerating, or occurring at
rest.(occurs with less exertion,lasts longer,less responsive to
medication)
It signals the presence of possible impending infarction.
Not associated with elevated cardiac markers but their can be st
segment elevation.
(first aid USMLE STEP 2)
585-- mechanism of destruction of joint in RA :
A – swilling of synovial fluid
B – anti inflamtory cytokines attacking the joint
d- ……………….
Synovial hyperplasia and endothelial cell activation are early events
in the pathologic process to uncontrolled inflammation and
consequent cartilage and bone destruction.
586-TB outbreak ..and one pt. come to doing tubercalin test andit's
negative .. what to do??
a- BCG
b- isonized
c- give rifampicin

105
587-all of the following is extrapyramidal Sx exept
?? a- dyskinisia
b- akathesia
c- xxxxx esia
e- clonic - tonic convulsion

588-female come to with 3 UTIs history in last 6 months,, what is your


advise?
a- wipe frome behinde to front afterdefecation
b- take a bath insted of
shaower. d- increse flude
intake

??589-in newborn exam .. what is more dangerous ?


a- hydrocele
b- abcent femoral pulses
d- CHD
e- - breast with milk discharge (normal it's with' milk)

??590-12- 28 gestation in NICU 900 gram wighet .. otherwise normal


.. what to do?? >>>> ‫دا‬
a- give hem milk orally
b- glucose infusion (because they develop
hypoglycemia) c- broad spectren antibiotic
??591-healthy child with pRBC in urin 15 cells/hpf .. what to do
..?? a- repeat urine analysis for blood and proten .
b- urine cytology.
c- Cystoscopy.
d- Renal biopsy.
592-tebial tubercle pain .. in 13 y/o boyhaving growth spurt ...Dx? >>>>
‫عنه ابحثوا‬..
a- osgood fracture
b- strees fracture
f- .........
most likely Osgood –schlatter,condition occurs in active boys and
girls from 9-16
the condition is self-limiting
-ismultible thare .. loss visual unilateral sudden with man dibetic old 593
‫ هالربع‬Dx??.. .. edema macular with retena in pigmentation ‫لمزم تقهم الفروق بين‬
‫خيارات دايم تجي في افسئلة الوفثا‬
a- retenal detachment ( wrong .. come with floters )
b- retinal artry occlosion ( wrong .. no chirry red spots )
c- retinal vien thrombosis ( my answer .. it'sfit with all data
106
given ) e- dibetic retinopathy

107
retinal
detachment:Aretinaldetachmentiscommonlyprecededbyaposteriorvitre
ous
detachmentwhichgivesrisetothesesymptoms:

▪ flashesoflight(photopsia)–
verybriefintheextremeperipheral(outsideofc
enter)partof vision

▪ asu dendramaticincreaseinthenumberoffloaters

▪ aringoffloatersorhairsjustothetemporalsideof
thecentralvision

▪ aslightfelingofheavinessintheeye

Althoughmostposteriorvitreousdetachmentsdonotprogresstor
etinaldetachments,thosethatdoproducethe
folowingsymptoms:


adenseshadowthatstartsintheperipheralvisi
onandslowlyprogressestowardsthe
centralvision

▪ theimpressionthataveilorcurtainwasdrawnover
thefieldofvision


straightlines(scale,edgeofthewal,road,
etc.)thatsu denlya pearcurved
(positiveAmslergridtest)

▪ centralvisua loss

(Noneofthisistobeconfusedwiththebrokenretinawhich
isgeneralythetearingofmuscleandnervebehindthe
eye)

 retinal artery occlusion:The most common presenting complaint is an acute persistent


painless loss of vision. In central artery occlusions, visual loss is central and dense. In branch artery
occlusions, visual loss may go unnoticed if only a section of the peripheral visual field space is affected.
 A complete visual field defect suggests central retinal artery occlusion (CRAO).
 The cherry red spot and a ground-glass retina are the classic findings but may take hours to develop.

Retinal vein thrombosis : Central retinal vein occlusion (CRVO) is essentially a diagnostic finding of
painless unilateral loss of vision. In some cases, this loss of vision is subtle in character, with intermittent

108
episodes of blurred vision. In other cases, it may be sudden and dramatic. The nonischemic type is often the
more subtle of the two, while the ischemic type is prone to the more acute clinical presentations.

 Nonischemic CRVO - Subtle, intermittent visual loss; painless; mild-to-moderate visual loss
 Ischemic CRVO - Acute visual loss; pain may be present; marked visual loss
BRVO is similar in presentation to CRVO. BRVO is often noted with an onset of blurred vision or visual field defect.
Vision loss may be subtle. Patients with small occlusions of a branch retinal vein may often be asymptomatic.
Larger obstructions can lead to significant visual loss. It is uniformly a unilateral disease. Nine percent of cases are
bilateral.

Nonischemic central retinal vein occlusion

 Mild vision loss, usually better than 20/120 measured


 Rare afferent pupillary defect

109
 Ophthalmoscopy findings consist of variable dot and flame hemorrhages in all 4 quadrants, optic nerve
swelling, retinal vein engorgement and tortuosity, cotton wool spots are few
Ischemic central retinal vein occlusion

 Marked visual loss, usually 20/200 to only hand motion


 Afferent pupillary defect
 Ophthalmoscopy findings of extensive retinal hemorrhages in all 4 quadrants, optic disc is edematous,
retinal vein markedly edematous and engorged
 Macular edema is often severe.
 Bleeding may result in vitreous hemorrhage.
 Retinal detachment may occur.
Branch retinal vein occlusion

 Patients with BRVO have retinal hemorrhages confined to the distribution of the retinal vein.
 The ophthalmoscopic examination may note triangular and flame-shaped hemorrhages.
 Mild obstruction of a branch may only show scant hemorrhage. Complete obstruction may have extensive
hemorrhage noted on examination, with cotton wool spots.

Diabetic retinopathy: In the initial stages of diabetic retinopathy, patients are generally asymptomatic; in
the more advanced stages of the disease, however, patients may experience symptoms that include floaters,
blurred vision, distortion, and progressive visual acuity loss.

Microaneurysms

Microaneurysms are the earliest clinical sign of diabetic retinopathy and occur secondary to capillary wall
outpouching due to pericyte loss. They appear as small red dots in the superficial retinal layers, and there is fibrin
and red blood cell accumulation in the microaneurysm lumen. A rupture produces blot/flame hemorrhages. Affected
areas may appear yellowish in time, as endothelial cells proliferate and produce basement membrane.

Dot and blot hemorrhages

Dot and blot hemorrhages occur as microaneurysms rupture in the deeper layers of the retina, such as the inner
nuclear and outer plexiform layers. These appear similar to microaneurysms if they are small; fluorescein
angiography may be needed to distinguish between the two.

Flame-shaped hemorrhages

Flame-shaped hemorrhages are splinter hemorrhages that occur in the more superficial nerve fiber layer.

Retinal edema and hard exudates

Retinal edema and hard exudates are caused by the breakdown of the blood-retina barrier, allowing leakage
of serum proteins, lipids, and protein from the vessels.

Cotton-wool spots

Cotton-wool spots are nerve fiber layer infarctions from occlusion of precapillary arterioles. With the use of
fluorescein angiography, there is no capillary perfusion. These are frequently bordered by microaneurysms
and vascular hyperpermeability.

Venous loops and venous beading

Venous loops and venous beading frequently occur adjacent to areas of nonperfusion and reflect increasing retinal
ischemia. Their occurrence is the most significant predictor of progression to proliferative diabetic retinopathy.

110
Intraretinal microvascular abnormalities

Intraretinal microvascular abnormalities are remodeled capillary beds without proliferative changes. These collateral
vessels do not leak on fluorescein angiography and can usually be found on the borders of the nonperfused retina.

Macular edema

Macular edema is the leading cause of visual impairment in patients with diabetes. A reported 75,000 new cases of
macular edema are diagnosed annually. This may be due to functional damage and necrosis of retinal capillaries.

Clinically significant macular edema is defined as any of the following:

 Retinal thickening located 500 μm or less from the center of the foveal avascular
zone (FAZ)
 Hard exudates with retinal thickening 500 µm or less from the center of the FAZ
 Retinal thickening 1 disc area or larger in size located within 1 disc diameter of the FAZ

594-16 y/o female become deaf suddenly.. her mother become


deafwhen she was 30.. Dx:
a- otosclerosis 100% sure(autosomal dominant,positive family history)
b- acostic neuroma
c-tympanic perforation
595-major hazard in post-menepause: >>>>>>>>>>(all true )
a- osteoprosis>>i'm 90%sure
b- hot flush
c- deppresion
d- pelvic floor weakness
596-which of the folloing b- blocker .. havean alpha blocking effect :
>>>>>>> ‫ابحثوا عنها ليني صقعت اي شئ‬
-metoprlol a
b- atenalol
c- mesoprolol
e- xxxxxx lol
f- yyyyyy lol
labetalol and carvedilol (block beta and alpha)
OSAS fros snoring primary recognize 597-theScreeningQuestionnaireto
is : ‫ وبعد بحث ينص فساعة بالنت لقيت افسم الكوفستشنير‬.. ‫فسوال ل تعليق‬
a- otowaQuestionnaire
b- HorchoverQuestionnaire
c- .........
g- ........
598-ADHD Rx : >>>>> ‫ ال ادوية كل احفظ‬ADHD a-
olanzapine
b- atomixtin
c-
,methylphenidate
111
magnesium pemoline
atomoxetine
dextroamphetomine
lisdexamfetamine
bupropion
venlafaxine
lmipramine
guanfacine
clonidine
599-patient with congistive heart failure and pulmonary edema, what is
the best treatment:
1-spronalctone
2-forsumide.
3

600-post partum women when she went back to work ,, she exposed
tothe sun and started to have brown discolortion in her face .. what is
thediagnosis:
1- uritcariA pigementosa (x)
melasma/chloasma (a patchy browen or dark brown skin
discoloration, that usually occurs on face and may result from
hormonal changes,generally found in sun exposed areas.
601-patient presented by left arm swelling , pain full axillary
lymphadenopathy … ttt by ;
a- oral antibiotics (if only lymphadenitis)
b.IV antibiotics ??(if systematic symptoms)
602- which one of the following is prognostic factor for CML ;
a- age.
b- chromosomal abnormality
in CML there is chromosomal translacation (Philadelphia)
CML was the first malignancy to be linked with clear
genetic abnormality.
603-baby sit e out support ,,crawling , walking by pulling up ,
age ..
a- 12 month
b- 10 month
c- 9 month
604-18- patient came with left arm stiffness and pain , he cant
abducted his arm .. dx
a- subcromial bursitis
b- glenohumoral arthiritis
c-
d-

112
Subacrlomia
bursitisisaconditioncausedbyinflammationoftheb
s
ursathatseparatesthe
superiorsurfaceofthesupraspinatu
tendon(oneofthefourtendonsoftherotatorcuff)fro
m the overlyingcoraco-
acromialigament,acromion,coracoid(theacromiala
rch)andfrom thed ep
surfaceofthedeltoidmuscle.

subacromialbursitisoftenpresentswithaconstelationofsymptoms

caledimpingementsyndrome.

Painalongthefrontandsideoftheshoulderisthemostcommonsympto

m andmaycauseweaknessandstiffness.
[4]
Ifthepainresolvesandweaknesspersistsothercausesshould
beevaluatedsuchasatearoftherotatorcuffora
neurologicalproblem arisingfrom

theneckorentrapmentofthesuprascapularnerve.

Theonsetofpainmaybesu

denorgradualandmayormaynotberelatedtotrauma.

Impingementmaybebroughtonbysportsactivities,suchasoverhe
adthrowingsportsandswimming,oroverhead
worksuchaspainting,carpentryorplumbing.

Activitiesthatinvolverepetitiveoverheadactivity,ordirectly
infront,maycauseshoulderpain.Directupward
pressureontheshoulder,suchasleaningonanelbowmayincrease
pain.

Nigh

timepain,especialyslepingontheaffectedshoulder,isoftenreport

ed. Localizedrednessorswelingarelesscommonandsu

gestaninfectedsubacromialbursa.

Thepatientactivelyabductsthearm andapainfularcoccursbetw
en80°and120°.
f s
Shoulderarthritiscanbeoneofthretypeso
arthritiintheglenohumeraljointoftheshoulder

113
The main symptom of shoulder arthritis is pain; this is due to the grinding of the bones against each other because of the lack of
cartilage. Pain usually occurs in the front of the shoulder and is worse with motion. People with shoulder arthritis will also
experience moderate to severe weakness, stiffness developing over many years, and the inability to sleep on the affected
shoulder.

[edit]Diagnosis

Diagnosis is simple; usually the doctor can tell if you have shoulder arthritis by your symptoms, but he or she will most likely
also take an x-ray or MRI.

[
edit]

605-- 10 yr old boy , dx as persistent moderate asthma , came


with exacerbation .. he is on bronchodialator q 4 hour,, how to
manage this patient to control his disease ;
a- no ttt needed
b- inhaler corticosteroid
c- anticholenergic

114
606-Which personality disorder is associated with inflexibility and
perfectionism:
a. Narcissistic personality disorder
b. Borderline personality disorder
c. Obsessive compulsive personality disorder
d. Histrionic personality disorder
607-male fall from 10 stairs , on examination contusion over the
nose .. ur action will be ;
a- CT scan
b- referred to ENT I think this is the correct answer bcz in exam I
choose CT scan and it was wrong
608- treatment of somatization is ;(cognitive behavioral therapy
is the best)
a- reffered to pain clinic
b- anti depressant
609-1- 50 yr old male, presented with yellowish discoloration of
both eyes and body, fatigue … O|E nothing except jaundice ,
pallor , vitiligo .. investigation ; wbc; 2500 , hgb ; 7.5 , plt ; 51 ..
LFT; elevation of total bilirubin and direct bilirubin..
Which one of the following is correct to complete this
syndrome ;
a- positive coombs test
c- antibodies against parietal cells
autoimmune hemolytic anemia:
-antibodies directed against the persons own red blood cells
-the primary illness is idiopathic ,secondary can result from
many other illness (autoimmune)
-evidence of hemolysis (incresse unconjugated bilurbin,decrease
haptoglobin,increase lactic dehydrogenase)
-specific investigation:positive direct coombs test.

610-3 yr old ingested aspirin the best gastric decontamination is:


a. Gastric lavage
b. Activated charcoal
c. Total bowel irrigation
Activated charcoal to soak up aspirin in the stomach
Laxative to cause bowel movements that help remove aspirin and charcoal from the body
Other medicines may be given through a vein, including potassium salt and sodium bicarbonate, which helps the body
remove aspirin that has already been digested.

If these treatments do not work or the overdose is extremely severe, hemodialysis may be needed to remove aspirin
from your blood.

611- 14years old girl failed in math exam .. then she hadpalapitation ,,
tachypnea and paracethesia .. this is :
115
1-hyperventilation syndrome
2-conversion

1-hyperventilation 2-parasthesia 3-pscho problems 4-no organic


causes. 612--child moved with his family to new city n he started to go
tonew school .. in the school he had low mood n doesn't want to
interactive
withany activity .. this a case of:
1-hypomania
2-depression
612-origin of pancreatic carcinoma :
Ductal epithilum
Of all pancreatic cancers, 80% are adenocarcinomas of the ductal epithelium.

613-pt. using haloperidol , developed rigidity (dystonia) ttt :


Antihistamine + anticholinergic
614- difference between primary and secondary
hyperaldosteronism :
Increase rennin in secondary
Typically, renin levels are suppressed to less than 1 ng/mL/h in patients with primary hyperaldosteronism, and
levels do not stimulate above 2 ng/mL/h with diuretics and upright posture. Because of this finding, some experts
suggest that suppressed renin levels should be used as a screen for detecting primary hyperaldosteronism.
However, in a 1993 series by Bravo, 30% of patients had renin levels that rose to greater than 2 ng/mL/h when
appropriately stimulated by sodium deprivation.[3]

615- attak rate for school children whom developed pink eye ,
first day 10 out of 50 , second day 30 out of 50 :
20
40
60
80
attack rate=number of infected/total number of exposedx100=80
616- group of diseases include , cystic fibrosis , liver failure , the
cause is :
Alpha one antitrpsin def

617- sinus tachycardia and atrial flutter , how to differentiate :


Carotid art message
Temporal art message
Adenosine iv
618- ear pain , headache . purulent discharge , right side
weakness and loss of sensation , diagnosis :
116
Epidural brain abscess

117
Subdural hematoma?
Spinal column abscess
HZV
Ramsay Hunt’s syndrome
Description:

Unilateral herpes zoster infection of the geniculate ganglion (respectively n. Intermedius) with lesions of the external
ear and oral mucosa. The symptoms include facial paralysis, severe pain in the ear with a bloody serous discharge
due to vesicles on the tympanic membrane.

source
http://www.ramsayhunt.org/epon.shtml

619- all of the following drugs contraindiacted in breast feeding


except :
Tetracycline
Chlorophenicol
Erythromycin
620- – GERD diagnosis :
By history only
History and UG Endoscopy
History and barium anema

621- old lady came to clinic as routine visit , she mention


decrease intake of Ca food , doctor suspect osteoporosis ,
next initial investigation :
DEXA
Ca in serum , thyroid function test , vit.D
According to family medicine doctor pt >65 start with
DEXA 622- long case of hemochromatosis with liver
cirrhosis and decrease weight last visit = 90 now 84 , next
step
investigation :
Hepatitis C serology
Alpha phetoprotein
Abdominal ultra sound
623- pt with rheumatic fever after untreated strep infection
after many years presented with Mitral regurge,the cause of
massive regurge is dilatation of:
1. Rt atrium
2. Rt ventricle
3. Lt atrium
4. Lt
ventricle

624- ibuproven contraindicated in :


118
Gastric ulcer
Hypertension

119
625- pregnant lady with cystitis , one of the following drugs
contraindicated in her case : imp.
Amoxicillin
Ceftriaxone
Flouroquiolone

626--female pt. with typical presentation of trichomons , ttt :


Metronidazole

627– classical case of candidal infection è itching , white


discharge from vagina , ttt is :
Miconazole
Amoxicillin
628- one of following true regarding systolic hypertension :
In elderly it’s more dangerous than diastolic htn
Occur usually due to mitral regurge
Defined as systolic , above 140 and diastolic above 100
( combined systolic and diastolic
629- 8 years boy BMI = 30 weight and hight above 95
percentile , next step :
Appointment after 1 year
Refer to surgeon
Life style modification

630– one of the following cause painful vision loss : imp.


Central retinal vein thrombosis
Retinal detachment
They are painless the painful condition is optic neuritis
631- Pt with symptoms of Mild intermittent asthma , converted
to mild persistant asthma and pt. on albterol
U have to add :
Long acting beta
Short acting inhaled steroid

_
632-Scenario for pt. with sever asthma , tight chest , tachypnea
and Co2 = 50 , next step :
Aminophylin …….
Intubation ……
Short acting beta and discharge him

120
(in acute asthma give venolin+ipratropium promide(atrovent))
633-18 month child with low grade fever , barking cough ,
inspiratory stridor your Dx :
Acute trachyobronchitis (croup)
Epiglottitis

634- Continuous murmur at left sternal area :


Patent ductus arteriosus
Mitral reg
Aortic stenosis
635- most commo cause of otorrhea ???(check question 606 in alqasim)
a) acute otitis media
b)cholesteatoma(chronic otitis media)
c)estichian tube dysfunction
Patients with acquired cholesteatomas typically present
with recurrent or persistent purulent otorrhea and hearing
loss. Tinnitus is also common.

636- the average menstrual cycle is


28 days

637- All features of tonsillar abscess except


: deviation of uvula to affected side:

638- Erethema nodosum :


painful red nodules

639- initial Inx in small bowel obstruction :


Erect& supine abdominal X- ray

640- Gold standard imaging in acute panceriatitis


: CT scan

641- pregnant lady 28 wks with chlamyda infection :(see


583) azithromycin
erythromycin or amoxicillin.
doxcyline(non pregnant)

642- common cause of male infertility:


primary hypogonadism

121
secondary hypogonadism
ejaculation obstruction

...Causes of Male Infertility


Varicocele: Varicocele is a condition that affects approximately 40 percent of infertile males. It is caused by
enlargement of the veins in the scrotum. If these veins are enlarged it can cause the temperature to increase.
Increased temperature of the scrotum affects sperm production and sperm quality.

Cryptorchidism: If the testes do not properly descend into the scrotom this is called cryptorchidsm. Cryptorchidism
is fairly common in premature births and occurs in full term births at a much lesser rate. Usually this resolves itself
within a few weeks, but sometimes corrective surgery is needed. Cryptorchism can seriously impair fertility rates. If
both testes are affected the chance of a successful pregnancy is very small.

Disease or illness: Diseases such as cystic fibrosis can affect male fertility. Men who have undergone
chemotherapy or had sexually transmitted desases may also be at greater risk of infertility.

Obstructions in the reproductive tract: Obstructions in the male reproductive system is another common cause
of infertility. Men with obstruction problems are usually making sperm but it is not making its way outside of the
penis. Common causes of obstructions are urinary tract infections, scarring from sexually transmitted disease,
injury, vasectomy or anatomical defects.

Injury: Accidents or sports injuries can cause damage to the male reproductive system resulting in infertility.

Hormone Deficiencies: Deficiencies in male hormone production such as testosterone can affect sperm
production. Doctors will usually ask questions about when puberty was started. The age at which males enter
puberty can be an indicator of hormonal disorders.

Medications: There are many medications that affect fertility and cause sexual disfunctions. It is important for a
doctor to evaluate what medications a man is taking to determine the possible impact on his fertility.

Retrograde Ejaculation: Retrograde ejaculation is caused when the nerves or muscles in the bladder do not
work properly. What happens is sperm is transported backwards into the bladder instead of forward and out
through the penis. Sometimes men will have cloudy urine after an ejaculation because of this.

If you are having infertility issues it is important that both partners are evaluated. A doctor will do a thorough
physical exam as well as evaluate the man's sperm. Men may be uncomfortable going in for an exam but it is really
necessary that both partner's be evaluated during a fertility work up. Treatment options will vary depending on what
the cause of his infertility is.

Read more:http://www.justmommies.com/articles/male-infertility.shtml#ixzz1k5jLlr00

643-18years old boy with back pain investigation to do except :


CBC ?
ESR
X -ray
bone scan

644- Hypothyroidisim :

122
free T4
TSH
T4
..

645- in which group you will do lower endoscopy for patients with
iron deficiency aneamia in with no benign cause:
male all age group
children
permanupausal women
women + OCP
Upper and lower GI investigations should be considered in

all post-menopausal female and all male patients where IDA

has

been confirmed unless there is a history of significant overt nonGI blood


loss. In the absence of suggestive symptoms (which are

unreliable) the order of investigations is determined by local

availability. The appropriateness of investigating patients with

severe co-morbidity or other reasons (in some circumstances

advanced age), especially if the result would not influence

management, should be carefully discussed with patients and

carers when possible.


http://www.bsg.org.uk/pdf_word_docs/iron_def.pdf

646- Patient with continous seizures for 35 min. despite taking 20 mg


Iv diazepam..what to do??
_ give 40 mg IV diazepam
_ give IV phenytoin
_ give IV Phenobarbital

when it reach to maximum dose of diazepam ( 20 mg ) or respiratory


depression occurs , we start second line of ttt : phynetoin at the dose of
15 mg /kg ( usually 900 - 1000 mg ) or at ratio of 10 mg of phyenoin per
123
1 ml normal saline but not exceeding 100ml over 1- 2 hour ( not
more than 50mg per minute ) ......

124
647- A 56 yr old his CBC showed, Hb=11, MCV= 93 Ret= 0.25% the
cause is:
a. Chronic renal failure
b. Liver disease
c. Sickle cell anemia
d. G6P dehydrogenase deficiency

648- 3 yr old with symptoms of acute urinary tract infection which of


the following you would like to do in this acute state:
a. Renal U/S
b. Folly catheter
c. VSUG
d- US

649-2 months old child complaining of spitting of food , abd examination


soft lax , occult blood – ve , what you will do ?
▪ Reassure the parents
▪ Abd CT
650- baby with streptococcus pharyngitis start his ttt after two days he
improved, Full course of streptococcus pharyngitis treatment with
amoxicillin is : imp.
▪ 10 days ( 9-11 days )
▪ 7days
▪ 14 days
651- Uncomplicated UTI ttt
▪ TMP-SMX for 3 days
▪ Ciprofloxacin 5 days
652- Facial injury suturing remove after? Imp.
▪ 24h
▪ 3 – 5 days ( most likely )
▪ 7 – 10 days
▪ 14 days
653- Lichen planus most common site ?
▪ Scalp
▪ Neck
▪ Knee
▪ Buttocks
▪ As far as I know the common site will be near the wrist and
the ankle but those sites were not one of the choices ?
654- One of the following condition does not cause hypokalemia
▪ Metabolic alkalosis
125
▪ Furosemide
▪ Hyperaldosteronism
▪ Acute tubular necrosis
▪ Diarrhea
655- Condition not associated with increase alpha feto protein
▪ Breech presentation
▪ Down syndrome
▪ Gastroschisis

656- Pt came with trauma to left eye by tennis ball examination


shows anterior chamber hemorrhage you must exclude ?
▪ Conjunctivitis
▪ Blepharitis
▪ Foreign body ( most likely )
▪ keratitis
657- Pt talking to doctor and the pt look to his right side most of the
time, when the doctor asked him why is that? He said that his mother is
there but in fact no one is there, after asking the pt family they said
that the mother died when he is child Dx?
▪ Visual hallucination (Or may be the doctor is blind )
▪ Auditory hallucination
▪ psychosis

658- Child after his father died start to talk to himself , walk in the street
naked when the family asked him he said that his father asked him to
do that , he suffer from those things 3 days after that he is now
completely normal and he do not remember much about what he did
Dx ???????????????????????
▪ Schizophrenia ×
▪ Schizoaffective ×
▪ Schizophreniform ×
▪ Psychosis
▪ There was a fifth choice I do not remember it, I think they
make from his father death a cause.

▪ +ve symptoms: Hallucinations (most often auditory), delusions,


disorganizedspeech, bizarre behavior, and thought disorder.
▪ -ve symptoms: Flat affect, emotional reactivity, poverty of speech,
lackof purposeful actions, and anhedonia.

126
 Schizophreniform disorder: Symptoms of schizophrenia with
a duration of < 6 months.
 Schizoaffective disorder: Combines the symptoms
of schizophrenia with a major affective disorder
(major depressive disorder or bipolar disorder).
❖ Axis I: Psychiatric disorders.
❖ Axis II: Personality disorders and mental retardation.
❖ Axis III: Physical and medical problems.
❖ Axis IV: Social and environmental problems/ stressors.
❖ Axis V: The Global Assessment of Functioning (GAF).

 Delusion: A fixed false idiosyncratic belief.


 Hallucination: Perception of an object or event without
an existing external stimulus.
 Illusion: False perception of an actual external stimulus.
 Evolution of EPS :
4 hours: Acute dystonia , 4 days: Akinesia , 4 weeks: Akathisia
, 4 months: Tardive dyskinesia
659- Pt with foreign body sensation in the eye , after the removal of
the foreign body it was insect ttt?
▪ Local antibiotic
▪ Local steroid
▪ Systemic antibiotic
▪ Systemic steroid
660- Q22/ Child with white yellow mouth lips lesion in erythmatous
base with gingivitis Dx?
▪ HSV ( most likely )
▪ EBV
▪ CMV

661- Pregnant never did check up before, her baby born


with hepatosplenomegaly and jaundice imp.
▪ Rubella
▪ CMV
▪ HSV
▪ Toxoplasmosis
662- Old Pt with abnormal ear sensation and fullness, hx of vertigo
and progressive hearing loss , invx low frequency sensorial hearing
loss Dx imp.
▪ Acoustic neuroma
127
▪ Neuritis
▪ Menieres disease
Ménière’s Disease :
o A cause of recurrent vertigo with auditory symptoms that affects at
least 1 in 500 in the United States. More common among females.
o Hx/PE: Presents with recurrent episodes of severe vertigo,
hearing loss, tinnitus, or ear fullness, often lasting hours to days.
Nausea and vomiting are typical. Patients progressively lose low-
frequency hearing over years and may become deaf on the
affected side.
663- The most difficult mode of transmission to prevent is
▪ Person to person
▪ Air droplet
▪ Vector
http://www.prep4usmle.com/forum/thread/32772/

664- Pt took high dose of acetaminophen C/O nausea vomiting Lab


increase alkaline phosphatase and bilirubin … which organ is
affected?
▪ Liver
▪ Brain
▪ Gastro
665- Female pt with hypothyroidism, TSH high But he did not give
the total T4 nor free , pulse normal BP normal she is in thyroxin what
you will do? Imp.
▪ Increase thyroxin follows after 6 months
▪ Increase thyroxin follows after 3 months
▪ Decrease thyroxin follows after 6 months
▪ Decrease thyroxin follows after 3 months
666- Pt with polycethemia vera the cause of bleeding in this pt is
▪ Increase viscosity
▪ Low platelets
▪ Thromboses and bleeding are frequent in persons with polycythemia vera (PV) and MPD, and they result
from the disruption of hemostatic mechanisms because of (1) an increased level of red blood cells and (2)
an elevation of the platelet count. There are findings that indicate the additional roles of tissue factor and
polymorphonuclear leukocytes (PMLs) in clotting, the platelet surface as a contributor to phospholipid-
dependent coagulation reactions, and the entity of microparticles. Tissue factor is also synthesized by
blood leukocytes, the level of which is increased in persons with MPD, which can contribute to thrombosis.

▪ Rusak et al evaluated the hemostatic balance in patients using thromboelastography and also studied
the effect of isovolemic erythrocytapheresis on patients with polycythemia vera. They concluded that
thromboelastography may help to assess the thrombotic risk in patients with polycythemia vera.[9]

▪ Hyperhomocystinemia is a risk factor for thrombosis and is also widely prevalent in patients with MPD
(35% in controls, 56% in persons with PV).

128
▪ Acquired von Willebrand syndrome is an established cause of bleeding in persons with MPD, accounting for
approximately 12-15% of all patients with this syndrome. von Willebrand syndrome is largely related to
the absorption of von Willebrand factor onto the platelets; reducing the platelet count should alleviate the
bleeding and the syndrome.

667- Pt k/c of SCA , the doctor planning to give him


pneumococcal vaccine which true ?
▪ Pt need antibiotic when there is hx of contact even with vaccine
( most likely )

668-50 years old pt complaining of episodes of erectile dysfunction , hx


of stress attacks and he is now in stress what you will do ?
▪ Follow relaxation strategy
▪ Viagra
▪ Ask for invx include testosterone
669-Rx. Of scabies in pregnant
women:
Permethrin and sulfur solutions
670- Child with enuresis, what is the most important single test you will
do ?
 Urine analysis
 IVP
 US
 Blood culture
 Urinalysis is the most important screening test in a child with enuresis.
Children with cystitis usually have WBCs or bacteria evident in the microscopic urinalysis.
Children with overactive bladder or dysfunctional voiding, urethral obstruction, neurogenic bladder, ectopic
ureter, or diabetes mellitus are predisposed to cystitis.
If the urinalysis findings suggest cystitis, urine should be sent for culture and
sensitivity. Urethral obstruction may be associated with RBCs in the urine.
The presence of glucose suggests diabetes mellitus.
A random or first-morning specific gravity greater than 1.020 excludes diabetes insipidus.))

emedicine

The most commonly accepted cause of nocturnal enuresis, but also the most difficult to prove, is delayed
functional maturation of the central nervous system, which reduces the child's ability to inhibit bladder
emptying at night.
...........
medscape

671- Old male pt came with fever, abd pain, diarrhea , loss of weight ,
+ ve occult blood , Labs shows that the pt infected with streptococcus
bovis , what you will do ?????
 Give antibiotic
 ORS
 Abd X-Ray

129
 Colonoscopy

130
 Metronidazole
_Colonoscopy should be performed in all patients with S bovis bacteremia or endocarditis. (emedicine)

672- Mallory weiss syndrome


 Mostly need surgery
 Mostly the bleeding stops spontaneously
 Associated with high mortality
673- Female pt around 35 years old, hx of thromboembolic
disease, what type of reversible contraceptive she can use imp.
 OCP
 Mini pills (no estrogen)
 IUCD
674- Child with fever and runny nose, conjunctivitis and cough then
he developed
Maculopapular rash started in his face and descend to involve the rest
of the body:this is case of measles
a. EBV
b. Cocxaci virus
c. Rubella virus
d. Vaccini virus
The first sign of measles is usually a high fever (often >104o F [40o C]) that typically lasts 4-7 days. This prodromal
phase is marked by malaise, fever, anorexia, and the classic triad of conjunctivitis (see the image below), cough,
and coryza (the “3 Cs”). Other possible associated symptoms include photophobia, periorbital edema, and
myalgias.

Blanching, erythematous macules and papules begin on the face at the hairline, on the sides of the neck, and
behind the ears (see the images below). Within 48 hours, they coalesce into patches and plaques that spread
cephalocaudally to the trunk and extremities, including the palms and soles, while beginning to regress
cephalocaudally, starting from the head and neck. Lesion density is greatest above the shoulders, where macular
lesions may coalesce. The eruption may also be petechial or ecchymotic in nature.

675- Avascular necrosis detect clinically AFTER


1/ 3 month
2/ 6 month??
3/ 9 month
4/ 11 month
5/ 15 month
676- - 5 yr old adopted child their recently parents brought him to you
with white nasal discharge. He is known case of SCA. What you will do
to him:
a. Give prophylactic penicillin

131
b.
c.
677- Wound, with greenish discharge, Gram + ve in long chain?
 Streptococcus
 Proteus
 Chlamydia

678- Female pt present with dysuria, urine analysis shows epithelial cast
 Contaminated sample
 Chlamydia urethritis
 Kidney disease
 Cervical disease
679- During the third trimester of pregnancy , all of the following
changes occur normally except
Decrease paco2
Decrease in wbcs
Reduced gastric emptying rate
Diminshed residual lung volume
Diminshed pelvic ligament tension

Pregnancy in the final month and labor may be associated with increased WBC levels.

http://labtestsonline.org/understanding/analytes/wbc/tab/test

680- The physiologic hypervolemia of pregnancy has clinical


significance in the management of severely injured , gravid women by:
Reduced the need for blood transfusion .
Increase the risk of pul. Edema .
Complicating the management of closed head injury
Reducing the volume of crystalloid required for resuscitation
Increasing the volume of blood loss to produce maternal hypotention

General Approach to the Trauma Patient


The primary initial goal in treating a pregnant trauma victim is to stabilize the mother's condition. The priorities
for treatment of an injured pregnant patient remain the same as those for the nonpregnant patient.

Primary Survey
As with any other injured patient, the primary survey of the injured pregnant patient addresses the
airway/cervical spine control, breathing and circulation (ABC; volume replacement/hemorrhage control), with
the mother receiving treatment priority. Supplemental oxygen is essential to prevent maternal and fetal
hypoxia. Severe trauma stimulates maternal catecholamine release, which causes uteroplacental
vasoconstriction and compromised fetal circulation. Prevention of aortocaval compression is also essential to
optimize maternal and fetal hemodynamics. Pregnant patients beyond 20 weeks' gestation should not be left
supine during the initial assessment. Left uterine displacement should be used by tilting the backboard to the
left or as a final measure, the uterus can be manually displaced.

132
Hypovolemia should be suspected before it becomes apparent because of the relative pregnancy induced
hypervolemia and hemodilution that may mask significant blood losses. Aggressive volume resuscitation is
encouraged even for normotensive patients.

http://www.trauma.org/archive/resus/pregnancytrauma.html

681- Anemia of chronic disease will show imp.


 high ferritin high iron low TIBC
 Low ferritin low iron high TIBC
 high ferritin low iron low TIBC ( this Q may come with ferritin or
without )
 Low ferritin high iron low TIBC

682- Iron deficiency anemia will show


 Low ferritin low iron low TIBC
 Low ferritin low iron high TIBC
 high ferritin low iron low TIBC
 Low ferritin high iron low TIBC


Inanemiaofchronicdiseasewithoutirondeficien
cy,ferritinlevelsshouldbenormalor
high,reflectingthefacthatironisstoredwithin
cels,andferritinisbeingproducedas
anacutephasereactantbu
thecelsarenotreleasingtheiriron.Inirondefici
ency anemiaferritinshouldbelow.[5]


TIBCshouldbehighingenuineirondeficiency,re
flectingeffortsbythebodytoproduce
moretransferrinandbindupasmuchironasposs
ible;TIBCshouldbelowornormalin
anemiaofchronicdisease.

683- Establishing to diagnosis shock must


include: Hypoxemia
Hypotention
Acidosis
Increase vascular resistance
Evidance of inadequate organ perfusion(FROM ATLS)

133
684- 35 yo male pt K/C of allergic rhinitis and bronchial asthma poorly
controlled presented with history of skin rash ,diffuse severe
abdominal pain and hand joints pain for 2days , on examination there
are diffuse purpuric skin rash and small joint tenderness with mild
effusion , the most likely diagnosis is ...
Churg strauss syndrome

134
685- 65 yo female pt who has a 10y history of symmetrical polyarthritis
is admitted to the hospital , examination reveals splenomegally,
ulceration over lateral malleoli and synovitis of the wrists, shoulders and
knees . initial investigation shows WBC 2500 ,the most likely diagnosis
is
....
Felty's syndrome !!

_
686- Patient with history of AF + MI , the best prevetion for stroke is : ?
a- Warfarin
d- Surgery procedure
e- c- Shunt
687- What is the side effect of steroid on the eye ?
a- Glaucoma .
b- Cataract .(POSTERIOR SUBCAPSULAR CATARCT) AND
GLAUCOMA IN THOSE WITH FAMILY HISTORY
c- Keratoconus .
688- Young male pt present e and pain for....... He denied any hx
of Truma or any medical illness before On ex..RR32b/min. Pulse
.....
Abdomen not tender or garding By invx WBC e in normalK >5Na..........RBS
23mmolWhat most important invx:-
U/s-
ABG
urine dipstick
689- The best investigation for kidney function :
a- 24 h collect urine
b- Creatinine clearance

690- Patient came to you with small swelling under his eye , on
examination he have inflammation in lacrimal duct , you refer him to
ophthalmologist before that what you will give him ?
a- Topical steroid
b- Topical antibiotic
c- General
antibiotic

691- What is true regarding spountinous


abortion: Can lead to infertility in future
Usually fused by any utrine abnormality
That occur on 2ed trimester..
692- The best investigation for acute diveticolitis is
135
:a- US
b- Barium
enema c- CT

136
d- Colonscopy
e- Sigmidscopy
Diverticulitis:
o Chest X-ray with the patient upright can aid detection of pneumoperitoneum.
o Abdominal X-rays may demonstrate small or large bowel dilation or ileus,
pneumoperitoneum, bowel obstruction, or soft tissue densities suggesting
abscesses.
o Contrast enemas: limited value; findings suggestive of diverticulitis include extravasated
contrast material outlining an abscess cavity, intramural sinus tract or fistula.
o CT scanning with intravenous, oral or rectal contrast: sensitivities and specificities for
CT are significantly better than for contrast enemas. When an abscess is suspected, CT
scanning is the best modality for making the diagnosis and following its course.
o Because of risk of perforation, endoscopy is generally avoided in initial assessment of
the patient with acute diverticulitis. Its use should be restricted to situations when the diagnosis
in unclear, to exclude other possible diagnoses.
http://www.patient.co.uk/doctor/Diverticular-Disease.htm

693- Food poisoning , group of people came with diarrhea and


vomiting diagnosis is:
a- Staphiloccous aureus poisoning
b- Salmonella poisoning

Usually if patients came with food poisoning after few hours of ingestion
mostly the cause is Staph due to preformed toxin, if they started to have
symptoms after one day of ingestion you are right compylobacter
salmonella shigella or E. coli
694- Patient with Rhumatoid arthritis on hand X-Ray there is
swelling what you will do for him
a- NSAID
b- Injection steroid
c- NSAIDs interfere with prostaglandin synthesis through inhibition of the enzyme cyclooxygenase (COX),
thus reducing swelling and pain. However, they do not retard joint destruction and, therefore, when used
alone, are not sufficient to treat RA

695- what is the drug that will preserve the histology in primary liver
cirrhosis:

696- propylthiouracil drug contraindicated with :


*Maternal HTN
*Maternal DM
*Maternal asthma
697- Differant between uvitis and kertits:
Dec visual acuty
Photophbia
Periorbiatl edma in keratitis
Cillry flush
137
698- 55 y/o male,, c/o angina and syncope on exertion ,, normal
ejection fraction ,, normal coronary arteries ,, there is only calcified
aortic valve with total area < .75 cm ,, the rest of examination and
investigations are normal ..Wt is ur manag. :‫مكرر‬
Avoid exertion
Medical therapy(for
angina) Aortic ballon
dilation Aortoc valve
replacrment
699- 10 months old baby came to the clinic with his mother ,
she breastfeed him 3 times a day ,, she is known cace of
epilepsy on phenobarbital,,,,,, What u going to tell her ufinal
answer
Stope breastfeeding immediately
Weaning over 2 weeks period
Breastfeed after 8 h from taking the drug ??
Respond to what the mother and child wish
henobarbital in breastmilk apparently can decrease withdrawal symptoms in infants who were exposed ..
If phenobarbital is required by the mother, it is not necessarily a reason to discontinue breastfeeding. Monitor the infant
for drowsiness, adequate weight gain, and developmental milestones
i http://www.drugs.com/breastfeeding/phenobarbital.html

700- 65 yo male pt presented with history of backache and fatigue


for the last 3 month , examination showed mild tenderness in
lumbosacral region , initial investigation revealed the following :
Hb 9
ESR 80
X ray spine showed osteolytic lesion, the most likely diagnosis is ......
Solitary myloma
701- medical therapy in ectopic pregnancy
>>> methotrexate

_
702- Best for diagnosis
E.P>> serial beta HCG

_
703- True about DUB >>
can occure in adolescent girls

704- Best anti HTN in pregnancy


>> hydralazine
methyldopa is the first line in treatment of pregnancy

138
_

139
705- Pregnant ,, smoker ,, h/o trauma >> dark red vaginal bleeding ,,
FHR 150 uterine contractions ...diagnosis :
Uterine contusion
Abruption

_
706- MC cause of 2ry amenorrhea e high LH & FSH
>> menopause

707- MC cause of milk discharge in non lactating women


>> prolactinoma

_
708- RTA case found damage of sup femoral artery 5cm in diameter , do :
*end to end anastmosing of the artery
*artery graft
*venous graft

709- Pt after URTI later on develop proximal muscle weakness ,


most probably:
*gurean barre syndrome.??
*osteoarthritis .??
710- pt have normal Na , Cl , urine PH ALL electrolyte were
normal except HCO3 was low : ( serum PH not mention )
*met acidosis ( not sure )
*met alkalosis
*res acidosis
*res alkaloso .?? (compensated)

711- 70 ys old with sever muscle pain , diarrhea , disorientation , he is


in diuretic the cause :*hyponatremia
*hypokalemia
712- Newly diagnosed Type 2 diabetic advice for diet and exercise
but still gain weight , which medication you want to add
1. insuline
2. metformin
3. sulfonylurea
713- mucopurulent discharge :
bacterial conjunctivitis

140
714- HCC :
🗷 10 %with liver disease
🗷 with chronic liver diseases
🗷 never come with smoking

🗷
Hepatocelularcarcinoma(HCC,alsocaledmalignanthe
patoma)isthemostcommontypeofliver
cancer.MostcasesofHCCaresecondarytoeitheraviralh
epatitideinfection(hepatitisBorC)
orcirhosis(alcoholism
beingthemostcommoncauseofhepaticcirhosis).[1]

🗷
Comparedtoothercancers,HCCisquiteararetumorintheUn
itedStates.Incountrieswherehepatitisis
notendemic,mostmalignantcancersintheliverarenotpr
imaryHCCbutmetastasis(spread)ofcancer from
elsewhereinthebody,e.g.,thecolon.Treatmentoptio
nsofHCCandprognosisaredependenton
manyfactorsbutespecialyontumorsizeandstaging.Tumo
rgradeisalsoimportant.

Themainriskfactorsforhepatocelularcarcinomaare:

▪ Alcoholism

▪ HepatitisB

▪ HepatitisC(25% ofcausesglobaly)[3]

▪ Aflatoxin

▪ Cirrhosisoftheliver

▪ Hemochromatosis


Wilsonsdisease(whilesometheorizetheriskinc
reases,[4]casestudiesarerare[5]and su
gestheo
positewhereWilson'sdiseaseactualymayconferp
rotection[6])

▪ Type2Diabetes(probablyaidedbyobesity)[7]
Ocp,tobacoo
141
Hepatocelularcarcinoma(HCC)mostcommonlya
pearsinapatientwithchronicviralhepatitis(hepatitisBor
hepatitisC,20%)or/andwithcirhosis(about80%).These
patientscommonlyundergosurveilance
withultrasoundduetothecost-effectiveness.

InpatientswithahighersuspicionofHCC(suchasrisingalpha-
fetoproteinanddes-gamma
carboxyprothrombinlevels),thebestmethodofdiagnosisin
volvesaCTscanoftheabdomenusingintravenous
contrastagentandthre-phasesca
ning(beforecontrastadministration,immediatelyaftercontra
stadministration,
andagainafteradelay)toincreasetheabilityoftheradiologi
s todetectsmalorsubtletumors.Itisimportan to
optimizetheparametersoftheCTexamination,becausetheund
erlyingliverdiseasethatmostHCCpatientshave
canmakethefindingsmoredifficultoa preciate.

142
715- female with problem in school -manula removal of her hair (baldness)
: (Trichotilloman )
i
Trichotillomania is hair loss from repeated urges to pull or twist the hair until it breaks off. Patients are unable to
stop this behavior, even as their hair becomes thinner.

716- pregnant 6 days in CS - staining in her throbs from abdomen :


Fascial dehiscence
717-16 w ek , polydipsia polyurea , less than 126 mg fasting ,, 6.8 :
impaird DM
IFG: ( 6.1-7.0 mmol)
IGT : ( 7.8-11.1 mmol/l, 2h after 75g )
- First step: One-hour 50-g glucose challenge test; venous plasma glucose is
measured one hour later (at 24–28 w eks). Values ≥ 140 mg/dL are
considered abnormal.
- Next step: Confi rm with an oral three-hour (100-g) glucose tolerance test
showing any two of the following: fasting >95 mg/dL; one hour >180 mg/ dL;
two hours > 155 mg/dL; three hours >140 mg/dL.
718- Pancreatitis :
increase by lying down
719-50 years , back pain , x ray sowed lytic lesion :
bone scan

1-bone ma row biopsy 2-protien electrophoresis of blood and urine>paraprotien


720- exa gerated reflex in jaw , no fasculation clty in swallowing :
, dffii pseudobulbar palsy

Pseudobulbar palsy results from an upper motor neuron lesion to the corticobulbar
pathways in the pyramidal tract. Patients have dffii culty chewing, swallowing and
demonstrate slu red speech (often initial presentation). Individuals with
pseudobulbar palsy also demonstrate inappropriate emotional outbursts.
S/S :
* Speech is slow, thick and indistinct - Gag reflex is normal, exa gerated
or absent - Tongue is small, stiff and spastic - Jaw jerk is brisk - upper motor
neuron
lesion of the limbs - Dysphagia culty in swallowing) - Labile affect –Dysarthria -
(dffii Uncontrollable laughing or
crying
Bulbar palsy refers to bilateral impairment of function of the lower cranial
nerves IX, X, XI and XII, which occurs due to lower motor neuron lesion either
at nuclear or fascicular level in the medulla oblongata or from bilateral lesions
143
of the lower cranial nerves outside the brainstem.[1]
S/S :

144
dysphagia culty in swallowing) - culty in chewing - nasal regurgitation -
(dffii dffii
slu ring of speech - choking on liquids - Nasal speech lacking in modulation and
dffii culty with all consonants - Tongue is atrophic and shows fasciculations -
Dribbling of saliva - Weakness of the soft palate, examined by asking the patient
to say aah - The jaw jerk is normal or absent - The gag reflex is absent -
lower motor neuron lesions of the limbs.
721- High senstive & specific for urolithasis :
CT scan
722- child in ER , with dyspnea , tachy pnea , subepiglottic na rowing in x-
ray : If thumb sign : epiglottitis , if steeple sign : croup
CROUP Diagnosis :
A frontal X-ray of the neck is not routinely performed, but if it is done, it may
show a characteristic na rowing of the trachea, called the steeple sign. The
steeple sign is su gestive of the diagnosis of CROUP , but is absent in half of
cases.
723- female with breast cancer :before 5 years received chmeotherpy .
diagnosed now CLL , causes of CML : risk factor .
The etiology is unknown, although there is some genetic contribution, as fi
rst- degree relatives of patients with CLL are three times more likely than
others to develop a lymphoid malignancy. Primarily affects older adults (median
age 65); the male-to-female ratio is 2:1.
724- antipsychotic drug side effect for onset :
4 hours: Acute dystonia , 4 days: Akinesia , 4 w eks: Akathisia , 4 months: Tardive
dyskinesia (often permanent)

_
725- chronic use of alcohol : first drug to give pt :

thiamine .
. All patients

being treated for AW should be given

100 milligrams (mg) of thiamine as

soon as treatment begins and daily

during the withdrawal period.


http://pubs.niaaa.nih.gov/publications/arh22-1/38-43.pdf

_
145
726- female with vaginal bleeding , abdominal pain : first
Inx : US

146
Vaginal Examination
727- stroke pt , most propable cause :
🗷 Polycethmia vera
🗷 Sickle ceLl anemia
🗷 2 ry polycethmia .

_
728- old male , back pain , ex is normal : gave him steroid , come again with
vesicle from back to abdomen : VZV
729- Female , Rt hand , lateral two radial styloid processes pain , since
month increase progressively , CS , t t of De
Quervain Tenosynovitis +ve Finkelstein test initial tt :
🗷 Nerve decompe rison
🗷 cast upper joint
🗷 cast with thumb raised
✓ Initial treatment for DeQuervain's syndrome is nonoperative : first
thumb- spica splint , NSAIDS may also be of value , corticosteriod
injection into the first dorsal compartment may provide sustained relief .
o In one study, Weiss et al. reported that injection was just as
effective in DeQuervain's syndrome as splinting alone or splinting
combined with injection. In another study of 63 patients, 71% of
patients had sustained relief of their symptoms with one first dorsal
compartment injection.
730- male with auscultation , not clear , left sterna border , scratching sound ,
vein distened in neck , muffled heart sound :
Cardiac tamponade
percarditis
731- Acanthosis Nigricans associated with :
polycystic ovary syndrome
732- celiac disease which not cause it :
rice&corn
oat
wh el
gluten
733- old man with generalized abdominal pain T:38.2,abscent bowel
sound,x ray:dilated small bowel and part of the transverse colon,no fluid
level:
🗷 Pancreatitis
🗷 perforated peptic ulcer
🗷 bacterial colitis
🗷 intestinal obstruction
147
🗷 Sentinel loop sign:isolated dilatation of a segment of gut consisting of jejunum, transverse colon or duodenum.
Colon cutof f sign:gas distition rt colon that abruptly stopes in mid or LF colon due spasm adjacent to
inflammation

734- baby with tonic clonic convulsions,what drug you'll give the mother to
take home if ther is another seizure:
🗷 Diazepam
🗷 phenytoin
🗷 Phenobarb
735- Significant DPL direct peritoneal lavage in diagnosis :
🗷 ≥10 ml blood or ≥100,000 RBC or ≥ 500 WBC
A positive DPL in an adult classically requires one of the following results:
✓ RBC : >100,000/mm3
✓ blood : 10 ml
✓ Amylase level (IU/L) : ≥20
✓ Alkaline phosphatase level (IU/L) : ≥3
✓ WBCs (per mm3) :>500
✓ the presence of enteric/vegetable ma
ter 736-Initial management for Frostbite
patient :
🗷 Debridement
🗷 b.blocker
🗷 corticosretoid
🗷 immersion in 40 C .
737- Patient with ACEI , was not uncontrolled , what do will add to the patient :

🗷 B.Blocker
🗷 ARBs
🗷 CCBs
🗷 Thiazide
🗷 Frusemide

_738- Patient with HTN , discovered DM 2 ,what the medication will add to him :
🗷 Blocker
🗷 ARBs
🗷 CCBs
🗷 Thiazide
🗷 ACEIs

739- Patient with hypertensive retinopathy grade 2 AV nicking , normal BP ,


no decrease in vision , with cupping of optic disc , what will do do the
148
patient :
🗷 Reassurance , the problem is benign

149
🗷 Convert hitm to ophthalmologis
🗷 Laser operation
740- Patient with sudden skin eruption over face and neck then palm
and sole ????????
🗷 Erythema multiform
🗷 Drug eruption
🗷 Measles
If syphilis is there it is more accurate
741-100 - Patient witn BA , using B.agonist if needed , now become more sever
, attacks come 5 times in w ek., you will give patient , I expect Moderate
persistent

🗷 Low dose steroid inhaler


🗷 Oral steroids
🗷 Ipratorpium bromide
🗷 Long acting B.agonist
🗷 Large dose steroid inhaler
742- One of them causes conductive hearing loss :
🗷 Acute ottis media
🗷 Syphillis
🗷 Meneria disease
743- CSF : normal protein , decrease glucose , turbid in color :
🗷 Viral menengititis
🗷 Tubercular meningitis
🗷 Pyogenic meningitis
🗷 Encephalitis

Cause A pearanc Polymorphonucle Lymphocyte Protein Glucose


e ar cell
Pyogenic Yellowish, Markedly Slightly Markedly Decrease
bacterial turbid increased increased increased d
meningitis or
Normal

Viral Clear fluid Slightly Markedly Slightly Normal


meningitis increased increased increased
or Normal or
Normal

Tuberculous Yellowish viscous Slightly ed or Normal


meningitis and increas
150
Markedly Incre ased Decre d
increased ase
Fungal Yellowish Slightly increased Markedly Slightly Normal or

151
meningitis and or Normal increased decrease
viscou d
s increase
d or
Normal

_
744- Patient with vomiting and dia rhea and moderate dehydration, how to treat:

🗷 ORS only
745- in knee examination : +ve lechman test indicate injury :
Anterior cruciate ligament
746-2 w eks after delivery , mother said that the baby , he will die ,
baby lack healthy , what is diagnosis :??
🗷 Post partum no bonding
🗷 Post partum psychosis
🗷 Post partum
depression 747-
antidepressant in elderly :
Will take time to see effect
"I make sure that patients and their family—who I always try to include—understand how long it can take to
respond to an antidepressant. People need clear expectations. The drugs take longer to work in older people
than younger ones, usually 8 to 12 weeks. The longer they stay on the medicine, the more improvement they
are likely to see,"
http://www.health.com/health/condition-article/0,,20188909_2,00.html
748- children on chemotharpy , he developed septicemia after introduce IV
canula , what is causative organisms : imp.??
Hib
Psudeomonas??
E.coli
strept
klebsiella

749- 1-2012 pt came e" painful rectal spasm , diaphorisis , tachycardia


at night what the DX :
a- thrombosed hemorhoid
b- Proctalgia Fugax

152
c- >>syndrome

Proctalgiafugax(o
r levatorsyndrom
e
)isasevere,
, l l .]
[1
episodicr
e sercta ands
i acrococcygea pain
Itcanbecausedbycrampof th

Itmostoftenoccursinthemidleofthenight[3]andlastsfrom
secondstominutes,[4]anindicatorforthedifferential
diagnosisoflevatoranisyndrome,whichpresentsaspainandach
inglastingtwentyminutesorlonger.Inastudy

153
publishedin2
07involving1809patients,theatacksoccuredinthedaytime,
(3percent)aswelasatnight(3
percent)andtheaveragenumberofatackswas13.Onsetcanbei
nchildh od,however,inmultiplestudiesthe
averageageofonsetwas45.Manystudiesshowedthatwomenareaffe
ctedmorecommonlythanmen.[5]

Duringanepisode,whichsometimesoccursafterorgasm,th
epatientfelsspasm-like,sometimesexcruciating,
painintheanus,oftenmisinterpretedasan
edtodefecate.Simultaneousstimulationofthelocalautonom
icsystem
cancauseerectioninmales.Becauseofthehighincidentofinter
nalanalsphincterthickeningwiththedisorder,itis
though
tobeadisorderoftheinternalanalsphincterorthatitisaneura
lgiaofpudendalnerves.Itisrecurentand
thereisalsonoknowncure.However,somestudiesshoweffect
iveuseofbotulinum toxin,pudendalnerveblock, andcalcium
cha
nelblockers.Itisnotknowntobelinkedtoanydiseaseproces
sanddataonthenumberofpeople
afflictedvaries,butismoreprevalen thanusualythought.

Thepainepisodesubsidesbyitselfasthespasm disa
pearsonitsown,butmayreoccur.[4]

Sometimesthereisadropinblodpressurethatmaycauselos
sofconsciousnessandpossibleinjury.Staying
downissu
gestedifinbedandlayingdownisrecommended.

750- old pt , e hx of MI 2 weeks back and discharge from hospital 24


hrs prior to his presentation <<<came with sudden lower limb pain and
numbness ,on ex the limb pale , cold >>the other limb normal what is
the DX :
a-Acute artery
thrombosis b- acute artery
embolus
d- DVT
e- D- ???
751-7- Neonate with mass on his eye :
a-Neuroblastoma
b-Leukemia

(Neuroblastoma
154
is the most common extracranial solid tumor in infancy and the third most common
pediatric malignancy after leukemia and central nervous system (CNS) tumors.
It is an embryonal malignancy of the sympathetic nervous system arising from
neuroblasts (pluripotent sympathetic cells). Signs and symptoms of neuroblastoma
vary with site of presentation. Generally, symptoms include abdominal pain, emesis,
weight loss, anorexia, fatigue, and bone pain. At diagnosis, the site of
neuroblastoma is predictably age-dependent. Infants often present with
compression of the sympathetic ganglia in the thoracic region, which might result,
for example, in Horner syndrome (myosis, anhydrosis, and ptosis) or superior vena
cava syndrome. Older children typically present with abdominal symptoms because,
as stated above, more than 40% of neuroblastomas are adrenal in origin. Children
who are preschool aged should have working differential diagnoses for an
abdominal mass, including lymphoma, hepatoblastoma, rhabdomyosarcoma, renal
cell carcinoma, and neuroblastoma.

http://emedicine.medscape.com/article/988284-overview#a0104

155
Leukemia
is a type of cancer of the blood or bone marrow characterized by an abnormal
increase of immature white blood cells called "blasts".In most patients, a diagnosis
of leukemia has been made before presenting to an ophthalmologist. However, in
some patients, ocular symptoms and examination lead to a diagnosis of
leukemia.Orbital manifestations

o Leukemic cells may infiltrate the orbit during the course of acute or chronic leukemia.
Unusual orbital involvement with leukemia has been reported to include infiltration of the
lacrimal gland and drainage system, rectus muscles, and dermis.

o Orbital involvement in children is more common in acute leukemias, whereas orbital


involvement in adults is more common in chronic leukemias.

o The leukemic infiltrate may range from insignificant, where it is virtually asymptomatic, to
a space-occupying lesion with its concomitant symptoms.

o The patient may have proptosis, ecchymosis, chemosis, diplopia, visual disturbance, or
motility disturbances.

o In children, the orbital involvement is characterized by an acute and rapid process that may
be confused with orbital cellulitis. In general, these infiltrates are bilateral and do not
destroy bone. Granulocytic sarcoma of the orbit, also known as chloroma, is an
extramedullary form of myelogenous leukemia.

o Unilateral, painless proptosis develops over weeks to months prior to a diagnosis of


leukemia. Eyelid redness or violaceous discoloration may be present, which turns into
ecchymosis that may be confused with rhabdomyosarcoma or metastatic neuroblastoma. If
AML or CML is already present, then a rapid and fulminant bilateral proptosis is
characteristic.

o http://emedicine.medscape.com/article/1201870-clinical#a0217

752- Pt with 3 month hx of feeling like depression, recently


employ what you will do:
a-
Tricyclic b
c-Psychotherapy support

156
d- Moi
the recommended treatment for adjustment disorder is psychotherapy

??753- WRIST WITH PAPULOVESICULAR RASH 12 ‫صوره‬


CHICKEN POX >> Chickenpox is usually diagnosed clinically on the basis of the
characteristic rash and successive crops of lesions.
Small, erythematous macules appear on the scalp, face, trunk, and proximal limbs,
with rapid sequential progression over 12-14 hours to papules, clear vesicles, and
pustules and subsequent central umbilication and crust formation.
Vesicles may appear on the palms and the soles and on the mucous
membranes, together with painful, shallow, oropharyngeal or urogenital ulcers.
Intense pruritus commonly accompanies the vesicular stage of the rash.

Vesicular eruption on the trunk demonstrating papules, vesicles, and crusts

Dewdrop on rose petal characteristic vesicle of chickenpox

SCABES>>superficial burrows

157
A typical linear burrow on the flexor forearm

A subtle linear burrow accompanied by erythematous papules on the sole of the


foot in a child with scabies.

Erythematous papules and papulovesicles on the flexor wrist.

ATROFIC DERMATITIS>> red, flaky and very itchy. The skin on the flexural
surfaces of the joints (for example inner sides of elbows and knees) are the most
commonly affected regions in people.

Flexural involvement in childhood atopic dermatitis.

Dermatitis herpitiform>>is characterized by grouped excoriations;


erythematous, urticarial plaques; and papules with vesicles.

Classic vesicles of dermatitis herpetiformis.

158
754- CHILD SEVERLY ILL AND FEVER FOR 2 DAYS ANOREXIA NAUSEA
VOMITING THEN PETECHIA RASH APEAR IN TRUNK AND SPREAD
IN THE BODY ??
MEASELS
MENINGOCOCCAL MENINGITIS
MOUNTAIN FEVER
Varicella low grade fever for 2 days then rash in the trunk then it spread
in the body
755- Patient on amitriptyline 30mg before bed time wake up with
sever headache and confusion what is the appropriate action ?
Shift him to ssri
Change the dose to 10mg 3time daily
756- Young patient with decrease hearing and familly history of
hearing loss ear examination normal rene and weber test revealed
that bone conduction is more than air conduction what would you do
?
Till him it is only temporal
Till him there is no ttt for this
condition Refer to audiometry
Refer to otolaryngologists
757- which one of these drugs causing hypertensive crisis
when it is not stoped gradually?
diltiazim>>Antidysrhythmics, IV; Calcium Channel Blockers,
colonidine>>Rebound hypertension on withdrawal
b blocker
Clonidine suppresses sympathetic outflow resulting in lower blood
pressure, but sudden discontinuation can cause rebound
hypertension due to a rebound in sympathetic outflow.
Clonidine therapy should generally be gradually tapered off when discontinuing therapy to avoid rebound effects from
occurring. Treatment of clonidine withdrawal hypertension depends on the severity of the condition. Reintroduction of
clonidine for mild cases, alpha and beta blockers for more urgent situations. Beta blockers never should be used alone to treat
clonidine withdrawal as alpha vasoconstriction would still continue

758- @1year old massive hepatosplenomegaly , brown skin


nodule, lateral neck lump investigation to diagnose
Ebv serology
Lp
159
bone marow
aspiration liver biopsy

759- child came with hematuria 15 RBC HPF , all examination


normal what will you do ?
a urine cytology
b renal bipsy
c – cystosocopy
c repeat urine for RBC and protein

760- 15 years old with palpitation and fatigue. Investigation


showed RT ventricularhypertrophy, RT ventricular
overload and right branch block what is the diagnosis
: a- ASD
b- VSD
Complications of VSDs include the following:
Heart block secondary to intracardiac repair
c- Coartaction of aorta

761- child came withor Toeing-In , set in W shape , when walk both
feet and knee inward with 20 degree , both femur inwarr rotation 70
degree
<<like this """ what the Dx :
a metatarsus adductus
b-femoral anteversion (femoral torsion )>>
Femoral anteversion
Normal femoral anteversion is 40º in the newborn and decreases to 10º by the age of 8 years. The acetabulum is angled
forward 15º. Femoral anteversion does not increase the risk of arthritis of the hip. Spontaneous improvement in the anatomic
position can occur until the patient is aged 8 years and can further correct by improving the gait through conscious effort until
adolescence.

Femoral anteversion is the axial angle between the plane of the neck of the femur
and the femoral condyles. It can be clinically deduced by measuring the hip rotation.
Normal range of external rotation is 45-70°, and internal rotation is 10-45°. As
femoral anteversion increases, the amount of internal rotation increases and
external rotation decreases. These children can have as much as 90° of internal
rotation and 0° of external rotation. They sit in the W position with their legs turned
out (a position not attainable by normal adults), but they cannot sit cross-legged
c-???

762- anticoagulation prescribed for


- one month
- 6 months
160
- 6 weeks
- one year

763- child with moderate persistant BA On bronch.dilat


inhaler. Presented with acute exacerbation what will you add
in ttt:
Corticosteroid inhaler
Ipratropum bromide inhaler

children with asthma are divided into 3 groups based on age: 0-4 y, 5-11 y, 12 y and older.

For all patients, quick-relief medications include rapid-acting beta2-agonists as needed for symptoms. The
intensity of treatment depends on the severity of symptoms. If rapid-acting beta2-agonists are used more
than 2 days a week for symptom relief (not including use of rapid-acting beta2-agonists for prevention of
exercise induce symptoms), stepping up treatment may be considered

Stepwise Approach to Asthma Medications


Intermittent
Asthma Persistent Asthma: Daily Medication
Age Step 1 Step 2 Step 3 Step 4 Step 5 Step 6
< 5 y Rapid- Low-dose inhaled Medium-dose Medium-dose High-dose ICS High-dose ICS plus
acting corticosteroid ICS ICS plus plus either either LABA or
beta2- (ICS) either long- LABA or montelukast; Oral
agonist Alternate acting beta2- montelukast systemic
prn regimen: agonist corticosteroid
cromolyn or (LABA) or
montelukast montelukast
5-11 Rapid- Low-dose ICS Either low-dose Medium-dose High-dose ICS High-dose ICS plus
y acting ICS plus either ICS plus LABA plus LABA LABA plus
beta2- LABA, LTRA, oral systemic
agonist or theophylline corticosteroid
prn Alternate OR Medium- Alternate Alternate Alternate regimen:
regimen: dose regimen: regimen: high- high-dose ICS plus
cromolyn, medium-dose dose ICS plus LRTA or
leukotriene ICS plus either LABA theophylline plus
receptor either LTRA or theophylline systemic
antagonist or theophylline corticosteroid
(LTRA), or
theophylline
12 y Rapid- Low-dose ICS Low-dose ICS Medium-dose High-dose ICS High-dose ICS plus
or acting plus LABA ICS plus LABA plus LABA (and either LABA plus
OR
older beta2- Medium-dose consider oral corticosteroid
agonist ICS omalizumab for (and consider
as Alternate Alternate Alternate patients with omalizumab for
needed regimen: regimen: low- regimen: allergies) patients with
161
cromolyn, LTRA, dose ICS plus medium-dose allergies)
or theophylline either LTRA, ICS plus either

162
theophylline, or LTRA,
zileuton

163
pt is a known case of moderate intermittent bronchial asthma. He is
using ventoline nebulizer. He develops 3 attacks per week. The drug to
be added is:
a) Increase prednisolone dose
b) Add long acting B agonist
c) Add ipratropium >>Anticholinergic bronchodilator
d) IV aminophylline

_
764- A 20 years old male who is a known asthmatic presented to the
ER with shortness of breath. PR 120, RR 30, PEFR 100/min.
examination revealed very quite chest. What is the most propable
management?
a) Nubelized salbutamol
b) IV aminophyline
c) Pleural aspiration
d) Hemlich maneuver
e) Chest drain

765- about crohn's disease are true :


Inflammation Involve superfacial layer of intestine
164
Involve sigmoid and rectum if (“skip lesions”).

165
Decrease incidence of colon cancer
The rectum is often spared. Transmural inf l ammation is seen.
(correct)
766- old age male with h/o of gastric acidity relieved by antiacids
,now he c/o forceful vomiting at night of food content >>>>> dx:
Gastric outlet obstruction?>>is a medical condition where there is an
obstruction at the level of the pylorus, which is the outlet of the stomach.

767- which drug increase incidence of reflux


oesophagitis: Thiophylline (correct)
Amoxicillin
Metoclopramide
Rantidine
Lansoprazole

_
768- which drug cause hypertensive crises when used with tyramine :
SSRI
Tricyclic antidepressant
MAOI (correct)

769- old ptn with HTN and migrane ttt:


B blockers (correct)
ACE I
Ca blockers
770- the most common fracture in osteoporosis : ( I couldn’t
remember the age)
Spinal fractures (compressed vertebrae)are the most
common osteoporotic-related fractures
Colles fracture (if prior 75
y) Fracture neck of femur
shaft of femur
hip fracture(if over 75y)
771- male ptn with scaly fine papular rash on fornt of scalp,nose
and retroauricular……..(i think tinea capitis) ttt is:
Ketoconazole cream…
Oral augmentin
……… cream
tinea capitis : single or multiple patches of hair loss,
sometimes with a 'black dot' pattern (often with broken-of f hairs),
that may be accompanied by inflammation, scaling, pustules, and
itching.
Treatment : oral antifungal agent; griseofulvin is the most
commonly used drug, but other newer antimycotic drugs, such
166
as

167
terbinafine, itraconazole, and fluconazole have started to gain
acceptance.
dx : Wood's lamp examination

772- Gram stain reveal gram negative diplococcic (you must know
about causative org. of this type)…….
Ceftriaxone IM or cefepime PO one dose..(Nisseria gonorrhea) >>
is a species of Gram-negativecoffee bean-shaped diplococcibacteria responsible
for the sexually transmitted infectiongonorrhea.
ttt : ceftriaxone (a third-generation cephalosporin)
773- A side effect of L-Dopa:
a) Dyskinesia (correct)
b) ..
c) ..
d) ..

774- Aspirin & clopedogril indicated for:


a) A 55 year old man with acute coronary syndrome.
b) A 65 year old man with hx of MI
c) A 65 yo man with stroke.
d) A 65 yo man with CHF
e) After cardiac catheterization ??
775- Plavix & aspirin used for:
a) Pt with previous history of ischemic stroke
b) Pt going for angiogram
c) MI
d) Acute coronary syndrome

_
776- A pt who is a KCO posterior MI presented with syncope.
Examination showed canon (a) wave with tachycardia, unreadable BP &
wide QRS complexes on ECG. The diagnosis is:
a) Atrioventricular re-entrant nodal tachycardia
b) Ventricular tachycardia
if the individual has a past history of a myocardial infarction, congestive
heart failure, or recent angina, the wide complex tachycardia is much more
likely to be ventricular tachycardia
c) Pre-existing AV block
d) Anterograde AV block
e) Bundle branch block
cannon wave It is associated with heart block, in particular third-degree
(complete) heart block
777- The drug with the least side effects for the treatment of SLE is:

168
a) NSAIDs (correct)>>Common side effects of NSAIDs can include:

 stomach upset

 heartburn

 diarrhea

 fluid retention

b) Methotrexate>>Immunosuppressant drug. Side effects may include:

 nausea

 vomiting

 hair loss

 bladder problems

 decreased fertility

 increased risk of cancer and infection

c) Corticosteroid>>Short-term side effects of corticosteroids include:

 swelling

 increased appetite

 weight gain

These side effects generally stop when the drug is stopped. It is dangerous (even life threatening) to
stop taking corticosteroids suddenly

Long-term side effects of corticosteroids can include:

 stretch marks

 weakened or damaged bones (osteoporosis and osteonecrosis)

 high blood pressure

 damage to the arteries

 high blood sugar (diabetes)

 infections

169
 cataracts

d) Hydroxychloroquin >>Side effects of anti-malarials can include:

 stomach upset

 damage to the retina of the eye (rare)

778- A young male who is a known case of sickle cell anemia presented
with abdominal pain & joint pain. He is usually managed by
hospitalization. Your management is:
a) In-patient management & hospitalization
b) Out-patient management by NSAID
c) Hydration, analgesia, monitoring. (correct)
d) Narcotic opioids

_
779- A lot of bacteria produce toxins which are harmful. Which one of
the following is used in amiddirs:
a) Botulism??
b) Tetanus
c) Diphtheria
d) Staph aureus

780- All the following cause hyponatremia except:


a) DKA
b) Diabetes insipidus<<Diabetes insipidus causes HYPER (correct)
c) High vasopressin level
d) Heart failure

781- Warfarin is given to all the following except:


a) Young male with Atrial fibrillation & mitral stenosis
b) Male with AF & cardiomyopathy
c) Male with AF & prosthetic heart valve
d) Elderly male with normal heart (correct)

170
782-. A 24 Y/o man presented with 4 month Hx of diarrhea with streaks of
blood & mucous. Ulcerative colitis was confirmed by colonoscopy. The
initial therapy for this patient:(481)
a) oral corticostreiod xx
b) azathioprine
c) infleximabe
d) 5-Aminosalicylic acid
e) Sulfasalazine
Sulfasalazine has been a major agent in the therapy of mild to moderate UC
for over 50 years. In 1977, Mastan S. Kalsi et al. determined that 5-
aminosalicylic acid (5-ASA and mesalazine) was the therapeutically active in
sulfasalazine.[citation needed] Since then many 5-ASA compounds have been
developed with the aim of maintaining efficacy but reducing the common
side effects associated with the sulfapyridine moiety in sulfasalazine

http://emedicine.medscape.com/article/183084-treatment#aw2aab6b6b2
783- A hypothyroid pt on thyroxin had anorexia, dry cough, dyspnea & left
ventricular dysfunction. She had normal TSH & T4 levels,
Hyperphosphatemia & hypocalcemia. The diagnosis is:
a) Primary hypoparathyroidism
b) Secondary hypoparathyroidism xx
c) Hypopituitaritism
d) Uncontrolled hyperthyroidism
784-A 55 yr old man presenting with Hx of streakes of blood in stool and
dull pain on defecation that persists for half an hour after defecation, on
examination there was a 3x2 cm thrombosed mass at 3 o’clock.What is the
management?
a) Sitz bath 5 times/ day.
b) Application of local anesthetic and incision.
c) Application of antibiotic
d) Band ligation and wait for it to fall
e) Application of local anesthetic ointment

_
785- A 42 year old woman presented with a painful breast mass about 4
cm in the upper lateral quadrant. It increases in size with the menstrual
period. Examination showed a tender nodularity of both breasts. What is
the management:
a) Hormonal treatment with oral contraceptive pills
b) Hormonal treatment with danazol?? xx
c) Lumpectomy
d) Observation for 6 months
787-
171
786- A 48 year old man complaining of right lower quadrant pain, bleeding
per rectum, nausea & vomiting. What is the best pre-operative investigation?
a) Air contrast enema
b) Fecal occult blood
c) CBC
d) …
788- Which of the following indicate large uncomplicated pneumothorax:
a) Symmetrical chest movement.
b) Increase breath sound
c) Dull percution note.
d) Tracheal deviation (correct)
e) Cracking sound with each heart beat

_
789-. A pregnant woman, multigravida, 38 weeks gestational age
presented with glucosuria. Gestational diabetes was confirmed by glucose
tolerance test. The next step is:
a) Repeat Glucose tolerance test
b) Cesarean section
c) Diet adjustment
d) Start sliding scale insulin

790-A young primigravida, 35 weeks gestation, had BP of 140/90,


headache, proteinurea & lower limb edema. What is the best management:
(dx preeclampsia)
a) Oral labetolol
b) Diuretics
c) Low sodium diet
d) Immediate C.section
e) Admission & observation of feto-maternal condition (correct)

791- A 30 yo lady in the third trimester of her pregnancy developed a


sudden massive swelling of the left lower extremity extending from the
inguinal ligament to the ankle. The most appropriate sequence of work up
& treatment:
a. Venogram, bed rest, heparin
b. Impedance plethysmography, bed rest, heparin
c. Impedance plethysmography, bed rest, vena caval filter
d. Impedance plethysmography, bed rest,
heparin, warfarin
e. Clinical evaluation, bed rest, warfarin
792-Elderly female married since 30 years had fever, chills, dysurea, and
diarrhea. No back pain. The diagnosis is:
172
a) Acute bacterial cystitis
b) Acute pyelonephritis >>In acute pyelonephritis, the classic triad of fever,
costovertebral angle pain, and nausea and/or vomiting may be present,
although they may not necessarily occur together temporally.
c) Bacterial gastroenteritis
d) Viral gastroenteritis

_
793- A young female patient who is an office worker presented with
itching in the vagina associated with the greenish-yellowish vaginal
discharge.
Examination revealed red spots on the cervix. The diagnosis is:
a) Trichomoniasis (correct)Women with trichomoniasis frequently report a
frothy yellowish-green vaginal discharge
b) Candidiasis>>whitish or whitish-gray cottage cheese-like discharge
c) Gonorrhea
d) Gardnerella vaginalis
794- Uterovaginal prolapse:
a) Increase heaviness in erect position (correct)
b) More in blacks
c) A common cause of infertility
d) ..

795- A patient presented with fatigue, loss a petite & bloody urine. She
gave History of sore throat 3 weeks back. The most likely diagnosis is:
a) hemorragic pyelonephritis
b) Post streptococcal GN(correct)
c) Heamorragic cystitis
d) membranous GN
e) IgA nephropathy

?? 796- A child of an HIV positive mother. He is not immunized. Which


of the following vaccines should not be given to him?
a) Live oral polio
b) DTP
c) MMR
d) Hepatitis B

797- A child swallowed his relative's medication. What is the best way of
gastric decontamination? ???????????? ‫ا‬
a) Gastric lavage
173
b) Total bowel irrigation (whole bowel wash)

174
c) Syrup ipecac
d) Activated charcoal>>(correct) For most medication ingestions,
single- dose activated charcoal is the modality of choice for
gastrointestinal decontamination. This treatment can generally be used
up to 1 hour after ingestion of a potentially toxic amount of medication

‫افي المذكرة صجونا بالقابستريك الفاج مع البسبرين‬

_
798- The commonest chromosomal disease is:
a) Down syndrome (Trisomy 21) (correct)
b) Klinfelter syndrome
c) Turner's syndrome

?? 799-A young girl pt had URTI 1 week ago & received septra
(trimethoprime + sulphamethoxazole). She came with crampy abdominal
pain & proximal muscle weakness. The diagnosis is:(the same question in
alqasem but other choises)474
a) Polymyositis
b) Gullian parre syndrome
c) Intermittent porphyria
d) Periodic hypokalemic paralysis
e) Neuritis

800-All the following are present in otitis media except:


a) Signs & symptoms of inflammation
b) Signs & symptoms of
effusion c) High grade
fever
d) Pain

801-Best ttt for chronic pain management:


Ibuprofen >> NSAIDare most beneficial in cases of acute pain, or flare-ups in patients with
chronic pain. NSAIDs are excellent at treating inflammatory conditions including tendonitis, bursitis,
and arthritis.
acetaminophen (correct)>>in cases of chronic pain, no inflammation is at the site of the
pain, and thus Tylenol may be an appropriate treatment choice.
In general, NSAID use is limited for patients with chronic pain because of concerns about the development to
stomach problems
naproxen
802- CPR:
2 breaths raise the chest,
you can DC shock 3 successive times,
cardiac massage in aortic stenosis
40%recovery

175
803- Lump in the bake with punktem not increase in size for year when
doctor press it the punctum came discharge yellow fuel smell
Remove to avoid rapture in derm??
Antibiotic first then remove

804- T score of 70 years old osteoprotic male is :


T-score of greater than minus-1 is considered normal.
A T-score of minus-1 to minus-2.5 is considered osteopenia, and a risk for developing
osteoporosis.
A T-score of less than minus-2.5 is diagnostic of osteoporosis
-3 (correct),
-2 ,
2,
3

▪ 805-retinal detachment all of the following are true


EXCEPT:
a) can lead to sudden loss of vision
b) more in far sighted than near sighted (correct)>>When looking at patients
who already have retinal detachments, you begin to see some interesting trends. Many of these
patients are myopic (near-sighted). Myopic eyes are physically larger and longer than normal
eyes and have thinner retina at the periphery this thin retina is more likely to break forming
small holes and tears that may progress to a detachment This thin retina is more likely to
break, forming small holes and tears that may progress to a detachment
c) follow cataract surgery
d) if you suspect it sent for ophthalmologist

806- A 40 year old man who become sweaty with palpitation before
giving a speech in public otherwise he does very good at his job, he is
having:
a) generalizes anxiety disorder
b) performance anxiety (correct)
c) agoraphobia
d) depression

807- A man is brought to the ER after having seizure for more than
30 min the most initial drug you will start with:
a) IV lorazepam (correct)>> initial treatment due to its relatively long (2–8 hour)
duration of action when injected, and its rapid onset of action, which is thought to be due to its
high affinity for GABA receptors and to its low lipidsolubility which causes it to remain in the
vascular compartment
b) IV phenobarbital

176
c) IV phynetoin
d) IV haloperidol

808- A women who lost her husband 2 weeks ago she is unable to sleep
at all you will give her:
a) floxitine
b) diazepam
c) halperidol
d) amytriptaline

_
a 65 yrs old lady came to your clinic with Hx of 5 days insomnia and
crying ( since her husband died ) the best Tx. For her is :

a- lorazipam
b- floxitein
c- chlorpromazine
d- haloperidol

809-17 year pt with dyspnea Po2 , PCO2 ,Xray normal PH increase so dd


is
- acute attack of asthma (correct)
-PE
- pneumonia
-pnemothrax

810- Picture of rash which appear pink on wood light:


-erythrasma>>is a macular brown area with few symptoms, most often found in the
armpits or groin. It is caused by overgrowth of diphtheroids of the normal skin flora. These areas
fluoresce coral PINK under long-wave ultraviolet radiation (Wood's light).”
Fungal infection
811-RTA pt with femur fx , he has laceration of the femoral artery .. What to
do : 1- end to end anastomosis
2- prosthetic graft
3-arterial graft
4- venous graft

_
812- Psycho pt swallow open safety pins,, x-ray show pins in the small
intestine,, what your next step:
177
-do emergent surgery(correct)>>Very sharp or pointed objects may perforate
the GI tract (sewing needles are notorious). Therefore, such objects should be
endoscopically removed from the stomach. If such an object has passed into the
intestines, early consultation with a surgeon is recommended. Objects that are too
long (eg, >6 cm) or too wide (eg, >2 cm) to pass through the pyloric sphincter
should be removed from the stomach.
-reassurance and discharge
-admitt and observe the pt
813-Theophylline interfere with absorption of:
-names of antibiotics
rifampin,ciprofloxacin,clarithromycin,erythromycin

?? 814- A child presented with respiratory distress & accessory muscle


contraction. What is your treatment?
a) Oxygen
b) Bronchodilator
c) IV corticosteroid
d) IV Theophylline
?? 815- A child presented with dysphagia, sore throat, postnasal drip,
drooling of saliva, rhonchi & fever of 38.50c. The treatment is:
a) Hydrocortisone injection immediately
b) Call otorhinolaryngology for intubation
c) Admit to ICU
d) Give antibiotics & send him home
816- A child was treated for otitis media with 3 different antibiotics for 6
weeks but without improvement. Which antibiotic is the best treatment:
a) Amoxicillin??
b) Penicillin
c) Cepahlosporin (ceprofloxacin)
d) Amoxicillin + Clavulonic acid>>
e) Erythromycin + sulfamethoxazol
818- A 70 year old female brought to your clinic by her daughter. The
daughter said her mother’s memory deteriorated in the last 2 years. She can
cook for her self but sometimes leave the oven on. She can dress herself
but with difficulties. The daughter mentioned that her mother’s personality
changed into a more aggressive person(pt has Alzehimer's disease).
According to this history what is your appropriate management?
a) Prescribe diazepam for the daughter and haloperidol for the mother
b) Refer the mother into chronic illness institute
c) Refer the mother to geriatric clinic (correct)
d) Immediate hospitalization

_
819- A man was intent as if he is listening to somebody, suddenly
started nodding & muttering. He is having: ????????
178
a) Hallucination (correct)>>is a perception in the absence of a stimulus
b) Delusion>>is a false belief held with absolute conviction despite superior
evidence
c) Illusion>>distortion of the senses, revealing how the brain normally
organizes and interprets sensory stimulation
d) Ideas of reference>>Ideas of reference and delusions of reference
involve people having a belief or perception that irrelevant, unrelated or
innocuous phenomena in the world refer to them directly or have special
personal significance
e) Depersonalization
820- A 45 yo lady was complaining of dizziness, sensory neural hearing loss
on her left ear (VIIIth nerve palsy), tingling sensation & numbness on her
face, loss of corneal reflex. MRI showed a dilated internal ear canal. The
diagnosis is:
a) Acoustic neuroma>>any unilateral sensorineural hearing loss is caused
by an acoustic neuroma until proven otherwise
http://emedicine.medscape.com/article/882876-overview#a0112
b) Glue ear
c) Drug toxicity
d) Herpes zoster
e) Cholesteatoma
821- A pt had hairline metatarsal fracture. The x-ray was normal. What is
the 2nd line?
a) CT scan
b) MRI(correct)>>MRI is sensitive for the diagnosis of fractures, it is
not required, because plain radiographic findings are fairly sensitive and
specific. MRI is useful in the assessment of fractures and dislocations,
soft tissue, the plantar plate, structures of the capsule, the extent of
marrow hyperemia, the exact number of bones involved, and small chip
fractures MRI is more sensitive than radiography and even scintigraphy
in the early diagnosis of stress fractures, because it shows bone marrow
edema exquisitely. MRI may be used to differentiate stress fractures
from early degenerative changes and early stress fractures from
synovitis
c) US
d) ..
822- A Case scenario about a male patient present with prostatitis
(prostatitis was not mentioned in the question), culture showed gram negative
rodes. The drug of choice is:
a) Ciprofloxacin (florqinlon)>>(correct)
b) Ceftriaxone
c) Erythromycin
d) Trimethoprime
e) Gentamicin
823- A pt complaining of left flank pain radiating to the groin, dysurea,
no fever. The diagnosis is:
179
a) Pyelonephritis >>It can cause high fever, pain on passing urine,
and abdominal pain that radiates along the flank towards the back.
There is often associated vomiting
b) Cystitis
c) Renal calculi >> (correct) The hallmark of stones that obstruct the
ureter or renal pelvis is excruciating intermittent pain that radiates from the
flank to the groin or to the genital area and inner thigh

824-A 10 yo boy presented with a 5 days history of skin lesion which


was scaly & yellowish. The diagnosis is:
a) Tenia corporum>> (ringworm,[1]tinea circinata,[2] and tinea
glabrosa[1]) is a superficial fungal infection (dermatophytosis) of the
arms and legs, especially on glabrous skin, however it may occur on
any part of the body.enlarging raised red rings with a central area of
healing (ringworm). The same appearances of ringworm may also
occur on the scalp (tinea capitis), beard area (tinea barbae) or the groin
(tinea cruris, known as jock itch or dhobi itch).
Other classic features of tinea corporis include:

 The edge of the rash appears elevated and is scaly to touch.

 Sometimes the skin surrounding the rash may be dry and flaky.

 Almost invariably, there will be hair loss in areas of the infection

825- A mother brought her baby & was complaining of diaper rash. She
used cornstarch, talc powder, zinc ointment & 3 different types of
corticosteroids prescribed by different physicians but with no benefit.
The rash was well demarcated & scaly with satellite lesions. The most
likely diagnosis:
a) Candidal rash (correct) >>fungal infection
b) Seborrhic dermatitis>>present with scaly, flaky,
itching red skin ,affect sebaceous gland-rich areas of skin
ttt:1/antifungal 2/anti-inflammatory 3/ topical steroid ..

c) Allergic contact dermatitis>>ttt : corticosteroid


826-A child presented with honey comb crust lesion. Culture showed
staph aureus. The diagnosis is:(539)
a) Impetigo (correct)
b) ..
c) ..
d) ..

180
827-. A pt presented with a 6 week history of itching & redness all over the
body with wheals. Which type of urticaria this pt has:
a) Chronic urticaria. (correct) >>Chronic urticaria (also known as
"Ordinary urticaria"[1]) is defined as the presence of evanescentwheals which
persist for 6 weeks or more
b) Solar urticaria
c) Allergic urtecaria
d) ..
828- A middle age man presented with sever headache after lefting heavy
object. His BP was high. He was fully conscious. Examination was
otherwise normal. The most likely diagnosis is:
a) Subarachnoid hemorrhage
b) Central HTN
c) Tension headache
d) Migraine
e) Intracerebral hemorrhage
829- You were working in a clinic with a consultant who prescribed a drug
that was contraindicated to the pt (the pt was allergic to that drug) but
you didn't interfere & assumed that he knows better than you do. Which
of the following you have violated:
a) Professional competence
b) Quality of caring of patient. (correct)
c) Honesty.
d) Pt relationship
e) Maintaining trust

_830- Physician's carelessness is known as:


a) Malpractice (correct)
b) Criminal neglect
)Malfeasance>> c‫ارتكاب عمل محضور‬
d) Nonfeasance >>‫التقصير‬

831-.The most important factor in attempt of successful cessation of


smoking is?
a) The smoker’s desire to stop smoking . (correct)
b) The pharmacological agents used in the smoking cessation program.
c) Frequent office visits.
d) Physician’s advice to stop smoking
e) Evidence of hazards of smoking
832- For health education programs to be successful all are true except :

a- human behavior must be well understood


b- Information should be from cultural background

181
c- Doctors are only the health educators(correct)>>health educator is “a
professionally prepared individual who serves in a variety of roles and is
specifically trained to use appropriate educational strategies and methods to
facilitate the development of policies, procedures, interventions, and systems
conducive to the health of individuals, groups, and communities
d- Methods include pictures and videos (mass media)
e- Involve society members at early stage
833- a 27 yrs. old female C/O abdominal pain initially periumbilical then moved to
Rt. Lower quadrant … she was C/O anorexia,nausea and vomiting as well ..
O/E : temp.38c , cough , tenderness in Rt lower quadrant but no rebound
tenderness.
Investigations : slight elevation of WBC's otherwise insignificant ..
The best way of management is:
a- go to home and come after 24 hours
b- admission and observation(correct)
c- further lab investigations
d- start wide spectrum antibiotic
e- paracetamol

what is the most likely diagnosis for the above patient ?

a- mesenteric lymph adenitits


b- acute appendicitis (correct)
c- peptic ulcer
834- a 24 yrs old pt. came for check up after a promiscuous relation 1 month ago .. he
was clinically unremarkable, VDRL : 1/128 … he was allergic 2 penicillin other line
of management is (dx syphilis ):

a- ampicillin
b- amoxicillin
c- trimethoprim
d- doxycyclin (correct) >>Doxycycline and tetracycline have been used
Tetracycline, erythromycin, and ceftriaxone[27] have shown antitreponemal activity in
clinical trials; however, they currently are recommended only as alternative
treatment regimens in patients allergic to penicillin

_
835-a 24 years old female pt. C/O : gray – greenish discharge , itching .. microscopic
examination of discharge showed : flagellated organism … most likely diagnosis is :

a- trichomoniasis ( trichomonas vaganalis )(correct)


182
836- a 43 yrs. old female pt. presented to ER with H/O : paralysis of both lower limbs
and parasthesia in both upper limbs since 2 hours ago .. she was seen lying on
stretcher & unable to move her lower limbs (neurologist was called but he couldn't
relate her clinical findings 2 any medical disease !!! ) when history was taken , she
was beaten by her husband … the most likely diagnosis is :

a- complicated anxiety disorder


b- somatization disorder
c- conversion disorder (correct)>>is a neurosis marked by the appearance
of physical symptoms such as partial loss of muscle function without physical cause
but in the presence of psychological conflict
d- psychogenic paralysis
e- hypochondriasis

the best treatment for the previous case is :

a- benzodiazepines
b- phenothiazine
c- monoamine oxidase inhibitor
d- selective serotonin reuptake inhibitor
e- supportive psychotherapy (correct)
837- a 58 yrs. old male pt. came with HX of fever, cough with purulent foul
smelling sputum and CXR showed : fluid filled cavity … the most likely diagnosis
is :

a.abscess>>Presence of air-fluid levels


b- TB
c- bronchieactesis >>

838- a patient ( known case of DM ) presented to u with diabetic foot ( infection) the
antibiotic combination is :

a- ciprofloxacin & metronedazole

_
839- a young pregnant lady (Primigravida) , 32 weeks of gestation came to you C/O :
lower limbs swelling for two weeks duration .. she went to another hospital and she
was prescribed ( thiazide & loop diuretic ) .. O/E : BP : 120/70 , mild edema , urine
dipstick : -ve and otherwise normal…. The best action is : normal

183
a continue thiazide & stop loop diuretic
b- cont. loop diuretic & stop thiazide
c- stop both
d- continue both and add potassium sparing diuretic
e- cont. both & add potassium supplement
840- a 17 yrs. old football player gave HX of Lt. knee giving off .. the most likely
diagnosis is :

a Lat. Menisceal injury


b- medial menisceal injury
c- lateral collateral ligament
d- medial collateral ligament
e ant. Curciate ligament (correct)

841- a 10 yrs. old boy presented to clinic with 3 weeks HX of limping that worsen
in the morning .. this suggests which of the following :

a- septic arthritis>>patient with rapid onset of joint pain


b-leg calve parthes disease>>Legg–Calvé–Perthes syndrome is characterized by
idiopathic avascular osteonecrosis of the capital femoral epiphysis of the femoral
head leading to an interruption of the blood supply of the head of the femur close to
the hip joint
Onset of pain may be up to 4 hours after inactivity. Knee pain is felt in the back of
the knee rather than in the front, not unlike a localized charley horse. This lasts for
an hour or so and returns nightly on inactivity
c- RA??>>
Morning stiffness and a limp that is worse in the morning suggest juvenile rheumatoid
arthritis
http://www.medscape.com/viewarticle/490135_4
d- a tumor
e- slipped capital femoral epiphysis >>
is a medical term referring to a fracture through the physis (the growth plate), which
results in slippage of the overlying epiphysis. Symptoms are waddling gait, loss of
motion in the hip joint, externally rotated foot, pain in the knee / groin / hip and
shortening of the hip. In up to 20% of cases slippage is bilateral. the knee starts to
get sore about 2-4 months before the actual hip goes. the pain in the knee can
come and go.
842-a 38 yrs old female … came to you at your office and her pap smear
report was unsatisfactory for evaluation .. the best action is :

184
a- consider it normal & D/C the pt.
b- Repeat it immediately
c- Repeat it as soon as possible
d- Repeat it after 6 months if considered low risk
e- Repeat it after 1 year if no risk
?? 843-- a 62 yrs. old female pt. a known case of osteoporosis & on 1
alpha + Ca supplement .. her lab works shows normal level of PO4, Ca &
ALP … her X-ray shows osteopenia with SD = -3.5 …. The best action is
to :

a- continue on same medications


b- start estrogen
c- start estrogen & progesterone
d-add alevdonate ( bisthmus phosphate)
844- a 17 yrs. old school boy was playing foot ball and he was kicked in
his Rt. eye .. few hours later he started to complain of : double vision &
echymoses around the eye .. the most likely Dx. Is :

a- cellulites
b- orbital bone fracture
(correct) c- global eye ball rupture
e- subconguctival hemorrhage
845- a 35 yrs old female pt. C/O : acute inflammation and pain in her Lt.
eye since 2 days .. she gave Hx of visual blurring and use of contact lens
as well … O/E : fluorescence stain shows dentritic ulcer at the center of
the cornea .. the most likely diagnosis is :

a- corneal abrasion
b- herpetic central ulcer >>Herpes virus cause a dendritic ulcer
c- central lens stress ulcer
d- acute episcleritis
e- acute angle closure glaucoma
846-a 25 yrs old Saudi man presented with Hx of mild icterus , otherwise
ok .. hepatitis screen : HBsAg +ve , HBeAg +ve , anti HBc Ag +ve (this
should be core anti body, because core antigen doesn’t leave
hepatocyte to the blood "prof. Yasawi" ) , the diagnosis :

185
a- acute hepatitis B?(correct)
b- convalescent stage of hep. B
c- recovery with seroconversion Hep . B
d- Hep B carrier
e- chronic active Hep. B
Serological test findings at different stages of HBV infection and in
convalescence

anti-HBc

Stage of infection HBsAg anti- IgG IgM HBeAg anti-


HBs HBe

late incubation period

+ - - - + or - -

acute hepatitis B or
persistent carrier state
+ - + + + -

HBsAg-negative acute
hepatitis B infection
- - - + - -

recovery with loss of


detectable anti-HBs
- - + - - -

healthy HBsAg carrier

+ - +++ + or - - +

chronic hepatitis B, persistent


carrier state
+ - +++ + or - + -

HBV infection in recent


past, convalescence
- ++ ++ + or - - +

186
HBV infection in distant past,
recovery
- + or - + or - - - -

recent HBV vaccination,


repeated exposure to antigen
without infection, or recovery
from infection with loss of - ++ - - - -
detectable anti-HBc

847-8 wk Primigravida came to you with nausea & vomiting choose the
statement that guide you to hyperemesis gravidarm :
a- ketonia (correct)
b- ECG evidence of hypokalemia
c- Metabolic acidosis
d- Elevated liver enzyme
e- Jaundice

?? 848-60 year old male was refer to you after stabilization investigation
show
Hgb 8,5 g/l , hect. 64% , RBC 7.8 , WBC 15.3
& Plt. 570 Diagnosis :
a- iron def. Anemia
b- Hgb pathy
c- CLL
d- 2ry polycythemia
e- Polycythemia rubra Vera

?? 849-Pregnant women G4P3+1 on GA 10 wk came to you with IUCD


inserted & the string is out from O.S what is the most important measure
:
a- leave the IUCD & give A.B
b- leave the IUCD & send to Ob/ Gynaecologist to remove
c- leave the IUCD
d- do laparoscopy to see if there is ectopic preg.
e- Reassurance the pt

187
850-30-Placenta previa excludes :
a- Pain less vaginal bleeding
b- Tone increased of uterus (correct)
c- Lower segmental abnormality
d- Early 3rd trimester

851-Pregnancy test +ve after :


a- one day post coital
b- 10 day after loss menstrual cycle (correct)>>qualitative hCG test .Doctors often
order these tests to confirm pregnancy as early as 10 days after a missed period
c- One wk after loss menstrual cycle

852-- 45 year old female complaining of itching in genitalia for certain


period, a febrile, -ve PMH, living happily with here husband since 20 year
ago on examination no abdominal tenderness , erythema on lower vagina ,
mild Gray discharge no hx of UTI . pyleonephritis
Most probable diagnosis:

a- Vaginitis (correct)
b- Cystitis
c- CA of vagina
d- Urithritis ( non gonococal )

Common Types of Vaginitis


Disorder TypicalSymptomsand Signs
CriteriaforDiagnosis MicroscopicFindings
DifferentialDiagnosis

Bacterial Gray, thin, fishy- Three of the Clue cells, Trichomonal

vaginosis smelling discharge, following: Gray decreased vaginitis

often with pruritus discharge, pH lactobacilli,

and irritation; no > 4.5, fishy odor, increased

dyspareunia and clue cells coccobacilli


Candidal vaginitis Thick, white Typical discharge, Budding yeast, Contact irritant
discharge; vaginal pH < 4.5, and pseudohyphae, or or allergic
and sometimes microscopic mycelia; best vulvitis Chemical
vulvar pruritus with findings* examined with irritation
or without burning, 10% K hydroxide Vulvodynia
irritation, or diluent
dyspareunia

188
Trichomonal Profuse, malodorous, Identification of Motile, flagellated Bacterial

vaginitis yellow-green causative protozoa, increased vaginosis

discharge; dysuria; organism by PMNs Inflammatory

dyspareunia; microscopy* vaginitis

erythema (occasionally by

culture)

Inflammatory
Purulent discharge, pH > 6, Increased PMNs, Erosive lichen
vaginitis
vaginal dryness and negative whiff parabasal cells, planus

thinning, test, and and cocci;

dyspareunia, dysuria; characteristic decreased bacilli

usually in microscopy

postmenopausal findings

women

*Culture is needed if microscopic findings are negative or symptoms persist.

853-20 year lady come to ER with Hx of Rt sever lower abdominal pain


with Hx of amenorrhea for about 6 wk the most serious diagnosis of your
deff. Diagnosis could reach by:

a- CBC
b- ESR
c- U/S of the pelvis (ectopic pregnancy) (correct)
d- Plain X-ray
e- Vaginal swab for C/S

_
854-Pt had arthritis in two large joint & pansystolic murmur ( carditis )
Hx of URTI the most important next step: (dx rheumatic fever)

a- ESR
b- ASO titre (correct but I'm not sure ) >>ASO is a test used to
detect streptococcal antibodies directed against streptococcal lysin O.
An elevated titer is proof of a previous streptococcal infection.
c- Blood culture?>>Blood cultures are obtained to help rule out infective
endocarditis, bacteremia, and disseminated gonococcal infection.

N.B :
Throat culture remains the criterion standard for confirmation of group A
streptococcal infection.
855-women complain of non fluctuated tender cyst for the vulva . came

189
pain in coitus & walking , diagnosed Bartholin cyst . what is the ttt:

190
a- incision & drainage
b- refer to the surgery to excision (after you reassure her)
c- reassurance the pt
d- give AB

856- 42years old male presented with history of sudden appearance of


rash – maculopapular rash – including the sole,& the palm, the most likely
diagnosis is :

a- syphilis
b- erethyma nodosum
c- erythema marginatum
d- pitryasis rocae
e- drug induced
857- years old lady on tricyclic antidepressent feels dizzy on standing,
resolves after 10-15 minutes on sitting, decrease on standing, most likely
she is having :
a- orthostatic hypotension>>Orthostatic hypotension, also known as
postural hypotension, is a form of hypotension in which a person's blood pressure
suddenly falls when the person stands up or stretches. The decrease is typically
greater than 20/10 mm Hg,[2] and may be most pronounced after resting. The
incidence increases with age.

858- what is the most appropriate treatment for the above patient :

a- antiemetic
b- antihistamine
c- change the antidepressant to SSRI
d- thiazide diuretics
e- audiometry

859- 23 years old lady with one month history of nasal discharge & nasal
obstruction, she complained of pain on the face, throbbing in nature ,
referred to the supraorbital area, worsen by head movement, walking,&
stopping. On - -- --------- examination , tender antrum with failure of
transillumination ( not clear ), the most likely the diagnosis is:

191
a- frontal sinusitis (we can NOT trannsiiluminate it)
b- maxillary sinusitis??
c- dental abscess
d- chronic atrophic rhinitis
e- chronic sinusitis
860- the cardiac arrest in children is uncommon but if occur it will be
due to primary
respiratory arrest (correct)
hypovolemic shock
neurogenic shock
861- Middle aged patient with an acyanotic congenital heart disease the X-ray
show ventrical enlargement and pulmonary hypertension:
a. VSD??
b. ASD
c. Trancus arteriosus>>cyanotic CHD
d. Pulmonary stenosis>>cyanotic CHD
862- role of surgery in. Stage C2 colon cancer
? Curative
Palliative
Diagnostic
Exploratory
Stage 0
Tis, N0, M0: The cancer is in the earliest stage. It has not grown beyond the inner layer (mucosa) of the colon or rectum. This
stage is also known as carcinoma in situ or intramucosal carcinoma.

Stage I
T1T2, N0, M0: The cancer has grown through the muscularis mucosa into the submucosa (T1) or it may also have grown into
the muscularis propria (T2). It has not spread to nearby lymph nodes or distant sites.

Stage IIA
T3, N0, M0: The cancer has grown into the outermost layers of the colon or rectum but has not gone through them (T3). It has
not reached nearby organs. It has not yet spread to the nearby lymph nodes or distant sites.

Stage IIB
T4a, N0, M0: The cancer has grown through the wall of the colon or rectum but has not grown into other nearby tissues or
organs (T4a). It has not yet spread to the nearby lymph nodes or distant sites.

Stage IIC
T4b, N0, M0: The cancer has grown through the wall of the colon or rectum and is attached to or has grown into other nearby
tissues or organs (T4b). It has not yet spread to the nearby lymph nodes or distant sites.

Stage IIIA
One of the following applies.

192
T1T2, N1, M0: The cancer has grown through the mucosa into the submucosa (T1) and it may also have grown into the
muscularis propria (T2). It has spread to 1 to 3 nearby lymph nodes (N1a/N1b) or into areas of fat near the lymph nodes but not
the nodes themselves (N1c). It has not spread to distant sites.

T1, N2a, M0: The cancer has grown through the mucosa into the submucosa (T1). It has spread to 4 to 6 nearby lymph nodes
(N2a). It has not spread to distant sites.

Stage IIIB
One of the following applies.

T3T4a, N1, M0: The cancer has grown into the outermost layers of the colon or rectum (T3) or through the visceral
peritoneum (T4a) but has not reached nearby organs. It has spread to 1 to 3 nearby lymph nodes (N1a/N1b) or into areas of fat
near the lymph nodes but not the nodes themselves (N1c). It has not spread to distant sites.

T2T3, N2a, M0: The cancer has grown into the muscularis propria (T2) or into the outermost layers of the colon or rectum
(T3). It has spread to 4 to 6 nearby lymph nodes (N2a). It has not spread to distant sites.

T1T2, N2b, M0: The cancer has grown through the mucosa into the submucosa (T1) or it may also have grown into the
muscularis propria (T2). It has spread to 7 or more nearby lymph nodes (N2b). It has not spread to distant sites.

Stage IIIC
One of the following applies.

T4a, N2a, M0: The cancer has grown through the wall of the colon or rectum (including the visceral peritoneum) but has not
reached nearby organs (T4a). It has spread to 4 to 6 nearby lymph nodes (N2a). It has not spread to distant sites.

T3T4a, N2b, M0: The cancer has grown into the outermost layers of the colon or rectum (T3) or through the visceral
peritoneum (T4a) but has not reached nearby organs. It has spread to 7 or more nearby lymph nodes (N2b). It has not spread to
distant sites.

T4b, N1N2, M0: The cancer has grown through the wall of the colon or rectum and is attached to or has grown into other
nearby tissues or organs (T4b). It has spread to at least one nearby lymph node or into areas of fat near the lymph nodes (N1 or
N2). It has not spread to distant sites.

Stage IVA
Any T, Any N, M1a: The cancer may or may not have grown through the wall of the colon or rectum, and it may or may not
have spread to nearby lymph nodes. It has spread to 1 distant organ (such as the liver or lung) or set of lymph nodes (M1a).

Stage IVB
Any T, Any N, M1b: The cancer may or may not have grown through the wall of the colon or rectum, and it may or may not
have spread to nearby lymph nodes. It has spread to more than 1 distant organ (such as the liver or lung) or set of lymph nodes,
or it has spread to distant parts of the peritoneum (the lining of the abdominal cavity) (M1b).

863- a patient with a large nodule in the nose which is painful


and talangectasia on the face you will give:
a) deoxycycline (correct)
b) clindamycin
c) retenoid

193
864-18 years old not sexually active came to your clinic complaining of
missed 2 period with sever abdominal pain on examination abdomen
can't examine because sever tenderness what you will do ?
Pregnancy test
Ultrasound
progesterone 100 mg for 10 days

865- Which of the follwing drug used in mycardial infaction


to prophylaxsis against arrythmia ?
Metoprlol
Adenosin
Atropin
Ca Channel blocker

_
866- In a day care center10 out of 50dovelop red eye. another
30 develop same condition in the next 2 week , what is the
attack rate
a) 40%
b) 60%
c) 80%
d) 20%
The term is defined as the number of exposed persons infected with the disease
divided by the total number of exposed persons
So 10+30/50 =80
867- Elderly patient who was smoking 2pack /day for 35 years
complaing of shortness of breath X ray done show plural effusion
plurocentesis show PH less than 7
What is the diagnosis ?
Cardiopulmonary edema
Empyema
Brochogenic carcinoma
868- most specific and sensitive Ix for renal
stone KUP
IVP
U/S
CT (correct)>>All stones are detectable on CT scans except very rare stones
composed of certain drug residues in the urinesuch as from indinavir.
869- Side effect of atropine :
Dryness of the mouth
Adverse reactions to atropine include ventricular fibrillation,
supraventricular or ventricular tachycardia, dizziness, nausea,
blurred vision, loss of balance, dilated pupils, photophobia, dry
mouth and potentially extreme confusion, dissociative
194
hallucinations and excitation especially amongst the elderly.
These latter effects are because atropine is able to cross the
blood-brain barrier. Because of the hallucinogenic
properties, some have used the drug recreationally, though
this is potentially dangerous and often unpleasant

870- diastolic" blowing" murmr best to heard in the left sternal


border increasing with squating imp.
AS
AR (correct)
MS
MR
MVP

Valve incompetence allows


backflow from aorta to ventricle.
Descriptio Caused by rheumatic heart
n disease, endocarditis, aortic
diseases (Marfan’s syndrome,
medial necrosis), syphilis,
ankylosing spondylitis, dissection,
cardiac trauma.

Heard with diaphragm, patient


Type and sitting and leaning forward; Austin-
Detection Flint murmur heard with bell;
ejection click heard in 2nd
intercostal space

Early diastolic, high pitch, blowing,


Findings often with diamond-shaped
on midsystolic murmur, sounds often
Examinati not prominent; duration varies with
on blood pressure; low-pitched
rumbling murmur at apex common
(Austin-Flint); early ejection click
sometimes present.
In left ventricular hypertrophy,
prominent prolonged apical
impulse down and to left Pulse
pressure wide; water-hammeror
biferiens pulse common in carotid,
brachial, and femoral arteries.
195
Heart
Sound S1 soft; S2 split may have tambour-
Componen like quality; M1 and A2 often
ts intensified; S3-S4 gallop is common.

871- the most lethal injury to the chest is


pneumothorax
rupture aorta (correct)>>Rupture of the thoracic aorta is the most lethal
injury following blunt chesttrauma
flail chest
cardiac contusion
872- lethal injury to the chest after motor accident:
puncture lung
spontaneous pneumothorax
rupture aorta (correct)
flail chest
all of the above

873- in acute abdomen the type of respiration is:


rapid and shallow (true)
rapid and deep
slow and shallow
slow and deep

874- pregnant female complaining of constipation what is the


managment :
tell her to to stop taking the iron supplements
give her fibers
give her bulking agents –
all of the above
Eat high-fiber foods such as whole-grain cereals and breads, brown rice, beans, and fresh fruits and vegetables
every day. Adding a couple of tablespoons of unprocessed wheat bran (available at health food stores) to your
cereal in the morning and following it with a glass of water can help, though it may take a few days before you
notice a difference.
 Drink plenty of water – at least six to eight glasses a day. A glass of fruit juice every day, especially
prune juice, can also be helpful. Some people find that drinking a warm liquid right after waking up helps
get things moving.
 Exercise regularly. Walking, swimming, riding a stationary bike, and yoga can all help ease
constipation and leave you feeling more fit and healthy.
 Your bowels are most likely to be active after meals, so make time to use the bathroom after you eat.
Listen to your body. Never put off going to the bathroom when you feel the urge.
 If your prenatal multivitamin contains a large dose of iron (and you're not anemic), ask your healthcare
provider about switching to a supplement with less iron.

196
 If the measures above don't help, talk to your caregiver about taking an over-the-counter
fiber supplement or stool softener.

_
875- patient has terminal ovarian carcinoma came to u complaining
of dull aching abdominal pain when u did xray u found a 10 cm
metalic clamp what will u do :
Call your lawyer for advice lol
call the surgeon for advice
no need to inform the patient since she is terminal and would not find
out about it
inform the patient and inform the surgeon and tell her it will dissolve
most likely in a ..... (certain abount of time dont remeber i think it was a
month ) (correct)
876- which one of the following Rx has lowset risk of tardive dyskinesia:
Clozapine >>clozapine has been shown to have a lower risk of tardive
dyskinesia than older antipsychotics
chloropromazine
haloperidol

877- family came to you complaining that their son sees humans as
(something... objects i think it was innate objects not sure ) and
plays alone and doesn’t play with other children and says "you"
when he wants to say "I"
which one of the following should not be done for the management of
this Patient:
narcoleptic medication
high ..... care program in school
mood stabilizers
878- patient male suddenly had bahevoral and cognitive imparment
and now sees a monkey in the room most likely Dx :
 schizophrenia
 Dementia like in parkinsonisim
 delrium
 depression

879- which of the following is correct about use of systemic retiniods :


*teratogenic(correct)

880- bupropion is contraindicated in which of the following :


197
*Hx of eating disorder ( Contraindicated in bulemia sure ) (correct)
bupropion should not be prescribed to individuals with epilepsy or other conditions
that lower the seizure threshold, such as alcohol or benzodiazepine withdrawal,
anorexia nervosa, bulimia, or active brain tumors.
881-8TB outbreak ..and one pt. come to doing tubercalin test and it's
negative .. what to do??

a- BCG
b- isonized
c- rifampin
882- baby with congugated hyperbilirubinemia:
Biliary atresia(correct)
ABO comp
G6PD
883- ld with URTI what is the most helpfully sign that it is
viral: Colorless nose discharge
Clinically significant rhinorrhea is more characteristic of a viral infection rather than
a bacterial infection. In viral URI, secretions often evolve from clear to opaque
white to green to yellow within 2-3 days of symptom onset. Thus, color and opacity
do not reliably distinguish viral from bacterial illness.

_
884- child with picture of SCA he should be maintained on
: Penicillin and folic acid (correct)
885- signs of androgen excess and ovarian mass , most likely tuner
: Sertoli-Leydig cell tumour (correct)

886- girl with hirsitism , deep voice , receding hair line :


Androgen excess (correct)

887- pregnant in the third trimester came with pain less vaginal
bleeding :
Placenta previa(correct)
888- picture of child with red rash on flexor surfaces
: Atopic dermatitis (correct )

Flexural involvement in childhood atopic dermatitis.

198
889- child with round palpable red rash on his right leg no pain or
itching for long time :
- granuloma annular
- tenia corpora
-erythema nodosum
-migratory

890- the goal of early management of inflammatory acne:


- to prevent physical scar
-to prevent spread of infection

891- prevention of malaria


Eradication of vector and protect against bites (correct)
http://www.netdoctor.co.uk/diseases/facts/malaria.htm

892- case of TB , what knd of injections u will give the contacts


: BCG
893- a picture of Snelling chart the q was how far should the
patient stand :
3m
6m (correct)
9m
894- pt with epistaxis , what is the most apropriate initial management :
Tampon ,
direct pressure on the soft part of the nose , (correct)
do nothing .
895- Sickle cell anemia patient non painful loss of vision in left eye,
on examination afferentflame sing and macula get??, what is the
cause?
- Retinal detachment
- Central retinal Artery occlusion
- Central retinal Vein occlusion
- ???
896- Patient with Obsessive compulsive disorder we need
to: A Decrease serotonin in the blood
B Increase Serotonin in the blood
C Increase sensitivity
D Decrease synthesis
897- Patient came to ER complain of pluratic chest pain and SOB. On
Examination the left lung is hyper resonant and no breath sound.
What is the diagnosis:
199
A Pulmonary effusion
B Pneumonia
C MI
D Cardiac Temponate
E Pneumothorax (correct)
898- Treatment of Pruritic
Folliculitis: A Topical Antibiotic
B Oral Antibiotic
C Oral steroid
Treatment
Pruritic folliculitis of pregnancy is typically treated like mild acne. Benzoyl peroxide
has been used with some success, but antibiotics are not needed. Oral antihistamines
are useful to treat the itching.
http://dermatology.about.com/cs/pregnancy/a/prurfoll.htm

899-51 year old male Hb7 MCV 112 AST 250 with Giant cells,
whats the diagnosis:
A B12 deficiency anemia
B Folic acid deficiency anemia
C Alcoholic anemia
D Thalasemia
E Iron deficiency Anemia
900- Patient was diagnosed to have Otitis media today you examined
the patient he is fine tempanic membrane is no longer erythematus but
there is collection of fluid behind it, whats your next step:
A Do Nothing
B Decongestion
C Antibiotics
901- Patient complaining of vetligo and he must hold something while
walking and sitting down, This is found in 75% of patients taking :
A INH side effect
B Penicillin side effect
C Streptomycin side effect
(correct) D Allergy to penicillin
E One more drug side effect

902-9year old boy cam to PHC with URTI and swap was taken and sent
home, after 5 days the result was Group A streptococcus and then you
called the family and they told you the boy is fine and no symptoms
whats you next step:
A Give Ceftixim IM one
dose B Penicillin for 7 days
C Penicillin for 10 Days

200
D Do Nothing
903- After start ttt of depression for Pt. who show improvement, the risk of suicide :
 Increase
 Decrease (correct)
 The same after and before ttt
 Will not suicide at all
904- the symptom/sign that comes 2ry rather than presented symptom in panic pt.
 tachycardia
 epigastric pain
 chest pain
 phobia
Mechanism of Action of drugs that inhibit Conversion of estriol
905-
to estrogen
.
(I forgot the exactquestion but it mentioned about ovulation and who inhibits
conversion of esterone toestrogen?) Options were:
aAromatase inhibitors

synthesis of estrogens starts in theca interna cells in the ovary, by the synthesis of
androstenedione from cholesterol. Androstenedione is a substance of moderate
androgenic activity. This compound crosses the basal membrane into the
surrounding granulosa cells, where it is converted to oestrone or oestradiol, either
immediately or through testosterone. The conversion of testosterone to oestradiol,
and of androstenedione to oestrone, is catalyzed by the enzyme aromatase.
Oestradiol levels vary through the menstrual cycle, with levels highest just before ovulation.

906- year old female patient of Cushings syndrome, had hip fracture
falling off stool, what will you screen for while also treating her fracture:
Hyperparathyroidism
Osteomyelitis
Osteoporosis
Osteomalacia
907- Drug table given each with3-4drugs, question which group
causes hyperuricemia.
ABCD
(I choose the group which had anti-neoplastics.)
The principal drugs that contribute to hyperuricemia by decreased excretion are the
primary antiuricosurics. Other drugs and agents include diuretics, salicylates,
pyrazinamide, ethambutol, nicotinic acid, ciclosporin, 2-ethylamino-1,3,4-thiadiazole,
and cytotoxic agents

201
908- Child 3 weeks ago had chicken pox, came to ER c/o sob, x-
ray shows enlarged epiglottis, what is the cause?
- Hemophlus influ, type b
- Diphtheria pertusis
- Rubella
- Measles
909-2 years old child fall down in homein x-ray there is spiral # in radial bone the
best management :
-call pediatric.
-call orthopedic.
-splint.
-open for fixation.
910- female married since four month and she noticed her husband washing his hand
several times the most likely diagnosis:
obsessive compulsive disorder
911- pt. admitted with pneumonia and the vaccine for this pt. is:
-pneumonia with H.influnza
-influenza alone.
-pneumonia alone.
912- pt. have peptic mass the most common organism for infection:
-H pylori
-HPV
-HIV
913- pt have swelling in lower eye lid and the lid was erythemic
and edemoutis with hair inside make corneal ulcer the dx:
-entropic.
-extropic
914- evidence based medicine:
-as in text book.
-according to departmental policy.
-according to latest published articles.
-according to strong scientific evidence.
915-16 wk GA with ++glycosuria FBS:4.4, 1Hr PP 8.2 , 2Hr PP 7.2:
-renal glycosuria??
-GDM.
-K.M syndrome
916- what is the best source of iron in a 3 month old infant?
-breast milk.
-low fat cow milk.
-yellow vegetables.
-fruit.
-iron fortified.

202
917- #pt with bilateral infiltration in lower lobe (pneumonia )which
organism is suspected :
-ligonella
-klibsella
-strept pnemoniae
- staph
-pnemococcus pnemonia

918- 33 year old make C/O of pain in his lip and right check . Pain
was stabbing like triggered with touch. O/E, cranial nerves were intact
, The best Rx Is:
a- Oxcrazepine
b- Propanol
c- Ergotamine
d- Lithium

919- Avoid with abnormal ALT >>(read about hyperlipidemia drug)


?!! HMG coA
920- isolated closed Fracture femur treatment
is: a- internal fixation with plate
b- Internal fixation with antegrade intramedullary locked nail (
Sabiston 17th ed page 563 )??
c- External fixation with intramedullary locked nail
d- traction & balance
e- cast around hip
The method of fixation of femoral shaft fractures has become fairly standardized. The treatment of
choice for closed fractures and types I through IIIA
open fractures is closed, locked IM nailing. In contrast to open reduction methods, this practice
reduces bleeding and soft tissue disruption at the
fracture site. These minimally invasive techniques reduce perioperative stress and decrease the
incidence of infection and nonunion.

921-27 yr old lady primi 35 wks pregnant, presented with mild Pre-eclampsia , BP
140/? Edema in her hands & feet, best treatment is:
f- Immediate delivery
g- Diuretics
h- Send home?
i- Hospitalize & materno-fetal monitoring ( this is the most likely
answer
Johns hopkins manual of gynecology and obstetrics 3rd ed Pages 183-184
Mild pre-eclampsia
i. If the pregnancy is >37 weeks, then the patient should be
managed as follows:
203
1. If cervix is favorable (or patient is noncompliant), then
proceed to induction of labor.
2. If cervix is unfavorable (bishops score <6), then
close maternal and fetal observation is essential.
a. Fetal—Ultrasound for growth/AFI, and, if reassuring,
no further sonography is required barring a
progression in symptoms or signs of poor fetal growth
or distress; NST/BPP 1 to 2 times per week.
b. Maternal—Weekly evaluation and laboratory
testing, as described earlier. Weekly cervical
exam—when favorable proceed to induction of
labor. Despite an unfavorable cervix, induction of
labor should be initiated by 40 weeks' gestation.
ii. Similarly, if the gestational age is >34 weeks and is
accompanied by progressive labor, preterm premature rupture of
membranes, abnormal fetal testing, or growth restriction, the
patient should also be delivered.
iii. Outpatient management. Some compliant patients with mild pre-
eclampsia (BP <150/100, 24-hr urine protein <1 g, no
symptoms, normal liver transaminases and platelet count) can
be followed from home.
1. Outpatient management with decreased physical
activity. Strict bed rest is not recommended (3).
P.183

2. No antihypertensives—not shown to improve perinatal


outcome (14)
3. Monitoring for symptoms of pre-eclampsia; daily kick counts
4. At least, weekly blood pressure checks and physical
examinations that evaluate for signs of worsening
pre- eclampsia
5. Nonstress test/biophysical profile 1 to 2 times per
week; ultrasound for growth/AFI every 3 to 4 weeks
6. Laboratory testing: baseline 24-hour urine protein
collection and weekly laboratory testing for platelet count,
liver enzymes, and serum creatinine
iv. Inpatient management. Management of these patients is
essentially the same, but monitoring is more intensive by hospital
personnel.
922-Not use in the prevention of preeclampsia with + protein urea & LL edema
: j- Admission &bed rest
k- Diuretics ( if you are understanding the underlying physiology you will
exclude this answer )
l- Non-stress test
m- Regular sonogram of baby
923-

204
40 yr old male with 4 days history of sudden eruption over the entire body
including palms & feet :
n- erythema nodosum
o- erythema multiforme
p- pit. rosea
There is no enough information to diagnose the case as whether it is itchy or not
but generally speaking it is either Pityriasis rosea ( more likely) or Sec. Syphilis
but it is rare now and in 75% it doesn’t involve face

_
925-urticaria, all true EXCEPT:
q- can be part of anaphylactic reaction
r- is not always due to immune reaction
s- always due to deposition of immune complex in the skin ( due to
increase permeability of capillaries )
t- due to ingestion of drug
u- due to ingestion of strawberry
926-10 yr old boy woke up at night with lower abdominal pain, important area to
check:
v- kidney
w- lumbar
x- rectum
y- testis (the Q is not clear for me but it seems to be related to
testicular torsion or something in inguinoscrotal area
927-All are true about hoarsness in adult , EXCEPT :
z- due to incomplete opposition of the vocal
cord aa- if > 3 weeks : need laryngoscopy
ab- if due to overuse, advise to whisper a few weeks
ac- commonly seen in bronchus Ca
ad- feature of myxedema
I can't judge

_
928- years old boy for evaluation of short stature. His height is of 6 year old &
bone scan of 5.5 years ,, Dx is:
ae- steroid therapy
af- genetic
ag- constitutional
ah- hypochondroplasia (not sure)

205
ai- hypothyroidism ( not mentioned in our exam)

929- Diagnosis of Alzheimer confirmed by:


CT brain
EEG
Neurological examination

None of the above(investigation is aimed at excluding other treatable


causes of dementia, as histological confirmation of Dx usually occurs only
after death Davidson's 19th 1173
930-12 yr old girl with malaise, fatigue, sore throat & fever. On examination:
petechial rash on palate, large tonsils with follicles, cervical lymphadenopathy &
hepatosplenomegaly. All are complications EXCEPT:
Aplastic anemia
Encephalitis
Transverse myelitis
Splenic rupture
Chronic active hepatitis

931-1 month old with massive hepatosplenomegaly, bluish skin nodules, & lateral
neck swelling, the next step is:
CBC
lumber puncture
Do EBV serology
BM scan ( It seems to be congenital Leukemia)
Liver biopsy
932-8 month old baby came with dehydration, fever 40 C, poor feeding &
convulsions. depressed ant. Fontanelle, vomiting, & crying with red ears. No neck
stiffness. Her 3 yr old brother is asymptomatic. What is the most important
investigation to do:
Blood culture
CBC & differential
CSF examination ( Our Prof. told us that british says if meningitis crosses
your mind do LP ) provided that C/P is not specific in this age
Chest xray
Urine analysis
933-17 year old boy presented to the ER complaining of sudden onset of
abdominal pain & leg cramps, he had history of vomiting2 days ago, he was
dehydrated .
206
Na = 150 , K = 5.4 ,, glucose = 23mmol
The best initial investigation is
CBC
Blood culture
ABG ( tha Dx is DKA)
Urinanalysis (dipstick)
U/S Iremember that this option was Serum amylase

_
934-young age male presented after RTA with injured membranous urethra ,
best initial ttt is :
Passage of transurethral catheter
Suprapubic catheter
Perineal repair
Retropubic repair
Transabdominal repair
935-young male presented to ER with a stab wound in his abdomen , u should:
should Explore the abdomen
observe patient & not explore if vitals remain stable
Exploration depend on U/s
Exploration depend on DPL
Exploration peritoneum penetrated

936-Contraindication of gastric lavage if the ingested material is:


Aspirin
diazepam
Dry clean (clorex) Corrosive
Castor beans
Vit D
937-65 year male presented with 10 days Hx of hemiplagia , CT shows :
infarction , he has HTN. He is on lisonipril & thiazide , 2 yr back he had gastric
ulcer . ttt that U should add :
continue same meds
Aspirin 325
aspirin 81
warfarin
dipyridamole ( Antiplatlet agent)

207
938-All of the fallowing are criteria of subarachnoid hemorrhage EXCEPT:
Paraplegia ( Davidson's 19th 1162)
confusion
nuchal Rigidity
Due to berry aneurysm rupture
Acute severe headache

939-After infarction , the patient become disinhibeted , angree & restless . The
area responsible which is affected:
premotor area
temporal area
pre- frontal area ( Davidson's 19th 1148)

940-All are true about the best position in hearing the murmurs, EXCEPT:
supine : venous hum
sitting : AR
sitting : pericardial rub ( opinion : according to my knowledge no special
position for it to be accentuated)
supine : innocent outflow obstruction
Lt lateral in : MS
941-All are true about the best position in hearing the murmurs, EXCEPT:
supine : venous hum
sitting : AR
sitting : pericardial rub ( opinion : according to my knowledge no special
position for it to be accentuated)
supine : innocent outflow obstruction
Lt lateral in : MS

942-3 year old child needs oral surgery & comes to your clinic for checkup. On
examination 2/6 continuous murmur , in upper Rt sternal borders that disappear
with sitting , next step:
Give AB prophylaxis
Ask cardiology consult
Clear for surgery
Do ECG

208
943-Glue ear
Managed by grommet tube ( Nelsson 19th 1955)
Lead to sensorineural hearing loss
Pus in middle ear
Invariably due to adenoid
944-12 months baby can do all except:
Walk with support one hand
Can catch with pincer grasp
Can open drawers
Response to calling his name
Can play simple ball
945-max dose of ibuprofen for adult is :
800 if they mean by dose
1600
3000
3200 per day

946-Regarding SEM (standard error of the mean):


SEM is observation around the mean
Standard deviation is measure of reliability of SEM
Is bigger than SD
Is square root of variance
Standard deviation advantage can be math manipulated
standarderror is as a measureof the precision of the sample mean.
947- Indications of surgery in crohn’s disease
internal fistula
external fistula
intestinal obstruction
Stagnant bowel syndrome

948-45 yr old lady presents with nipple discharge that contains blood. What is
the MOST likely Dx:
ductal papilloma
ducta ectasia
fibroadenoma
duct CA
949-after aspiration of cystic mass in the breast the result was clear fluid, next
step
a-Send the aspirated content for cytology and if abnormal do mastectomy
209
b-Reassure the patient that this lump is a cyst and reassess her in 4
weeks c-Book the patient for mastectomy as this cyst may change to
cancer.
d-Put the patient on contraceptive pills and send her home
950-after 2 wks ant. wall MI , old age female developed sudden leg pain , it is
pale & pulsless. Dx :
acute arterial thrombus
acute arterial embolus
DVT
Ruptured disc at L4-5 with radiating pain
Dissecting thoraco-abdominal aneurysm

_
951-a 34 yr old divorced lady complains of 15 months amnorrhea , FSH very high ,
Dx :
Pregnancy
ovulation
Premature ovarian failure
Hypothalamic lesion
Pituatary microadenoma
952-he developed severe pain over the wound site , with foul smelling discharge ,
his temp is 39 & HR is 130/min . Gram stain showed G+ve rodes with terminal
spores , ttt
Massive IV pencillin V
clostridium antitoxin
wide surgical debridement
chlormphinicol
wide surgical debridment & Massive pencillin V
953-ectopic pregnancy, all true EXCEPT:
20% ovarian
doubling HCG useful clinical tool
empty uterus + HCG before 12 wks is Dx
laparascopy can dx it

_
954-a 28 yr lady with 7 week history of amnorrhea has lower abdominal pain ,
home pregnancy test was +ve , comes with light bleeding, next step:
Check progesterone
HCG
210
Placenta lactogen

211
Estrogen
Prolactin

_
955- All causes hyperprolactenemia,
EXCEPT: pregnancy
acromegaly
methyldopa
allopurinol
Hypothyroidism

956- complication of long term use of


steroid: asthma
Breast CA
Other Ca
myopathy in pelvic girdle
osteomalacia
957- All are complications of long term use of phenytoin, EXCEPT:
Ataxia
osteoporosis
Osteomalacia
Macrocytosis

958- physiological cause of hypoxemia


hypoventilation
improper alveolar diffusion
perfusion problem(ventilation-perfusion mismatch)
elevated 2.3 DPG>>increase o2 supply in the blood.
959- A 15 yr old boy came to your clinic for check up. He is asymptomatic. His
CBC showed: Hb 118 g/l WBC 6.8 RBC 6.3 (high) MCV 69 (low) MCH (low) Retic 1.2
(1-3)% what is the most likely diagnosis?
Iron deficiency anemia
Anemia due to chronic illness
β-thalssemia trait
Sickle cell disease
Folic acid deficiency

212
960- Hb electrophersis done for a patient shows HbA1=58% , HbS = 35% , HbA2
= 2% , HbF = 5 % , Dx :
Thalasemia minor
Thalasemia major
Sickle cell trait
Sickle cell anemia
Sickle cell thal.
961- first sign of LSHF
orthopnea
dyspnea on exertion
pedal edema
PND
chest pain

962- all can cause congenital infection IUGR,


EXCEPT: Rubella
CMV
Syphilis
HSV II
Toxoplasmosis

963-75 yr old female with 2 days hx of MI is complaining of abdominal pain ,


vomiting , bloody stool . X-ray shows abd distension with no fluid level , serum
amylase is elevated. Dx :
Ulcerative colitis
acute pancreatitis
Ischemic colitis
Diverticulitis

_
964-CCB drugs like verapmil , dilitazem, nifedipine are effective EXCEPT:
Prinzmetal angina
Hypertension
Atrial tachycardia
Ventricular tachycardia
Effort angina

213
965-a 5 day old child vomited blood twice over the last 4 hr , he is healthy ,
active & feeding well by breast , Dx :
esophigitis
esophageal varices
gastritis
duodenal ulcer
cracked maternal nipple

966-5 yr old seen in ER presented with fever & sore throat , which of the
fallowing suggest viral etiology :
Presence of thin membrane over the tonsils
Palpable tender cervical LN
Petechial rash over hard or soft palate
absence of cough
Rhinorrhea of colourless secretion

967-The following are complicationsof laproscopic cholecystectomy EXCEPT:


Bile leak
Persistent pneumoperitonium
shoulder tip pain
ascites
Supraumbilical incisional hernia

_
968-one of the fallowing drug combination should be avoided :(the same in
alqaseem but all choises are here)
Cephaloridine & paracetamol
Penicillin & probenicid
Digoxin & levadopa
sulphamethaxazole & trimethoprim
tetracycline & aluminum hydroxide
969-40 yr old male presented to ER with 6 hr hx of severe epigastric pain,
radiating to the back like a band , associated with nausea . No vomiting , diarrhea.
No fever . On examination he was in severe pain & epigastric tenderness. ECG
was normal, serum amylase was 900 u/l, AST & ALT elevated double the normal.
Which of the following is the LEAST likely precipitating factor for this patient:
Hypercalcemia
chronic active hepatitis
214
chronic alcohol ingestion
hyperlipidemia
cholelithesis

970-Which of the following not transmitted by mosquitoes


Rift valley fever
Yellow fever
Relapsing fever (caused by ticks)
Filariasis
Dengue fever

971-A on-opaque renal pelvis filling defect seen with IVP , US revels dense
echoes & acoustic shadowing , The MOST likely Dx:
blood clot
tumor
sloughed renal papilla
uric acid stone
crossing vessels
972-coarctation of the aorta in commonly associated with which of
the following syndromes?
A)down
b)turner
c)patau
d)Edward
973-46 yr old female presented for the third BP reading, high blood pressure
160/100 . she is not on any medication. Lab investigation showed
Urea: normal
Creatinine: normal
Na=145 (135-145)
K= 3.2 (3.5 – 5.1)
HCO3= 30 (22-28)
What is the Dx?
Essential hypertension
Pheochromocytoma
Addison’s Disease
Primary Hyperaldosteronism

_
974-32 yr old lady works as a file clerk developed sudden onset of lower back
pain when she was bending to pick up files, moderately severe for 3 days

215
duration. There is no evidence of nerve root compression. What is the proper
acrion:
Bedrest 7-10 days
Narcotic analgesia
Early activity with return to work immediately
CT for lumbosacral vertebra
975- fracture of rib can cause all
except: pneumothorax
hemothorax
esophageal injury
liver injury
976- anal fissure more than 10 days, which is
true: Loss bowel motion
Conservative management
Site of it at 12:00

977- after delivery start breast feeding :-


as soon as possible
8 hrs
24 hrs
36 hrs
48 hrs
978- All are true for the prescripsion of antidepresents ttt for patient with
depression & somatisation disorders, EXCEPT:
Smaller doses may needed in elderly
potential side effect should not explain 2 the patient , b/c he will develop
it
fluoxetine safe for elderly

_
979- the most specific investigation to detect pulmonary embolism is
: perfusion scan
pul angiogram
ventilation scan
CXR

_
980- obstructed labor, which is true:
common in primi
216
excessive caput & molding are common signs
most common occipto- ant
can not be expected before labor

981-45 year old female come to the ER complaining of Rt hypochondrial pain


which increases with respiration , on Ex there is tenderness over the Rt
hypochondrium,
Next investigation is
X-ray
US of upper abdomen
CT

982- a 48 hour old newborn infant in critical care unit with respiratory
distress and jaundice.HB 9g/dl, retic 4%,. Maternal Hx of previous
normal term pregnancy without transfusion,blood typing shows hetero
specifity between mother and child.Indirect Coombs test +ve.the most
probable Dx is
a- Sickle cell disease
b- Thalassemia
c- Maternal – fetal blood
mismatch d- Hereditary genetic
disease

e-septicemia.

983-a disease lasts 2-3 wk with fatality rate 30%:


a-incidence=prevalence.
b- incidence >prevalence.
c- incidence<prevalence.
d- incidence=1/2prevalence.
e-has no relation.

_
984-what is the least effective AB of the following to staph.
Aureus: a-clindamycin.
b-erythromycin.
c-amoxicillin.(over 80% of staph aureus are resistant to
penicillin) d-vancomycin.
985-35 y/o presented with left iliac pain and dysuria, mangment include
all the following:
a-blood C+S
b-microscopy of
urine. c-IVP.
217
d-urine C+S

218
e-norfloxacin.
986-Colle’s fracture:
a-distal end of the
radius. b-scaphoid
fracture.
c-around the elbow.
d-head of the
radius.
987-a child fell on an out-stretched hand and flexed elbow,exam showed
swelling around the elbow with no radial pulse, best management :
a-closed reduction.
b-closed reduction then check radial pulse.
c-open reduction.(because of the vessel involvement best way by open
repair)
d-cuff and collar for 3wks.
988-most common association with acanthosis
nigricans: a-hodgkin lymphoma
b-non-hodgkin lymphoma.
c-Internal malignancy.
d- DM
e- insulin resistance.

989-xanthoma:
a-on lateral aspect of the upper
eyelid. b-hard plaque.
c-around arterioles.
d-is not related to hyperlipidemia.
e-deposited in dermis.

990-patient suspected to have brain abscess,the most important q. in


the history is :
a-frontal sinusitis. (contiguous suppurative focus (45-
50%) b-ear discharge.
c-head injury.
d-bronchioctasis.
e-Hx. of vomiting.
991-the following are true about H. pylori
except: a-related to gastric outlet incontinence.
b-can cause gastritis but not related to duodenal
ulcer. c-can be eradicated by ampicillin and
metronidazole.
d-there will be histological improvement after eradication.
e-it can split urea.

219
992-60 Y/O lady on OCP 21 days a month having recurrent vaginal
bleeding(spotting) after the stop of estrogen, best Tx:
a-endometrial Bx.

220
b-papsmear of the
cervix. c-add
progestone .
d-stop estrogen.
e-abdominal US or laproscope.
993-most common site of gonococcus infection in females
in: a-cervix.
b-posterior fornix.
c-urethra.
994-post D&C the most common site of perforation is
the: a-fundus
b-ant.wall of the corpus
c-post. Wall of the
corpus. d-lat.wall of the
corpus.
e-cervix.
995-regarding typhoid fever, all are true except:
a-fever and red spots appear on the same time .(fever first then
rash) b-can be completely eradicated even in the prescence of gall
stones. c-transmitted by food ,milk and water.
d-can be treated by quinolones.
996-high output HF causes include all except:
a-anemia.
b-MR.
c-AV fistula.
d-pagets disease.
997-a middle aged man having black spots on his thigh for years, it is
starting to become more black with bloody discharge,the best
management is to:
a-wide excision.(malignant melanoma ttt)
b-incisional Bx.
c-cryotherapy.
d-radiotherapy.
e-immunotherapy.

998- Old patient .. stopped smoking since 10 years … suffering


from shortness of breath after exercise but no cough … and
there was a table
Fev1=71%
Fvc=61%
FEV1/fvc=95%
Tlc=58%
What's the dx?

221
a- Restrictive lung
disease b- Asthma

222
c- Bronchitis
d- Emphysema
e- Obstructive with restrictive

999-its c/I to stop preterm delivery in the following condition:


a-aminochoronitis.
b-placental abruption.
c-preeclampsia.
d-A&B.

1000-PPH happens more commonly with:


a-multiple pregnancies.(due to increased risk of uterine atony)
b-anemia.
c-preterm delivery.
d-antithrombin iii deficiency.
1001-Before you start instrumental delivery it is important to check if there is :
a-face presentation.
b-CPD
c-breech presentation.
d-cord prolapse.
1002-in occipitoposterior malpositioning of the fetal head ,all of the following are true
except:
a- 0% of all vertex deliveries.
b- it causes significant delay of labor duration compared to the anterior presentation.
c-andriod pelvis is a predisposing factor.
d-flexion of the head helps the rotation to the ant. Position.
1003-important tools for listening to a patient include:
a-using tools for asking.
b-imagination.
c-using similar words and expressions as the patient.
d-a sense of humor.
e-all of the above.

_
1004-the mechanism of action of heparin:
a-activation of antithrombin iii

_
1005-Cover one eye onother go laterally?
Strabismus
Ambylobia
3rd nerve palsy

223
1006-a 6 year old girl presented with low grade fever and arthralgia for
5 days. She had difficulty in swallowing associated with fever 3 weeks
prior to presentation.physical examination revealed a heart rate of
150/min and pansystolic murmur at the apex. There was no gallop and
liver was 1 cm below costal margin. The most likely diagnosis is:
a-bacterial endocarditis.
b-viral myocarditis.
c-acute rheumatic
fever. d-pericarditis.
e-congenital heart failure.

1007- What is the most common congenital hear disease associated


with rheumatic heart disease ?
Asd
Vsd
Coacotation of aorta

1008- Child with URTI then complained from ear pain on examination
there is hyperemia of TM &+ve insufflations test he tri 2 drug no benefit
what is the best TTT>>>
ugmentine
azithromycin
ciprofloxacin
steroid??

1009-Our target hb A1C >> in DM

10

1010-Check pt after
RTA; LOC

1011-3m baby with hx of broncholities >>


224
1012- - Coronary artery disease atherosclerosis inside
artery or lumen ?!!

1013- Heart receive blood through >>


aorta constrict
IVC dilate
Increase pul resistance

225
1013- Pt fall from stairs >> no air
entry Mange by ;
ENdotracheal entbation
, oxygen ,

1014-Most common cause of otitis media all age


>> staph
strept . pneumonia

1015-child with proptosis , red eye , restrict eye movement , normal


examination:
Orbital cellulitis

1016- Psychosis postpartum ?!!


insidious onset
common
usually suicide

1017- HTN with hyperaldosternism ?!!


spironlCtone

1018- Patient admitted as a case of emphysema, according to the


vaccine what you will do
a) give pneumococcal vaccine now
b) give flu vaccine now
c) give all vaccine 2week after discharge
d) give flu vaccine now and pneumococcal vaccine 4week after discharge

226
1019- what is most sensitive for DX of duodenal ulcer
: a- Epigasteric pain starting 30-60 min after the
meal
b- Epigasteric pain staring immediately after a meal
c- Increasing of pain when lying supine
d- Pain radiating to the back
1020- The greatest method to prevent the diseases :
a- Immunization
b- Genetic counsling
c- Enviroment modefication
d- Try to change behavior of people toward health??
e- Screenig
1021-/least effect on tardia dyskinesia :
a. halipridol
b.respridol
clozepine

1022-pt received varicella vaccine after 30 min he developed itching …..


treatment is :
Subcutenous epinephrine
1023-case scenario pt with HZV ttt :
a. acyclovir for 3-5 days
b. acyclovir and refer for ophthalmology

1024- pt with hirshitism , obese , x ray showed ovary cyst she wants to
convieve best treatment :
Clomophine citrate.
1025- adolescent with asymptomatic hernia :
a.surgical is better than medical ttt
b.contraindication to do surgery in reducible hernia
c.can cause hypoinfertilty
1026- pt use antacid ,complain of vomiting and pain due to :
a. GERD
1027- scenario about ectopic pregnancy B-HCG 5000 hemodynamically
stable ttt:
a.observation
b.medical.
c.laproscopy
d.laprotomy.

1028-effect of niacin is :
227
a.decrease uric acid .
b.hypoglycemia
c.increase LDL
d.increase HDL
e.increaase triglyceride

1030-pt with DM and obese ,plane to reduce his wt is :


a.decrease calori intake in day time
b.decrease calori and increase fat
c.decrease by 500 kcal/kg per week
d.decrease 800 per day

1031-Romberg sign lesion in :


a. dorsal column
b. cerebellum
c. visual cortex
1032-ttt to increase fetal Hb in sickle cell disease :
Hydroxurea
1033-angioedema due to use of :
B blocker
ACEI

1034-fetal distrees in :
a.early deceleration
b.late deceleration

1035- Pregnant Teacher in her 20th week reported 2 of her student


developed meningitis. Prophylactic Treatment:

a) Observe for the sign of meningitis


b) Meningitis Polysaccharide vaccine
c) Ceftriaxone 500mg PO once
d) Cefuroxime 250 mg IM or IV
once e)Rifampicine 600 mg BD for
2 days

1036- treatment of erosive gastritis


? a-Antibiotics
b- H2 blocker
c- depend on the pt situation
d- total gastroectomy
e- sucralfate

228
1037-Patient 2 h after delivery have sever vaginal bleeding initial
management ?
Ergometrin
Ringerlactate solution
Blood transfusion

1038- Patient with digoxin toxcicty the most important to give ?


Immune fab
k
1039- female pt , with RTA ,she has bilateral femur fracture >>>like this
scenarion , systolic blood pressure 70 >>>what will you do:
a- Iv fluid
b- blood tranfusion
c-
1040- Long hx of pt with recurrent vomiting for 2 days
Heamocrit 65 the doctor can report this result caused by
Cytokine
Glucagon
C r protin
Apoprotein

1041-secondary amenorrhea
a-due to gonadal agenesis
b-sheehan's syndrome
c- It is always pathological

1042- pt with DM and obese ,plane to reduce his wt is :


a.decrease calori intake in day time
b.decrease calori and increase fat
c.decrease by 500 kcal/kg per week
d.decrease 800 per day

1043-chickpeas.kidney beans and lentils contain which element of


following
bromide
chromium
iron

selenium

229
1044- what food causes bleeding in a patient on anticoagulates
garlic
spinach
avacados
ginko

1045- Patient on asprin, phenyton for sizuers came to clinic for rotein
follow up, on examination she has bilateral painless lymph nodes, no
other symptoms or signs, lymph node biopsy showed hyperplasia.
DDx: A- chronic lymphocytic leukemia.
B_hodgkin lymphoma
C- TB
D-???
most likely it is side effect of phenytoin

1046- Pt complaining of hypotension & bradycardia. Electrolytes show:


dec.Na, & inc. in the following K,Cl,Urea.
So the cause of this is:
a. hyponatremia
b. hyperkalemia
c. hyperchloremia
d. uremia
1047-old pt ,she have MI and complicated with ventricula tachycardia ,
, then from that time recive Buspirone
he came with fatige >>>>normotinsive , pulse was 65
what INX must to be done

a- thyroid function
b- liver and thyroid

1048- patient with hypopigmented macules.loss of sensation.thickend


nerves.diagnosis was leprosy.which type
tuberculoid
lepromatous
borderline

Loss of sensation is a feature of tuberculoid leprosy, unlike lepromatous


leprosy, in which sensation is preserved.)
Emedicine

1049- An active 64-year-old male complains of dysphagia.


Endoscopic biopsy confirms esophageal squamous cell carcinoma. A
predisposing factor to this condition is:

230
a) Hiatus hernia
b) Achalasia
c) esophageal varices
d) Diffuse esophageal spsm
e) Mallory-Weiss syndrome

1050- ulcer on the nose with averted edges


basal cell ca
herper simplex
1051- Boy with nocturnal enurisis psychotherapy fail to show result yo
will sart him on :
Imipramin vassopressin
Imipramin guanfacin
Clonidin vasopressin
Clonidin guanfacin

1052- pt 2 para 2 gravida had pco took cyclic progestrone,she is ona


increased risk of
endometrial cancer
cervical dysplasia
hip fracture

_1053- smelling sputum increase with lying down + clubbing


1-bronchectasis
2- ba
3- pneumonia

1054-2.female presented to er with HCL burn on her face there was


partial thickness burn.management
irrigation with water
irrigation with soda bi carb
immidiate debridement

1055-young female with left sided abdominal pain.no dysuria or change


in bowel habit.history of hysterectomy 4yrs back but ovaries and tubes
were preserved.on ex abd tender but no guarding.inv show leucocytosis
and few pus cells in urine.there was also history of unprotected coitus
with multiple partners.
i didnot get the scenario well but i think it was salpingitis.
management
consult surgeon

231
oral antibiotics
diagnose as ulcerative colitis

_
1056-a picture of JVP graph to diagnose.patient had low vol pulse,low
resting bp.no murmr.pedal edema.
constrictive pericarditis
tricuspid regurg
tricuspid stenosis
pulmonary hypertension

_
1057-treatment of psoriasis:
topical steroid

_
1058-picture of pelvic x ray what is diagnosis
normal
paget disease
spondylitis
osteoporosis

1059- pt with vomiting, constipation pain and distention past history of


appendectomy 7 month ago
dX

1- mechanical IO
2- ileus

1060- In Pedia ,,IV fluid (LR) can be given at


age: A 3 months
B 8 months
C 12 months
D 24 months
1061- old aged female with atypical squamous cells of undetermined
significance (ASCUS) on pap smear, started 30 day ttt with estrogen
& told her 2come back after 1 weak, & still +ve again on pap smear,
what's next:
- vaginal biopsy
- endometrial biopsy
- syphilis serology
coloposcopy and cervical biopsy cone

232
1062-advise to pt. to avoid food high in cholestrol
- liver
chicken
tuna
egg white

1063- pt with meniere disease advised to take


low salt no caffiene
low salt high caffiene
high salt no caffiene
high salt low caffeine

1064- most common psychiatric condition come with mania ?


paranoid
grandiosity

1065- bad breath smell with seek like structure, no dental caries & Ix
are normal, what's the likely cause:
- cryptic tonsillitis
- Sojreen's synd.

1066- 24 y. Female with new Dx of DM2, she weared glasses for 10


years, you will advice her to follow ophthalmic clinic every:
- 6 months
- 12 months
- 5 years
- 10 years
1067- - normal child ,he want to walking , he have brother dead after
walking ,
what of the following must be excluded before walking ???

a-PDA
b-VSD
C-hypertrophic cardiomyopathy
D-!!!

1068- The best ttt for binge eating disorder:


- cognitive - behavioral therapy
- problem - solving therapy
- interpersonal therapy

233
1069- female pt ,KCO rheumatic heart , diastolic murmur ,complain of
aphasia and hemiplegia ,
what will you do to find the >>>etiology<<< of this stroke:
a-MR angiography
b-Non-contrast CT
c-ECHO
D-ECG
E-carotid doppler
1070- A case of a patient with polycythemia and develop itching after
taking a bath ..

a. increase histamine sensitivity


b. abnormal histamine release
1071- pt with DM and obese ,plane to reduce his wt is :
a.decrease calori intake in day time
b.decrease calori and increase fat
c.decrease by 500 kcal/kg per week
d.decrease 800 per day

1072- the most common complication following hemorrhoidectomy is :


a. fecal impaction
b. bleeding
c. urinary retention
d. infection

1073- Pediatric came to you in ER with wheezing, dyspnea, muscle


contraction (most probably asthma), best to give initially is:
1. theophylline
2. Albuterol nebulizers
3. oral steroids
4. oxygen

1074- ld man presented to u complaining of rectal pain mostly at night


with itching .. what is the Dx:

a. Hemorrhoids
b. Gay bowel syndrome
c. Proctalgia fugax

1075- 28 yrs old known case of sickle cell anemia hospitalized two
times in the last two months because of abdominal pain, this time he
present with abdominal pain, back pain, and chest pain.. what will you
234
do:

235
a. Hospitalize the patient and give him analgesics and observe him
b. give him IVF and treat him as an outpaient
c. Referred the patient to Tertiary center specialized in his problem
d. Give analgesics
e. blood transfusion

1076- diabetic pstient with ulcer in foot , not healing , not infected , high
? blood glucose
a- high blood glucose stimulate bacteria to grow
b- decrease phagocytosis
c- dec. Immune system

1077- on flow cytometric analysis of a sample of fetal thymus a certain


population of cells is identified that is positive for both cd4 and cd8 cell
surface Antigens . These cells are best characterised as which of the
following cells ?!
A- immature cortical T lymphocyte .
B- mature cytotoxic T lymphocyte.
C- mature helper T lymphocyte.
D- antigen presenting cells.
E- Natural killer (NK) cells .
1078- A case of a man who ride a motorcycle and make an accident
then had a basal skull fracture .. he developed a loss of taste, and loss
of sensation in the Anterior 2/3 of the tongue, and deviation of the angle
of mouth .. if u will choose one nerve injury .. which nerve u will choose:

a. CN I (Olfactory)
b. CN III (Oculomotor)
c. CN V (Trigemenal)
d. CN VI (Abducens)
e. CN VII (Facial)

1079- dysfunctional uterine bleeding :


a. most common in postmeneposal women
b.adolesent

1080- celiac disease severe form involve


1. proximal part of small intestine
2. distal part of small intestine
3. proximal part of large intestine
4. distal part of large intestine
236
1081- pt with moderatly sever acne valgarus best ttt

oral isotretinoin

topical retinoid

topical clindamycine

oral antibiotic
1082- Contraindicated in acute glaucoma management:
a. Pilocarpine
b. Timolol not sure
c. B-blockers, CA inhibitors, NSAID, Mannitol
d. Diprovin ??
e.

contraindicated drugs(topiramate antiepliptic ,steroid eye, tricyclic


antidepressant,Parkinson drugs,atropine,antispasmolytic ,antipsychotic

D......it is DIPIVEFRIN (alpha agonist). Used in open angle glucoma and


contraindicated in acute angle glaucoma
The same question came to my friend and the choices were: Pilocarpine -
Timolol - Dorzolamide - Flvin. (and this was the answer)

1083- a colorectal carcinoma that invades the submucosa and has


two positive lymph nodes and no metastasis is :
a. stage 1
b. stage
2 c. stage
3
d. stage 4
1084- long case Pt.obese and newly Dx by FBS> 126 with long list of
lab come to me in the exam screen all normal including liver function
test On examination: pt had palpaple midly enlarge liver what you will
give him:
a) Biguanieds
b) Sulphanylurea

1085- what is the definition of insomnia?


inability to have immediate sleep when you are very tired
disturbance of sleep cycle??
inability to get sleep even if you take medication
237
disturbance of sleep rhythm that person sleepy at daytime and insomnic
at night

1086- which of the following is the first test that should be performed in
a patient with lower GIT bleeding?
a. nasogastric aspiration
b. anoscopy
c. proctoscopy
d. colonoscopy

1087-pt with constipation . He had previous surgery in the


past ( there is X- ray )

a- surgery for
obstruction b-rectal
decompression
c- treatment of Ileus

1088- yr old in his normal state of health presented with decrease visual
acuity bilaterally without any defect in visual field his VA Rt eye= 20/100
VA Lt eye=20/160 fundoscopic exam showed early signs of cataract
and drusen with irregular pigmentations. No macular edema or
neovasculirization. The appropriate action beside antioxidants and Zn
is:
a. Refer the pt for emergency laser therapy
b. Refere the pt for cataract surgery
c. See the patient next month
d. No need to do anything

1089- Male patient present with exercise intolerance, HG is 9 and MCV


is 78 and positive fecal occult test. Upper GI scope show chronic
gastritis. How u treat him:
• Oral iron?
• IV iron
• Blood transfusion
1090- Newborn, 28 w, 900 g, successfully resuscitated in delivery room ,
then in NICU he had PH 7.35, Pco2 42, Po2 63, what to do first:
• IV bicarbonate infusion
• Phototherapy
• IV vancomycin
• Start entral feeding
glucose infusion

238
1091- female pt ,KCO rheumatic heart , diastolic murmur ,complain of
aphasia and hemiplegia ,
what will you do to find the >>>etiology<<< of this stroke:
a-MR angiography
b-Non-contrast CT
c-ECHO
D-ECG

1092- female with positive urine pregnancy test at home what next to do:
Sreum beta HCG
CBC
1093- Infant in respiratory distress ,hypercapnia , acidosis & have
rhinitis , persistent cough +ve aglutenation test & the doctor treat him
by ribavirin DX
pertusus
RSV
1094- ptn with pharyngo tonsilities he took antibiotic and improved in 2
days <the full course of antibiotic should be for:
1- 5 days
2-7 days
3- 14 days
4- 10days
1095- 43y old female with irregular menses 3m back & 1-2d
spotting what is
next to do:
US
Human chorionic gonadotropin
Placental ,,,,,,,,, ,,,,,,,,,,,,,,,,,,,
FSH
LH
1096- Drug that will delay need of surgery in AR:
a. digoxin
b. verapamil
c. nefidipine
d. enalpril
1097-Notching on the lower edges of the fourth to the ninth ribs indicate enlarged
intercostal arteries eroding the lower border of the ribs in cases of
coarctation of the aorta
1098-- An outbreak of TB as a prophylaxis you should give :
a) Give BCG
vaccine b)
Rifampicine
c) Tetracycline
d) H. influenza vaccine
239
1099-n old man who had stable angina the following is correct except:
a) angina will last less than 10 min
b) occur on exertion
c) no enzymes will be elevated
d) will be associated with loss of consciousness

_
1100-5 yr old adopted child their recently parents brought him to you with white
nasal discharge. He is known case of SCA. What you will do to him:
a) Give prophylactic penicillin

_
1101-a patient with acne of several appearances open .. closed .. red .. it is most
likely:
a) obstructive
b) inflammatory
1102-70 yr old presented with wt loss, fatigue, anemia , upper quadtrant pain without
any previous history, the stool sowed high fat he is a known somker:
a) Acute pancreatitis
b) Chronic pancreatitis
c) Pancreatic carcinoma
1103- ATRIBUTED risk:
Difference btw incidence in exposed and incidence in non
exposed 1104-tricky case:: young female with pain during
menstruation and mood ‫طلعت مضبوط قريتها لما بس‬ PMS ‫على قصدهم حسبت وهله لول‬
‫الكيس في ركزوا‬
A) 1 ry dysmenorrheal
1105-gush of bleeding in 9th month:
a) Placenta privia
1106-best way in advance age to increase bone density in OA:
a) Walking ................
b) High repletion and low resistance

1107-pt ttt from endocarditis likely of recurrence :


12%
1108- psycatric pt see alien talke to her and insertion of idea:
Start antipsychotic ttt
1109- best way to kill dust mite
Wash sheet and clothes in hot water

1110- which one make you relief when you aspirate a Brest mass:
Clear serous fluid in the needle

1111- mastitis with lactation:


Cont. brest feeding
1112-pt. with celiac spore best diet:
Gluten free
202
1113-Farmer with allergic conjunctivitis in spring and he can't avoid working what to
advice to do at night
a) Cold eye compression sure 
b) Other not include antihistaminic
1114-To measure the cognition in old
pt:
a) Clock test
b) Memory test my answer but I think A is the right answer u
c) Other I did't remember
1115-0-Blue swelling below tung or inside lower
lip: A. Ranula

1116-pt with elevated TSH and elevated


T3,T4 2ry hyperthyroidism

1117- Reflux
esophegitis: Mimic
heart dis.

1118- Barrit esophagus how to precced:


f/u endoscopy for.......................to look for change in metaplasia
‫ينسيت مدة الفولو اب وينسيت الختيارات بس دورت ع االجابه وطلعت صحيحه حتى االجابه مو بالنص مكتوبه بس في اختيار شبه اللي‬
‫كتبتو‬
1119-- About lyme to educate parents
Wear long clothes to avoid bite my
answer

1120- Child with elbow and knee pain with decrease ROM ..in the back < 1cm
red macule
Lyme dis.
‫فكرت اينه اللي في الظهر عضة الجرثومه وهللا اعلم‬
1121-- 19 y.o yong male with good body and well muscular with bad mouth
brith c/o
Acne:
He use anabolic steroid
1122- Child his mother let him to go to bathroom befor sleeping and avoid drinking
befor sleep this mangment of:
Enuresis
1123-Pt with meningitis what to give initially:
Penicillin

203
1124- pt with liver dis. Jaundice Bx showed fibrosis which diet is good for him:
Low protein diet not sure

204
1125-2months infant with white plaque on tongue and greasy ,past h/o
clamydia conjunctivitis after birth treated by clinamycin what is ttt:
a. Oral nystatin
b. Topical steroids
c. Topical acyclovair
d. Oral tetracycline
1126-psycatric pt . with liver imparment best to give ?:
a) lithum
1127- man walking in street and saying bad words to stranger , he is
not aware of his conditiond he kept doing that as if he asked to , what
is the description :
a. flight of idea
b. insertion of idea
c. loosening of association
‫شكيت في الجابه ب بسبب المكتوب بالحمر بس اتخترت الجابه التخيره؟؟؟‬
1128-a mother came with her son who is 7 years old with poor
concentration. Lack of intelligence and play and repeat some of his
action .....................forget the rest
a) Autism
b) Hyper active disorder
‫في شي في السؤال تخلني اشك في الجابه التانيه بس نسيت اش هي‬
1129-pt with vesicle in mouth with gingivits and also vesicle in arm and
leg most likely cause
a. HSV type 1
b. HSV type 2
1130-early symptom of heart failure :
a) Orthopnia
b) PND
c) Palpitation
d) Chest pain
1131-young adult obese with snoring when sleeping and some time get up from
sleeping
Best mangment :
a) Reduce wt
b) tonsillectomy

1132- young adult in endemic area cripitation bilaterally with monopheseal sound in
auscultation what to give vaccination :
a) Hemophlous influenza
b) Meningococcal

1133- pt with frothy hemoptysis, palpitation >>>>>>>>>>> forget the rest it's
long scenario
a) Mitral stenosis
b) Congestive heart failure
c) CAD

205
Mitral stenosis cause frothy hemoptysis on cardiac exam>>middiastolic murmur in
apex and malar rash on cheek
Congestive heart failure is an important cause of pink frothy hemoptysis on cardiac exam>> S3+ basal
crept.
1134-most benign vaginal bleeding:
a)Cervical polyp
b)Myomyoma
c)Endometrial hyperplasia
d)I forget the exact sentience but it's related to dryness in menopausal
women (atrophic vaginitis)
‫كل الختيارات باثولواجيكال ما عدا الخير‬
1135- non hormonal ttt of premenopausal flushes:
paroxitine
1136-mother after delivery have bad mood , depression , crying a lot for only
1 week , but she is o.k now Dx:
A. Maternal blues (transiet condition)
B. Post partum psychosis
1137- pt with pruritic foliclitis best ttt:
A. Local antibiotic xx

1138- pt. e hair loss in the rt. Temporal area. On examination there is elevated
mass with come spacious yellow crust:
‫مني فاكره الكيس بالظبط‬
.cyst Spacious A ‫حااجه م زي كدا‬
B. Tricotelomania
1139-young male athletes with palpitation and.................maybe syncope and the fail
to do for
Him something.......i'm sorry u:
A. Hypertrophic cardiomyopathy

1140- which one of the congenital heart dis. Have least complication with...................I
think endocarditis:
A. ASD
B. VSD
C. PDA
1141- child fall from stairs came with mild injury to the nose, no bleeding
and edema in the nasal sputum , ttt :
a- Nasal packing
b- Reassure
c- Analgesia
d- Refer to ENT

206
1142- eye exam. corneal ulceration. her symptoms freq. repeated..
which on of the following is triggring for recurrence of her symptoms:
1- Dusts & pollen
2- HTN & hyperglycemia
3- dark and driving at night
4- ultraviolet light & stress ( this is the answer 100 % )

1143- What is special about placenta abruption:

a. Vaginal bleed
b. Fetal distress
c. Uterus pain and back pain
d. Abnormal uterine contraction
1144- 24 Y/o man presented with 4 month Hx of diarrhea with streaks of
blood & mucous. Ulcerative colitis was confirmed by colonoscopy. The
initial therapy for this patient:
a)oral corticostreiod
b)azathioprine
c)infleximabe
d)5-Aminosalicylic acid
e)Sulfasalazine
1145- I study done on 10,000 people for about 3 years in the beginning
of the study 3,000 developed the disease and 1,000 on the end of the
study what is the incidence:
100
12.5
10.5
0.1
1146- 45 years old female came to ER with acutely swollen knee +
ballotment patella .. The most important to do is:
MRI of the
knee Aspiration
Complete blood count
Rhumatoid factor
1147- peritioial lavage when to say the amount is suffusion
: 2 l blood
1000 wbs \ rbs
500 wbs
1148- 19- What best explain coronary artery
disease: a- Noatherosclerosis
b- Fatty deposition with widening of artery
c- Atherosclerosis with widening of artery
1150- Incidence is calculated by the number of:
A. Old cases during the study period.
207
B. New cases during the study period.
C. New cases at a point in time.
D. Old cases at a point in time.
E. Existing cases at a study period.
1151-9- secondary prevention one true:
a- physician screening quetionaire about the use of tobacco is sufficient
b- the screening of colon cancer is insufficient
c- the screening of breast cancer is
decreasing d-
1152-Which drug contra indication in cluster headache ?
a. Buperbion
b. Lithum
c. valium

1153-Most common cause of recurrent tonsillitis is :


- Group a beta hemolytic streptococcus
- The other choises are

virusis.. parainfelunsa

rhinovirus..
http://en.wikipedia.org/wiki/Rhinovirus

1154-Child with leukemia he has septicemia from the venous line the
organism is:
a) E coli
b) GBS
c) Pseudomonas

1155-Baby born & discharge with his mother , 3 weeks later he started
to develop difficulty in breathing & become cyanotic what is most likely
DX :
a- VSD
b- Hypoplastic left ventricle
c- Coarctaion of aorta
d- Subaortic hypertrophy

1156-Man who is having a severe pain on his big toe with knee pain
and examination revealed negative perferingent crystals:
a) uric acid deposit secondary to synovial fluid over saturation
b) Ca pyrophosphate secondary to synovial fluid over saturation

208
1157- 6 years old child presents with straddling gait and in ability to
stand or walk without support, he is irritable with vomiting 3 times, he
has a history of chickenpox 3 weeks ago. O/E all are normal except
resistance when trying to flex the neck, what is the most likely
diagnosis:
• Fradrich's ataxia
• Acute cerebellar ataxia
• Meningioecephalitis
• Gullian Barre syndrome
http://en.wikipedia.org/wiki/Acute_cerebellar_ataxia_of_childhood#Etiology

1158-a man fell down from the ladder, c/o SOB ( and i think cyanosis ),
on exam breath sounds are decreased even in the right side ( this is
how they wrote it !! ), u will do :
needle thoracotomy
insert endotracheal tube
other options i forgot

1159- Which of the following is true regarding antepartum (third trimester


) hemorrhage :
a- Can be caused by polyhydrominos
b- Rare to be associated with hypofibrogenemia
c- Cervical problems are a major cause
1160-case infant has genital rash ( the rash spares genital fold ) not
response to antibiotics , most likely Dx;
A-candida albicans
b-napkin dermitis (diaper dermatitis)
c-contact dermatitis
d- atobic dermatitis
e- sebborich dermatitis
1161- Male patient complain of excruciating headache, awaken him
from sleep every night with burning sensation behind left eye,
lacrimation and nasal congestion. What is effective in treating him :

• Ergonavine
• Sumatriptan SC
• Methylprednisolone
• NSAID

1162-34 yo female presented with cough , dyspnea for months ,


exam show cervical adenopathy and hepatomegally . To confirm the
likely diagnosis you will do ..

209
1- liver biopsy

2- bronchoscopic lung biopsy

3-scalene nodal biopsy

4- ACEI level.

yes, lung biopsy to confirm the diagnosis of sarcoidosis.

1163-Fresh frozen plasma in what case ?


a- Hemophilia a
b- Hemophilia b
c- Von willbrand
d- Dic
e- Coagulopathy form liver disease

1164-which of the following cause hirsutism


? a- Anorexia

a- Digitalis

b- clomiphine citrate

d- OCP

_
1165- pt with epilepsy came with Lt shoulder pain , on examination
flattened contour of the shoulder, fixed adduction with internal
rotation .. ur DX ?!!?

a- Inferior dislocation
b-subacromal post Dislocation
c-subglenoid ant dislocation
d- subclavicle ant
dislocation e-sub…….. ant
dislocatio

1166-pt with asthma on daily steroid inhaler and short acting B2


agonist what category:
a- Mildintermittent
b- Mildpersistent

210
c- Moderate
d- Sever

1167-Pt with active hepatitis what medication should not to give


:
a- Ranitidine
b- Heparin
c- Atrovastsin

1168-A man who is having severe vomiting and diarrhea and now
developed leg cramps after receiving 3 liters of dextrose .. he is having:
a) hypokalemia
b) hyponatremia
c) hyperkalemia
d) hypernatremia

1169-Pneumococcal vaccine :
A not recommended in healthy child
B cant be given with MMR
C cant be given to child less than 2 years
D if given to sickler and exposed to infection has to take pencilin

1170-Long scenario for pt smokes for 35 y with 2 packets daily, before


3 days develop cough with yellow sputum, since 3 hours became blood
tinged sputum, X ray show opacification and filtration of rt hemithorax,
DX:
Bronchogenic CA
acute bronchitis
lobar pneumonia
1171-Post partum female with recurrent attack of hearing loss ,
which diagnosed as conductive hearing loss , on CT the is dehesion
in the of semi circular canal diagnosis >>>>
otosclerosis
miner's
Tuberus sclerosis

1172- True about dermatomyositis :


1-associated with inflammatory bowl
dz 2-indicate underlying malignancy
3-present as distal muscle weakness

211
1173- newborn presented with conjunctivitis and O.M , whats the
treatment
i guess this is a case of infection with chlamyedia intrauterine , they
asked about several AB
there is no dyoxycyclin nor erythromycin
1174- Differences Btw dementia and delirium (read about it
) i picked Amnesia !!
1175- about shoulder that is Adducted and internally rotated (what is
the mechanism of dislocation)
1-Anterior subclavicular
2- Ant
3-Post
4-POst
1176- anteriolateral placenta , term pregnancy , can't be felt when
examiner admit his finger through the Cervix :
1-Low set placenta ??
2-Marginal
3- normal
4- complete placenta
previa 1177- Tinea capitis
RX.
1- start Nystatin
2- wood's lamp (true)
1178- Rosacea case (redness patch on face with talangectasia ) what is
the ttt
Dyoxycyclin
1179- child smile at
1- at birth
2- 1month
3-2 months
4- 6 months

_
1180- pt has GERD for 5 years , now EGD reveals >> columnar cell
surrounded by Sq cell
1- Sq .c.c
2- Adenocarcnoma
3-barret esophgous
1181- old pt , has loin pain , U/S reveals bilateral hydroneprosis , whats
the cause :
1-prostate cancer
2- bladder cancer
3- urethral stricture
1182- infertile women for 3 years with dysparunia

212
1- endometritis

213
2- Salpengitis
3- endometriosis (True)
1183-difficult consultation :
1- use medical
term 2- open ended
Q
3-close Ended Q = True
4-
1184- hematology case … prophral blood smear reveals target cell
=SCD
1185-old pt with progressive weakness of hand grip , dysphagia ,
….
1- MG dz
2- Mysthenia gravis sx .
1186- male singer with colon cancer stage B2 ; which of
the following correct ?
a- no lymph node
metastases b-one lymph
node metastasis c-2 ===
d-lymph node metastasis + distant metastasis
1187- Young boy presented to the ER with inguinal mass, pain and
vomiting. O/E the mass is tender to touch, erythemetous skin over
scrotum, (blue dotes) in the pole of testis, intact cremasteric reflex , Dx
is :
a- Testicular torsion
b- Testicular hematoma
c- Incarcerated herniad-torsion appendix of testis
1188- A middle age man presented with severe headache after heavy
lifting objects. His BP was high. He was fully conscious. Examination
was otherwise normal. the most likely diagnosis is:
a) Subarachnoid hemorrhage
b) Central HTN
c) Tension headache
d) Migraine??
e) Intracerebral hemorrhage

1189- PT WITH UTI ALLERGIC TO SULFA AND PENICILLIN ?


NITROFUNTON
CEPHLAXIN-
SMT

1190- bad breath smell with seek like structure, no dental caries & Ix are
normal, what's the likely cause:
- cryptic tonsillitis
214
- Sojreen's synd.

215
1191- use of antibiotic in acne :
A - to prevent spread
B- to decrease scaring
1192- a 38 yrs old female … came to you at your office and her pap
smear report was unsatisfactory for evaluation .. the best action is

a- consider it normal & D/C the pt.


b- Repeat it immediately
c- Repeat it as soon as possible
d- Repeat it after 6 months if considered low risk
e- Repeat it after 1 year if no risk
1193- Mass in the upper back .. with punctum and releasing
white frothy material…
a- It's likely to be infected and Antibiotic must be given before
anything
b- Steroid will decrease its size
c- It can be treated with cryotherapy
d- It must be removed as a whole to keep the dermis intact??
1195- A pregnant lady full term presented with agitation and change
level of
consciousness she is having difficulty breathing:
a. Pulmonary embolism??
b. Pulmonary edema
c. Amniotic fluid embolism
1196-Pt presented with nausea and vomiting and nystagmus with
tinnitus and inability
to walk unless he concentrate well on a target object. His Cerebellar
function is
intact:
a. Benign positional vertigo
b. meniere's disease
c. vestibular neuritis

1197-tympanic mem. perforation in cases of cholesteotoma are


commonly situated in :
a- anterior part of mem. tensa
b- centre of mem. tensa
c- posterior superior segment
d- posterior inferior segment of the tympanic membrane

1198-Peripheral neuropathy can occurin all except:


a) Lead poisoning
216
b) DM
c) Gentamycin
d) INH
1199-child with 2 * 2 cm hair loss at the temporal area , normal
examination , microscopic examination of hairs arround the area
show clubbed and attenuated hairs , the diagnosis is :
a- tinea capitus
b- alopecia
areata c-
Trichotillomania
d- Telogen Effluvium
e-?
1200- What is the most important in councling
a. Exclude physical illness
b. Establishing rabbot
c. Family
d. Schedule appointement

1201-year old had an episode of rheumatic fever without any defect to


the heart. The
patient need to take the antibiotic prophylaxis for how long:
a. 5 months
b. 6 years
c. 15 years

IF :Rheumatic fever with carditis but no residual heart disease (no valvular
disease†) 10 years or until age 21 years (whichever is longer) IF: Rheumatic
fever without
carditis
5 years or until age 21 years (whichever is longer)
http://www.aafp.org/afp/2010/0201/p346.html
1202- Pt daily asthma , nothing at night, using herbal for 2
months with no improvement :
inhalation salbutamol(the best answer)
, high
dose steroid inhaler
, ipratrubium

1203- when an a cyanotic middle age adult has roentgenographic


evidence of enlarged pulmonary arteries and increased lung markings,
most likely diagnosis is:
a) Ventricular septal defect(according to sle studing)
b) Coarctation of aorta
c) Pulmonary valvular stenosis
d) Atrial septal defect
217
e) Truncus arteriosus

218
1204-all of the following will improve the patient compliance except:
a-making the appointment convenient?
b-simplify the regimen
c-writing the instructions clearly
d-tell about the danger of missing
doses e-involve the patient as active
participant
1205- drug of choice of genaralized anexity dis. Is:
-acetalopram
-bubropione
-buspirone
-beat blocker
1206- Uric acid in body how the body removed by
a-increase co2 execreation of uric acid in urin
b-increase metabolism of uric acid in
liver c-execretion of uric acid by lung

In humans and higher primates, uric acid is the final oxidation (breakdown) product
of purine metabolism and is excreted in urine

1207- A man travelled to Indonesia and had rice and cold water and ice
cream. He's now having severe watery diarrhea and is severely
dehydrated. Most likely he has:
A. V. Cholera
B. C. Difficile
C. C. Perfringens
D. Dysentery?
E. Shigella
1208- Newborn has vomiting after every meal intake. The examination
was normal and the only abnormality was dehydration. No other
clinical signs. No tests ordered yet. What will you do?
a- Order abdominal CT
b- Reassure the parents
c- Refer to GS
d- Discharge on ORS
1209- child with SOB and runny nose came with fever (38 ) all the sign
of respiratory distress there .. There is diffuse wheezing on the chest
with prolonged expiration and inspiratory cracles ,, diagnosis:

-viral pneumonia
-bronchiolitis
-croup
-bacterial pneumonia
1210- pt came with fatigue , w.t loss and diarrhea .. He recived a blood
transfusion when he was in kenea .. He has low grad fever .. The vitals
219
are stable .. Skin EX. There is contagiosum mollosum in groin ( i guess
it written like this ) .. There is generalized lymphadenopathy and
palpable liver ,, what is the diagnosis:

-secondary syphilis
-persistent chronic hepatitis B
-HIV
-acute lymphoma.
1211- the best description of the lesion in herpes :

-soft tender chancer


-firm non-tender chancer
-raised tender papule
-vesicle with fluid material
-................
1212- penis numbness after sitting for long time
Pudendal Nerve Entrapment

1213- What is the organism that will growth in the agar in a sample
from a cat bite ?!
salmonella
bacteroid species ??
streptococcus
1214- 5 y/o child had abdominal blunt trauma, doctor confirm presence
of intramural hematoma in 1st and 2nd parts of duodenum, high
amylase, Mx ?
a- CT-guided hematoma drainage
b- duodenal resection
c- exploratory laparoscopy
d- don't remember.. I think conservative

1215- pt with recent Hx of URTI , develop sever conj. Injection


with redness, tearing , photophopia ,
So, what is TTT ?
a)Topical ABx
b) Topical acyclovire
c) Oral
acyclovire d)
Topical steroid

photophopia is caused by adenovirus treated by topical steroid

220
1216- Pt came with mild abdominal pain, headache, oral ulcer, joint
pain,thrombophlebitis of leg
Ulcerative cholitis
SLE
Behchet

1217- Patient with echolalia, echopraxia, poor hygiene, insomnia, and


weird postures. Treatment? (catatonia)
A. Lithium

Benzodia
szepine arethefirstlineoftreatment

1218- 43 yr old lady complaint about non itchy;white nonsmelling


vaginal discharge after intercourse;she z nt using any contraceptive
or vag douche;what z diag??
A;prescibe azithromycine
b;local steroids
c;local antifungal
d;vaginal douche
e;do nothing
1219- If a patient of Magrine head ache has nt been treated;which
condition do u suspect the patient will develope??
A.hearing loss
b.depression
c.dysphagia
d.loss of vision
1220- a lady patient otherwise healthy complaint a hissing sound n her
ears at night during sleeping;her bedroom is sound proof and no noise
coming from outside.
Diagnosis?
A;migrine
b;otosclerosis
c;otitis media with effusion
d;tinnitis
1221- acute diarrhea with epithelial
infiltration E- coli
Salmonella
Cholera
Rota virus
Shigella
1222- Boy presented with painless neck mass, 5-week history of fatigue,
generalized pruritis, and mild cough. Dx?
A. Hodgkin's lymphoma

221
B. Lyme's
C. Infectious mono
1223- what is the organism hat cause meningitis in college dormities :
1-h.influenza
2- nisseria gonorrhea
3- strep. pneumonia
4- staph. aureus
Neisseria meningitidis bacteria (meningococcal disease)
1224- a 19 yr sexually active lady came for her annual check up:she z
otherwise healthy using no contraceptive;her pap smear and all
investigations are normal;what will u suggest regarding her next check
up???
A;afer 6 months
b;after 1 yr
c;after 3 yrs
d:after 5 yrs

1225- 6 yr old school going boy complaaint abt itchy scalp;n school his
10 friends have the same problem:wt z ur diag??
A;lice (Pediculus humanus capitis)
b;tinea capitis
c:seborric dermatitis
d:scabies

1226- Nonmedical treatment of premature ejaculation??


the use of acupuncture
1227- A 54 YO female with chronic pelvic pain is found to have a
right sided ovarian mass. After the initial evaluation, surgery is
planned to remove the mass. To avoid excessive bleeding during the
surgery , the surgeon should ligate which of the following structures?
A) Round ligament
B) Suspensory ligament
C) Ovarian ligament
D) Transverse Cervical ligament
E) Mesosalpinx
Suspensor ligament of ovary contains the ovarian artery, ovarian vein,[1] ovarian
plexus,[4] and lymphatic vessels.[3]

1228- leukemia with blast cells:


AML
ALL
222
CML
CLL

1229-cloboma when to do the


operation>> 1stwk,
3month,
1 year
1230- Table with investigation
Na 112
Osmolality 311 low
What is the diagnosis?
a- Conn’ssyndrome
b- Cushingsyndrome
c- SIADH(the only choice with low na)
d- Diabetes insipidus

1231- Female wants a temporary contraceptive method, which one is


recommended by research?
A) OCP
B) IUCD
1232- pt sustained major traum came to Er 1st thing to do

open air way give 2 breath


open airway remove foreign body
2 breath fowlled by chest compression
chest com after feeling pulse
1233- PT DYSMENORRIA – DYSPARUNIA –INVERTED UTERUS ???????
ADENOMYOSIS
-ENDOMETRIOSIS
-UTERINE LIOMATA-
UTERINE
CARCINOMA
1234-In moderate to severe asthmatic patient , u will find all the following
EXCEPT :
A PO2 <60
B PCO2 > 60
C low HCO3
D IV hydrocortisone will relieve symptoms after few hours
E dehydration
1235-12 year old female , non pruritic annular eruption in the right foot
for 8 months , looks pale and not scaling , no response to 6 weeks of
miconazole
A discoid lupus erythramotosis

223
B erythema nodosum
C tinea corporis
D granulomatous annulare
E choricum marginatum??
1236-A man who bought a cat and now developed watery
discharge from his eyes he is having:

a) Allergic conjuctivitis
b) Atopic dermatitis
c) cat scratch disease
1237-Senario about wilson (kayser-fleischer ring, low ceruloplasmin)Rx:
A)desferrioxamine
B)penicillamine

1238-effective ttt of mastalgia


? a- Caffeine
b- OCP “ “
c- tamoxifen
d-danazol
1239-Pt with abdominal pain and distension with vomiting and
constipation. He has mild symptoms of dehydration. There is
evidence of air in the rectum. The Rx:
a. Rectal decompression with IV antibiotics
b. Nasogastric tube with IV isotonic fluid
c. Systemic antibiotics

1240-5 months old baby , in ER with sudden abdominal pain , pain


last 2-3 min with intervals of 10-15 mins between each attack :
A- intussesption
B- infantile colic
C- appendicitis
1241-Patient with a scenario going with liver cirrhosis with acsites, diet
instructions:
High carbs, low protein
Sodium restriction

1242- which of the following is used in minor


burn: a- apply cold water
b- apply room temperature water??
c- debridement
1243-Pt presented with severe hypothyroidism & serum sodium =
108. What do u do?
a- Intubate, give 3% sodoium then treat hypothyroidism status
224
b- treat hypothyroidim & monitor S.NA level every 6 hours
c- Give 3% sodium, hydrocortisone & treat hypothyroidism status

1244-pt. with primary biliary chirhosis wich drug helps the histopathology
of the liver:
- Steroid
- Interferon
- Ursodiol

_
1245-patient complaining of gradual loss of vision & now he can only
identify light. Which of the following is the LEAST cause of the problem
? a retinal detachement
b central retinal artery embolism
?? c vitreous hemorrhage
d retinitis pigmentosa
e retrobulbar neuritis
1246- Patient with DM and HTN, gradually decreasing vision. Eye exam
shows maculopathy, Treatment:
A. Panretinal photocoagulation
B. Photocoagulation of macular area
1247- Child presented with gum and nose bleeding and bruising all over
the body after an episode of URTI. Dx:
1) Henoch Scholein Purpura
2) Idiopathic thrombocytopenic purpura
3) Vitamin K deficiency
4) Hemophilia

1248-Patient on isoniazid for TB prophylaxis, what test should be


regularly done:
1) Spirometry
2) LFT
3) RFT

1249-Child presented with jaundice, vomiting, hepatomegaly.. etc. What


hepatitis virus is more likely to be the cause:
1) A
2) B
3) C
4) D

225
1250- Regarding strep pharyngitis: same as q13 put choices are
complete
1) No treatment should be given until strep infection is proven.
2) Treatment has no effect on rapidity of solution of infection
3) Treatment prevents post-strep glomerulonephritis
4) Treatment can be postponed for 9 days
5) Clindamycin is the drug of choice

1251-. What could cause painful vision loss:


1) Acute close angle glaucoma
2) Retinal detachment
3) Retinal vein occlusion
4) Retinal artery occlusion
1252- Female patient with hirsutism, obesity, infertility. US shows
multiple ovarian follicles. Dx:
1) Klinefelter’s syndrome
2) Asherman’s syndrome
3) Kallman syndrome
4) Stein-Leventhal syndrome other name of PSO
1253-. HbA1c is useful in:
1) Adjustment of insulin
2) Monitoring diabetic control on day-to-day basis
3) Longer term diabetic control monitoring
1254-. Child with iron toxicity, best way of management:
1) Gastric lavage
2) Ipecac syrup
3) Magnesium citra
Forgot the rest of choices, but there was no deferroxamine or charcoal.

1255- Patient presented with typical symptoms of hyperthyroidism.


What’s the most effective and rapid way to relieve symptoms:
1) Propranolol
2) PTU
3) Radioactive iodine
4) Surgery
1256- Young patient with unremarkable medical history presented with
SOB, wheeze, long expiratory phase. Initial management:
1) Short acting B agonist inhaler
2) Ipratropium
3) Steroids

226
4) Diuretic

1257-Patient with moderate persistent BA, on short acting B agonist


and low dose steroid inhaler. What will be the next step:
1) Add long acting B agonist to steroid
2) Increase dose of steroid
3) Theophylline

4) Ipratropium

1258-young male has a painless mass in the testis that is increasing


with time what is your advice:

US and consult surgeon

1259-drug binds to the bile and prevent its reabsorption:

cholystramine

1260-female patient on antiepliptic drugs wants to become pregnant


what well you tell her regarding eplipsy:

use of antiepliptic has risk of fetal malformation

epliptic attacks affects the fetus

1261-pt has solid dysphagia best for diagnosis:

endoscopy +biopsy

1262-pt has DM2 and HTN an CCB+metformin+glyburide+statin still


having increase BP what is your advice:

add ARBs

increase CCB

dose start thiazide

1263-In cervical LNs there are well differentiated thyroid cells, during
operation you find no lesion on thyroid what will you do next
A. Total thyroidectomy
B. Total thyoidectomy + radical cervical LNs dissection
C. Total thyroidectomy + specific LNs dissection
D. Thyoid lobectomy with -----

227
e-thyroid lobectomy and isthmusthectomy and removal of all local
enlarged lymph nodes (new choice not in alqaseem q)

1264-a young healthy male complain of sleep apnea on examination


there is only enlarged tonsils mangment:

adenoidectomy

reduce weight

1265-long scenario about young male with spoon shaped nails:

iron deficiency anemia

1266-Which of the following radiological feature is characteristic of


military TB?
Sparing of lung
apices Pleural
effusion Septal lines
Absence of glandular enlargement
Presence of small cavity

1267-Pt with sudden cardiac arrest the ECG showed no electrical


activities with
oscillation of QRS with different shapes. The underlying process is:
a. Atrial dysfunction
b. Ventricular dysfunction
c. Toxic ingestion
d. Metabolic cause
1268-An old patient presents with historyof uncontrolled HTN and DM .
his hand is big and tall in examination bulsitale abdominal mass. What
is the diagnosis?
A. Peptic ulter
B. GERD
C. aortic aneurysm
D. marfan syndrome??
1269-in Window period of hepatits B
1) HBc
2) HBs ag
3) HBs antibody

HBc Ab ( IgM )

228
1270-Children while he was playing a football , the ball hit
his hand from lateral fingers, after a while the children
complaing pain and swelling on those fingers and painful
middle finger with hyperextension of interphalengial joint,
swelling was more in the DIP and IP Joints , also , there was
pain on his palm, what is the most likely cause:
Rupture of profound ms in hand
Rupture of superfacial ms
Intra articular interphlengial joint fracture
Extra articular

rupture of flexor digitorum profundus

1271-

229

You might also like